Você está na página 1de 376

Pathology Question Bank

1.

General

2. Skin
3. Oral Cavity and Oropharynx
4. Mandible and Maxilla
5. Respiratory Tract
6. Mediastinum
7. Thyroid and Parathyroid
8. Gastrointestinal Tract
9. Salivary Glands
10. Liver
11. Gallbladder
12. Pancreas
13. Adrenal Gland
14. Urinary Tract
15. Male Reproductive System
16. Female Reproductive System
17. Breast
18. Lymph Nodes
19. Spleen
20. Bone and Joints
21. Soft Tissue
22. Peritoneum and Retroperitoneum
23. Cardiovascular System
24. Central Nervous System and Pituitary
25. Cytology
26. Grossing and Special Studies
27. Forensics

1. General
1.

What is released from within cytotoxic T cells that initiates apoptosis


a. Granzyme B
b. NBkf

2. Cytotoxic T cells can recognize antigens when they are presented on which of
the following
a. MHC I
b. MHC II
c. MHC III
3. During acute inflammation, histamine's effect is seen on which of the following
a. Post-capillary venules
b. Pre-capillary arterioles
c. Capillaries
4. Histamine is released from which cell
a. Mast cells
5. Which cell can regenerate after injury
a. Liver
b. Myocardium
c. Retina
d. Dermis
6. Anti-P53 (?) stops regenerating injured cells at what stage
7. Which immunoglobulin is released during acute bacterial infection, doesn't
cross the placenta, and

8. How does aneuploidy occur


9. In general, the mode ofinheritance ofdisease with structural protein
rearrangement (?) [not enzyme] and occurs in adulthood [not childhood] is
a. Autosomal dominant
b. Autosomal recessive
c. Mitochondrial
d. X-linked dominant
e. X-linked recessive
10. Which factor prevents cells from regenerating
11. Finding AFB within nerves indicates
a. Leprosy
12. Female patient with hypopigmented skin lesions on the face, arm, elbows, and
knee. Skin biopsy shows large cells containing mycobacterium
a. Lepromatous leprosy
b. Tuberculoid leprosy
c. Tb
13. 30 year old male had hip replacement surgery and there was blue-black
pigmentation on the cartilage, also his urine would turn dark ifleft standing,
what abnormality does he have
14. Active process by which blood goes out ofdilated vessels
a. Hyperemia
b. Congestion
15. Diabetic patient has periorbital infection with non-septated fungal hyphae,
what is the organism
16. Patient in a car accident with femur and arm fracture, developed dyspnea a
few days later in the hospital and died. What was the cause
a. Fat embolism
17. 50 year old male smoker with lung mass has weight gain, hyponatremia, and
hyperosmolar urine, what is the cause ofthis

18. 30 year old patient from an underdeveloped country with leg edema and
draining sinuses in the foot. The micro-organism is positive for gram and
partially for AFB and shows arrangement in branching filaments
a. Nocardia
b. Actinomyces esraieli
c. Cryptococcosis
d. Diphtheria
e. Pneucystis carinii
19. What is the causative organism of atypical pneumonia
a. Mycoplasma pneumonia
b. Klebsiella
c. Legionella
20. Function ofcaspases
a. Apoptosis
b. Coagulative necrosis
c. Caseaus necrosis
d. Pinocytosis
e. Phagocytosis
21. Acute phase proteins
a. IL 6
b. Bradykinin
c. Interferon
d. Histamine
e. Amyloid associated protein
22. Pro-apoptotic gene
a. P53
b. Bcl 2
c. Bcl X
23. All of the following cells can go from G0 G1 after abdominal trauma EXCEPT
a. Skeletal muscle
b. Smooth muscle
c. Liver cells
d. Endothelial cells

e. Fibroblasts
24. HLA B27 is found in
a. Ankylosing spondylitis
b. CREST
c. SLE
25. Rheumatoid artheritis joint lesions are due to (which is the most common
mediator involved in RA)
a. TNF
b. IL6
c. G-csf
d. Histamine
e. Bradykinin
26. Uncontrolled diabetes, glycosylation of amino acids will cause
a. Cataract
b. Peripheral neuropathy
c. Retinal microaneurysms
d. Atherogenesis
e. Relapse amyloid islets
27. Which of the following is associated with karrhiorexis and cell fragmentation
a. Viral hepatitis
b. Chronic alcoholic liver disease
c. Renal graft rejection
d. Barbiturate overdose
e. Brown atrophy of the heart
28. Before allograft transplantation we have to do MHCIImatching to prevent
a. Cytotoxic CD8 T cell activation

b. CD4 T helper cell activation


c. GVHD
29. Which of the following is considered the gatekeeper gene
a. P16
b. RB
c. P53
30. 30 year old African female domestic worker presented with lymphadenopathy
which shows granulomas microscopically, the causative organism is
a. Treponema pallidum
b. Gardinela vaginalis
c. HSV

d. Candida infection
31. Which of the following is an obligate intracellular organism
a. Pseudomonas auregenosa
b. Nisseria meningitides
c. Borriella burgderphori
d. Yersenia
e. Rickettsia
32. A newborn has a mass at the lower lumbar spinal region that transluminates
also has flattening ofthe skull, he is at risk for what
a. Hydrocephalus
b. Medulloblastoma
33. Young tall male patient presented with rupture ofan aortic aneurysm and
ocular dislocation (Marfan syndrome), which gene mutation does he have
a. FMR
b. Fibrillin
34. Hypersensitivity reaction with pronounced eosinophils is important in exposure
to
a. Inhaled dust
b. Liver flukes
35. Which of the following prevents collagen synthesis in wound healing
a. Infection
b. Diabetes
c. Vitamin C deficiency
d. Zinc deficiency
36. Which of the following is considered to be an oncogenic infectious agent
a. T cell lymphotropic virus
37. One of the following is oncovirus effect
a. T cell leukemia
b. Angiosarcoma ofthe liver
c. Pancreatic carcinoma
38. Which tumor may develop ifthere's a history of viral infection

39. Which of the following organs is affected by red infarction


a. Spleen
b. Heart
c. Liver
d. Kidney
e. Lung
40. Accumulation offluid in inflammation is due to
a. Endothelial contraction
b. Lymphatic obstruction
c. Arteriolar vasoconstriction
41. Example ofinfection with cross reactivity (antibody attacking the infection and
the body due to resemblance between them)
a. Rheumatic heart disease
42. Pain in appendicitis is due to
a. Bradykinin and PG
43. A 54 year old man experienced sudden onset of severe substernal chest pain
over a period of3 hours. An ECG showed changes consistent with MI. After
thrombolytic therapy his serum CK level increased. Which of the following
events most likely occurred following t-PA?
a. Reperfusion injury
b. Cellular regeneration
c. Chemical injury
d. Increased synthesis of CK
e. Myofiber atrophy
44. Black pigment in Alkaptonuria
a. Ochronosis

45. Ochronosis
a. Autosomal recessive
46. What are the changes that occur in the breast during pregnancy and lactation
a. Lobular hyperplasia
b. Atrophy
c. Ductal hyperplasia
47. What changes are seen in cells infected with HPV
a. Hyperplasia
b. Atrophy
c. Hypertrophy
d. Metaplasia
e. Carcinoma
48. Which tumor is associated with microbial infection
a. Liver cancer
b. Gastric lymphoma (small bowel/gastric cancer)
c. Skin cancer
d. Breast cancer
e. Lower urinary tract
f.

Bile duct cancer

49. A baby with trisomy 18, had micrognathia, abnormal kidney, rocky foot, and
overlapping fingers, the diagnosis is
a. Edward disease
b. Down syndrome
c. Patau syndrome
50. Patient with history ofmultiple ependymomas and meningiomas with defect in
chromosome 22q12
a. NF2
b. NF1
c. Digeorge disease
51. Which of the following deposits in the pancreas of a patient with
hemochromatosis
a. Iron

b. Copper
c. Lead
52. Patient admitted to the hospital with pancreatic carcinoma and develops right
leg swelling and tenderness, what's the cause
a. Recurrent venous thromsosis
53. What is the pathogenesis ofseptic shock
a. Vasodilation
b. TNF IL-1
54. Initial outcome of septic shock
a. Release ofTNF and IL-1
55. Which is the following is true for septic shock
a. Due to dissemination ofinfection in blood
b. Caused by G-ve cocci
c. Mortality rate 10-20%
d. Due to generalized vasodilation
e. Commonest form of shock
f.

Hypovolemia predisposes to septic shock

g. Caused by widespread infection


h. Lipopolysacharides in bacterial wall
56. Commonest cause of inherited mental retardation
a. Down syndrome
b. Fragile X syndrome
57. In alpha 1 antitrypsin deficiency why are there many PASpositive granules
a. Endoplasmic reticulum accumulation of abnormally folded protein due to
error in alpha-1-antitrypsin

b. Abnormal protein in lysosome


58. What is the prototype disease for lysosomal storage disease
a. Glycogen storage disease type III
b. Familial hypocholesterolemia
c. NFI
59. Male child with severe mental retardation, blindness, whorled appearance
within the neurons, motor dysfunction and cherry-red spot on retina. What
does he have?
a. Tay-sachs disease
b. Niemenn-Pick disease
60. After falling off a bicycle there is damage to the superficial layers ofthe skin
(superficial sloughing or epidermis). What is this defect called
a. Abrasion
b. Laceration
c. Contusion
61. What is released in the cytosol that initiates apoptosis
a. Cytochrome c
b. Caspase

62. Defective inflammation causes which ofthe following


a. Decreased susceptibility to infections
b. Allergic reactions
c. Autoimmune disease
d. Increased production of IL-6 and TNF
e. Delayed wound healing and tissue damage
63. What mediates anaphylaxis
a. IgE
64. Atopic dermatitis is due to
a. Local immediate hypersensitivity reaction
b. Cellular immunity
c. Delayed hypersensitivity
d. Immune-complex
e. Antibody cytotoxicity
65. What would delay wound healing
a. Corticosteroids
66. What would enhance wound healing
a. Approximation ofwound edges
67. When will a wound gain its maximum strength
a. 3 months
68. During the healing process, wound strength returns to normal
a. Never returns to normal
b. 1 month
c. 2 weeks
69. What does RT-PCR stand for
a. Reverse transcriptase PCR

b. Real time PCR


c. Recombination ofDNA from RNA
70. Amyloidosis characteristics
71. Ifa patient is suspected to have amyloidosis, what is the best investigation
a. Rectal biopsy and congo red stain
72. Female with vaginal discharge, smear shows multinucleated cells with groundglass nuclei. What is the organism?
a. HSV
b. CMV
c. Fungal
d. Dysplasia
73. Male with inguinal lymph node enlargement and genital ulcers, Donovan bodies
are seen on sections. What is the causative organism?
a. N gonorrhea (lymphogranuloma inguinale)
b. Syphilis
74. What is a granuloma?
a. Collection ofactivated (epithelioid, squamous cell-like) macrophages

75. The following is an example of granulomatous disease


a. Sarcoidosis
76. All are due to chronic inflammation except
a. Suppurative abscess
b. Rheumatoid arthritis
77. Cause of hydrops fetalis
a. Mother O-, fetus O+
b. Mother A+ , fetus Ac. Mother B+ , fetus Od. Mother AB, fetus B
78. What is the best good prognosis predictor in invasive breast carcinoma
a. No lymph node metastasis
b. Strong ER positivity
79. What does it mean when a tumor is grade 1 stage 1
a. Well differentiated and localized
80. What does grading refer to
a. How closely the tumor resembles normal tissue
81. Tumor of the breast (4 cm) that is fixed to the chest wall with lymph node
involvement and masses seen in the lung
a. T4N1M1
82. Collection ofmature elements normally found in a tissue
a. Hamartoma
83. The target antibody used for the diagnosis ofHIV
a. P24
b. Gp120
c. Gp41
d. Reverse transcriptase
84. Patient with weight loss and multiple thrombotic episodes, which tumor is he
most likely to have
a. Pancreatic carcinoma

85. The commonest cause ofsystemic emboli


a. Left ventricular thrombus
b. Left atrial thrombus
c. Right ventricular thrombus
86. After an MIwhich is the most common vascular insult
a. Systemic embolism following intramural thrombosis
87. Which is the commonest cause of thromboembolic vegetation in the heart on
an area over myocardial infarction
a. Hypercoagulability state
b. Endothelial injury
c. Change in normal blood flow
88. What is the cause of edema around a brain tumor
a. Lymphatic obstruction
b. Increased hydrostatic pressure (correct)
c. Decreased osmotic pressure
d. Increase pressure due to mass effect
89. Which of the following is NOT a paraneoplastic syndrome
a. Fever
b. Cancer Cachexia
c. ACTH, cushing disease
d. Clubbing offingers
e. Marantic endocarditis (non-bacterial thrombotic endocarditis)
90. Patient with small cell carcinoma of the lung, high blood pressure, puffiness of
the face (Cushing), what is the tumor most likely producing

a. ACTH
91. Which deficiency causes major ecchymosis and bleeding tendency
a. Vitamin K deficiency
b. Low vitamin E
92. Which vitamin deficiency causes glossitis, neuropathy, cheilosis, and dermatitis
a. Riboflavin (vitamin B2 ???)
b. Nacin (???)
93. Which lab test would signify major organ dysfunction after a large paracetomol
overdose
a. Ketonuria
b. Hypokalemia
c. Increased liver ALT
94. Which causes vasodilation, fever, pain
a. Prostaglandins
b. Bradykinin
c. TNF
d. Histamine
95. When do you see the acute inflammatory cells
a. Within 1st 5 days
b. Within 1st week
c. 2nd week
d. Within 24 hours
96. What is most characteristic about chronic inflammatory cells
97. Anticentromere antibodies are seen in
a. CREST/limited scleroderma
b. Hashimoto thyroiditis
c. Reiter disease
98. Incorrect matching of antibody with disease
a. SLE> > AntiSm
b. Sjogren> > AntiSS
c. Scleroderma> > SCL

d. Polyarteritis nodusa> > Anti ds DNA


99. Antismith antibody is seen in
a. SLE
b. Ascending cholangitis
c. Sjogren syndrome
100.

Obese patient underwent abdominal surgery then 1 day later died due

to respiratory problems, what is the cause ofdeath


a. Pulmonary embolism
101. Patient developed a firm mass or nodule after burns which showed excess
fibroblasts and collagen
a. Keloid
102.

Tumor cells which respond to radiotherapy, what is the cause ofcell

death
a. Through the release offree radicals (ROS)
b. Point mutation to tumor cells
c. Decreased ATP
d. Decreased blood supply
e. Thrombosis and occlusion
f.
103.

Fever
Characteristic ofSLE

a. Subendothelial immune complex deposits in glomeruli


104.

Patient with renal failure is found to have subendothelial deposits ofC3

in glomeruli by immunofluorescence
a. Immune complex deposition
105.

Not true regarding Sjogren syndrome

a. Hypogammaglobulinemia

106.

Commonest cause of premature labor

a. Preterm premature rupture ofmembrane


b. Low birth weight
107.

Newborn develops tetany then has history of opportunistic infection,

the pathogenesis of his disease is


a. Defect in 3-4 pharyngeal pouches
108.

Stop codon mutation

a. Beta thalassemia
109.

In neuroblastoma what is a good prognosis predictor

a. Age less than 1 year


110. Chemotaxis
a. Migration ofleukocytes to site of injury
111. Patient non-smoker with cytology report saying atypical cells resembling
squamous cell carcinoma cells, what was he most likely exposed to
a. Carbon/benzene
b. Radon
c. Asbestos
d. Inorganic substance
e. Exposure to birds
f.

Exposure to silica

112. Patient with colon cancer will mostly likely have


a. Kras mutation
113. Patient gives history ofeyelid droop and fatigability with repetitively used
muscles, what is the pathogenesis

a. Antibodies in neuromuscular junction


114. Healed old rheumatic heart valve disease gives rise to
a. Dystrophic calcification
b. Amyloidosis
c. Metastatic calcification
115. A woman had a palpable mass on the dome ofthe uterus and 10 years later it
doubled in size but the woman still has no symptoms, most likely
a. Leiomyoma
b. Leiomyosarcoma
116. Zinc protopherin is seen in
a. Chronic lead poisoning
b. Heroin abuse
117. What is secreted from macrophages that degrades ECM
a. Protease
b. IL-2
c. Catalase
118. Patient with maternal (??? Paternal) imprinting ofPrader-Willi gene (abnormal
chromosome 15), the most likely complication
a. Hypogonadism
b. Tall stature
c. Ataxia
d. Happy ???
119. During microscopic examination, what is the most likely feature which indicates
malignancy
a. Invasion

120.

Boy develops a rash on his face which looks like a slap. The rash, which

is composed of small red spots, subsequently involves the upper and lower
extremities. This boy also has arthralgia and then develops life-threatening
bone marrow anaplastic crisis, what is the microorganism is responsible?
a. Parvovirus
b. Rhinovirus
c. Parainfluenza virus
d. Measles
e. Rubella
121. Why are eosinophils effective against fungi
122.

Which is a complication ofacute inflammation (leukocytic induced

injury)
a. Vasculitis
b. Tissue destruction
c. Sarcoidosis
d. Interstitial fibrosis
e. TB
123.

Cell death in case ofhepatitis A is mediated by

a. DNA damage
b. Stimulation ofBcl2
c. Stimulation ofBclx
d. ATP
e. Free radicals
124.

Feature can be seen in necrosis

a. Inflammation in the surrounding tissue


125.

All the following are features of reversible cell injury except

126.

A patient with DM and hypertension, his renal biopsy would show

a. Hyaline arteriosclerosis
b. Glomerulosclerosis/arteriosclerosis
c. Fibrinoid necrosis of blood vessels
127.

A lady on oral contraceptive pills also smokes, which ofthe following

will she most likely develop

a. Cardiovascular disease/ischemic heart disease


b. Thromboembolism/thrombophlebitis
c. Endometrial carcinoma
d. Breast carcinoma
e. Ovarian carcinoma
128.

A person working in batteries industry will most likely develop

a. Renal toxicity
b. Lung cancer
c. Mesothelioma
d. Interstitial lung disease
129.

Which ofthe following statements is true

a. Prostaglandins and cytokines are cell-derived mediators


b. Most ofthe mediators are long acting

130.

The most common site in the brain for CMV infection in AIDS patients

is
a. Periventricular/intraventricular
b. Occipital
131. What is the commonest complication seen in ameobiasis
a. Ulceration, perforation of the colon
132.

A diabetic mother had a baby with bloody diarrhea. Which one of the

following is true
a. Abdominal radiograph shows submucosal gas bubble
133.

Marfan syndrome

a. Abnormal fibrillin disrupts mierfibril by a dominant negative effect


b. Some patients may die due to cardiac failure
134.

Which ofthe following is an autosomal dominant disease

a. Familial polyposis coli


b. Von willbrand disease
135.

Which ofthe following is an autosomal recessive disease

a. Spinal muscular atrophy


136.

Definition ofgenomic

a. Study ofmany genes with their interactions


b. Study ofsingle gene
c. Study ofall protein/gene products
137.

DNA microarray

a. An example of study ofgenetics


b. An example of study ofproteins
138.

Which ofthe following leads to granulomatous inflammation

a. Leprosy

139.

Which ofthe following may reduce the effect of toxic metabolites

a. Glutathione
140.

Which ofthe following vitamins is anti-toxic

a. Vitamin E
b. Vitamin K
c. Vitamin C
141. Which of the following is an inherited mutated gene in familial polyposis coli
a. APC
142.

Amyloidosis ofaging occur commonly in

a. Heart
b. Lung
c. Liver
143.

Feature ofautosomal recessive disease

a. Tends to be more uniform than autosomal dominant disease


144.

Features ofautosomal dominant disease

a. Some patients with mutant gene do not show the features ofdisease
b. Reduced penetrance
c. Patient should have affected parents
d. Onset ofclinical features appear early in life
145.

Changes seen in the lower esophagus as a result ofreflux

a. Goblet cell metaplasia


b. Squamous metaplasia

146.

The common finding in case of vasculitis

a. Neutrophilic infiltrate
147.

All ofthe following are features ofacute inflammation, except

a. Mononuclear inflammatory cell infiltrate


148.

Cytokines are mainly released from

a. Monocytes and lymphocytes


b. Neutrophils
149.

Fatty changes in liver in alcoholic patients

a. Impaired lipoprotein secretion from the liver


b. Increase transport of lipid from the peripheral tissue
150.

The most common feature seen after blood transfusion

a. Accumulation ofiron in the liver


151. Opsonization
a. C3b
b. C5a
152.

Inflammation that shows eosinophilic fibers or meshwork thread, is

called
a. Fibrinous inflammation
b. Serous inflammation
c. Suppurative inflammation
153.

Which ofthe following may prevent the process of wound healing

a. Absence ofinflammatory reaction


b. Abnormal immune system
c. Infection
154.

Cause ofdeath in children between 1 4 years in order ofmost

frequent
a. Accident, congenital malformation, chromosomal anomalies, malignant
neoplasm, homicidal, heart disease
155.

With regard to measles, all are true except

a. Recurrence is common
b. Multinucleated giant cells
c. Kuplik spots
156.

The most common cancer in a child with a previous history of

radiotherapy to a neck tumor


a. Thyroid carcinoma
157.

Triple repeat mutation

a. Amplification oftrinucleotide sequence


158.

Class IIMHC

a. Include HLA-DP type


159.

In hypersensitivity reaction type I, the production ofIgE and secretion

ofcytokines, is mainly mediated by


a. CD4 T helper cells
b. Basophils
c. Dendritic cells
160.

In AIDS, the infection reaches the CNSthrough

a. Infected macrophages
161. Window period in AIDS
a. The interval between time ofHIV infection and presence ofdetectable
antibodies in serum (seroconversion), it is about 3 weeks
162.

Large area of subcutaneous blood collection is called

a. Ecchymosis
163.

Causative organism of rheumatoid fever

a. Streptococcus pyogens, group A


164.

Newborn baby with a flat face and hypoplastic lung and deformed feet.

This combination is called


a. Sequence
b. Syndrome
c. Malformation
d. Disruption

165.

During inflammation killing ofviral agent is seen in

a. Phagolysosome
166.

In alpha 1 antitrypsin deficiency, genetic mutation ofprotein leads to

a. Improper folding or partial folding of proteins which aggregate in the ER


ofliver and not secreted (missfolding)
167.

The primary defect in cystic fibrosis

a. Abnormal folding of protein, resulting in subsequent loss or degradation of


protein
b. Defect in protein synthesis
c. Defective conductance
d. Defective regulation
e. Reduced abundance
168.

All ofthe following are features ofchronic inflammation, except

a. Purulent exudate
169.

The following is a feature of chronic inflammation

a. Destruction to stroma or tissue


170.

Autoantibodies ofpolyarteritis nodosa

171. Phenylketonuria is
a. Autosomal recessive disease
b. Autosomal dominant disease
c. X-linked disease
172.

Which ofthe following is a lysosomal storage disease

a. Niemann-pick disease
173.

What does the following mean 46, XY, r (14)?

a. Ring chromosome 14 with deletion of a portion of both long and short arms
b. Ring chromosome 14 with translocation
174.

During wound healing, the role of neutrophils start at

a. 6 24 hours
b. Before 6 hours

c. 24 48 hours
175.

Amoebiasis in liver is characterized by

a. Multiple
b. Size 2 15 cm
c. Thick membrane with fibrous wall
176.

Systemic amyloidosis is associated with

a. Chronic infection
177.

Sjogren disease

a. Perivascular and periductal lymphatic infiltrate


b. Associated with non-Hodgkin B cell lymphoma
178.

Patient with a bee bite, developed a rash. What is the mechanism

a. IgE production
179.

Helminthis reaction is due to release of

a. Histamine
180.

What does 46, X, r(X) give rise to

a. Turner syndrome
181. Patient received anti-Tb therapy (isoniazid), the most common complication
a. Vitamin B6 deficiency (pyridoxine)
b. Vitamin A
c. Riboflavin
182.

In pulmonary embolism, the embolus usually comes from

a. Lower limb
b. Left ventricle
183.

Pathogenesis seen in SLE

a. Type IIIimmune complex deposition


184.

A biopsy from a patient shows malignant neoplasm. The tumor is

composed ofspindle cells with high N:C ratio and marked pleomorphism. The

cells are positive for vimentin, negative for keratin and CD45. This type of
neoplasm is most likely to have been diagnosed in which ofthe following
patients
a. 15 year old man with a mass in left femur and lung metastasis
b. 35 year old woman with left breast mass and enlarged axillary LN
c. 55 year old woman with massive ascites and multiple peritoneal metastases
d. 25 year old man with enlarged testis
e. 5 year old boy with a right renal mass
185.

What is the usual complication of premature infants

a. Low birth weight


b. Alveolar hypoplasia
c. PDA
d. Hydrops fetalis
186.

Defective inflammatory process results in

a. Allergic reaction
b. Autoimmune disease

c. Impaired wound healing


187.

All ofthe following chromosomal abnormalities result in loss of genomic

material except
a. Robertsonian translocation
b. Ring chromosomes
c. Isochromosomes
d. Inversion
188.

26 year old female with (normal or fair) intelligence, never

menstruated, with short stature. Karyotopic analysis of buccal cells shows


some of the cells having single barr body, what is the diagnosis
a. Male patient
b. Mosaicism turner
c. 45XO
189.

Following abdominal surgery, wound strength over the next week is

mainly due to deposition of


a. Glycoprotein
b. Collagen I
c. Collagen III
d. Fibronectin
190.

A term newborn with 8 and 9 APGAR score at 1 and 5 min respectively,

3 days later he develops bleeding through the umbilical stump wound, this
baby most likely has
a. Vitamin K deficiency
191. Rolling of leukocytes on the endothelial surface by binding to selectins is
a. Firm adhesion to the endothelial cells
b. Result in endothelial and leukocyte activation

c. The initial step in leukocyte transport to the site ofthe inflammation


d. Involve transduction of the leukocytes between the endothelial cells
192.

Plegra (Niacin deficiency)

a. Dermatitis
b. Diarrhea
c. Dementia
193.

Regarding giant cell arteritis which is true

a. 3 histologic types, negative in 1/3 ofpatients


b. Granulomatous vasculitis with fragmented internal elastic lamina and giant
cell reaction in 2/3 ofcases
c. Non-specific leukocyte infiltration of the vessel wall
d. Intimal fibrosis with wall thickening and luminal narrowing
194.

Takayasu arteritis

a. Perivascular (adventitial, vasovasorum) mononuclear cell infiltrate


b. Medial layer fibrosis, occasionally granuloma and acellular intimal thickening
195.

PAN and Kawasaki have similar morphology

a. Related HBsAg antigenemia


b. Immune complex play a role in pathogenesis (type III)
c. Morphology: all histological stages can be seen concurrently
d. Acute lesion: sharply circumscribed arterial fibrinoid necrosis with
neutrophilic infiltrate might reach the adventitia
e. Healing lesion: proliferating fibroblast superimposed on the fibrinoid
necrosis
f.

Healed lesion: fibrous vascular thickening, fragmentation at internal elastic


lamina and aneurysmal dilatation at the vessel wall

196.

Wegner granulomatosis

a. Morphology: resembles acute PAN accompanied by granuloma


197.

Thrombangitis obliterans (Beurger)

a. Segmental acute and chronic inflammation ofthe vessels


b. Neutrophilic infiltration of the wall with mural thrombosis and
microabscesses with giant cell formation
198.

Autopsy ofthe lung ofa patient showed multiple 2 3 mm nodules

scattered throughout the lung parenchyma bilaterally and formed of


epithelioid macrophages, these findings is mainly due to
a. Tb
b. Sarcoidosis
199.

Brain changes seen in patients with Alzheimer disease

a. Hyperplasia
b. Atrophy
c. Hypertrophy
200.

Exposure to carbon tetrachloride is toxic mainly to

a. Liver
b. Lung
c. Heart
d. Kidney
e. Skin
201.

Child with bilateral retinoblastoma, blood sample taken, leukocyte

analysis of his genetic make-up will show


a. Alteration in single allele ofRB locus

b. Alteration in single allele ofMyc gene locus


c. Alteration ofboth allele ofMyc gene locus
d. Alteration ofboth allele ofRB gene
e. No alteration in RB or Myc gene
202.

In acute inflammation, neutrophils are recruited at the site of

inflammation by the influence of


a. C5a
b. Lymphokines
c. Hageman factor
203.

Frame shift mutation occurs when there is

a. One or two base pair insertion or deletion in the coding sequence


b. One or two base pair insertion or deletion in the non-coding sequence
c. Insertion or deletion of3 base pairs in the coding sequence
204.

An example illustrating the molecular mimicry as a mechanism for

infection involvement in the developing of autoimmune diseases is


a. Anti-Rheumatic fever
205.

An autopsy revealed that the main pulmonary artery showed a

hemorrhagic band of connective tissue across the lumen, which of the


following explains this finding
a. Infection
b. Hemorrhage
c. Thromboembolism
206.

A person having an AD genetic alteration and isn't showing any

features ofthe alteration, this is called


a. Reduced penetrance
b. Variable expressivity

207.

35 year old patient complaining of dry eyes, dry mouth, and facial

swelling, is found to have parotid gland swelling, the most important thing to
rule out is
a. NHBL
b. Hodgkin lymphoma
c. Parotid abscess
208.

The following can cause hypovolemic shock

a. Severe burns
209.

Cause ofhyperacute rejection

a. Presence ofdonor antibody in the host circulation (recipient previously


sensitized to antigens in a graft)
210.

Which bacteria commonly causes abscess

a. Staph aureus
211. After radiation injury which can protect the cells
a. Phospholipase
b. Glutathione
c. Endonuclease
212.

Patient with history oflupus nephritis

a. Type IIIhypersensitivity
213.

Patient with history ofsmall cell carcinoma develops dermatomyositis,

what is the reason for this


214.

Which is the commonest childhood disease

a. Congenital disease
215.

Breast DNA microarray

a. Genetic
b. Genomic
c. Clonal

216.

Heart defect in marfan syndrome

a. AR
217.

Acute phase reaction include all the following, except

a. Fever
b. Decreased apetite
218.

Prostaglandin and leukotriene

a. Cellular mediators
219.

Amebic dysentery complication

220.

A patient is a smoker and on OCP, she develops right upper quadrant

pain but no gallstones, only 3 well-defined masses on the liver, what will she
most likely develop next
a. Breast cancer
b. Spleen tumor
c. Brain tumor
d. Adrenal tumor
221.

Anti-microsomal antibody

a. Myasthenia gravis
b. Type I diabetes
c. Hashimoto's thyroiditis
222.

MHC class II

a. HLA A, B, C
b. HLA-DP
c. HLA-DQ
223.

Metaplasia closely related to carcinoma developement

a. Intestinal metaplasia of the lower esophagus


b. Intestinal metaplasia of gastric mucosa
c. Metaplasia ofcervical
224.

In MI, early granulation tissue starts in

a. 1 week (7 10 days)
b. 1 month

c. 24 hours
d. 1 hour
225.

Which ofthe following is a component ofMEN1

a. Parathyroid hyperplasia, pancreatic carcinoma, pituitary adenoma


226.

All ofthe following are vascular complications ofdiabetes, except

a. Atherosclerosis
b. Retinopathy
c. Nephropathy
d. Hyaline arteriosclerosis
e. Malignant arteriosclerosis
227.

Bacillary angiomatosis in which organism

a. Bartonella
228.

Question about MEN IIB

229.

External receptor that initiates apoptosis

a. Fas
b. Bcl-2
c. C-myc
230.

In acute inflammation, which ofthe following is the cause ofinterstitial

edema
a. Lysozyme released from neutrophils
b. Endothelial retraction
c. Adhesion molecules
d. Lymphatic obstruction
e. Arteriolar constriction
231.

Free radicals that causes cytosolic oxidation

a. NADPH
b. NADPHsynthase
232.

Beta-Naphthalamine exposure will predispose to which cancer

a. Transitional cell carcinoma ofthe bladder

b. Transitional cell carcinoma ofthe kidney


c. Squamous cell carcinoma ofthe bladder
233.

Which ofthe following combination will cause episomal replication

a. HPV 6 & 11
b. HPV 16 & 18
c. HPV 30 & 33
234.

Which ofthe following will appear in the second month ofwound

healing
a. Collagen I
b. Collagen III
c. Fibrillin
d. Fibronectin
e. Myofibroblast
235.

Invasive ductal carcinoma differs from in-situ in that it has

a. Increase collagenase IV activity


236.

Polymorphs kill strept by

a. Hydrogen peroxide
237.

Inversion definition

a. 2 breaks with 180 rotation


238.

Mechanism offragile X syndrome

239.

Oncogenic role of HPV

a. E7
240.

All are true oftype I diabetes except

a. Amyloid deposition
241.

Apoptosis

a. Promoted by BCL-2
b. ATP dependent process
c. Inhibited by Bax
242.

Rhodococcus equi infection in immunocompromised patient causes

a. Malakoplakia
b. Pseudoinflammatory tumors
c. Pseudotuberculosis
243.

Which ofthe following antibodies detects BAF47

a. INI-1
b. PTEN
244.

Stop codon

245.

Which ofthe following statements regarding amyloidosis is false?

a. 1ry and 2ry forms exist


b. Metachromatic stains are useful in making the diagnosis
c. Intracytoplasmic hyaline globules are characteristic
d. EM typically demonstrates 7.5-10 nm non-branching filaments
e. Nodular and diffuse forms ofliver involvement have been described
246.

Fungal organism which to date has not been cultured on synthetic

media
a. Loboa loboi
247.

Cutaneous lesion resulting from igestion ofbromides or iodides

a. Halogenoderma
248.

Fungus whose cigar-shaped yeast form is thought to be diagnostic in

tissue sections

a. Sporothrix schenkii

249.

2.
1.

Skin

What causes squamous cell carcinoma by forming DNA pyrimidine dimers


a. UVB
b. UVC
c. UVA

2. What variant of basal cell carcinoma is associated with recurrence


a. Micronodular
b. Infiltrative
3. Where does acral lentiginous malignant melanoma commonly occur
a. Palms ofthe hands and soles ofthe feet
(also subungal areas, mucocutaneous junctions oforal and nasal cavities,
and anus)
4. Morphology oflichen sclerosis et atrophicus
a. Epidermal atrophy, hyperkeratosis, and hydropic degeneration of the basal
layer, with complete obliteration ofthe structure ofthe upper dermis
5. What lesion has the following morphology: lichen planus dermatitis with periadnexal lymphocytes
6. What is the lesion with the following morphology: nevus with large nuclei and
prominent nucleoli
a. Spitz nevus
b. Dysplastic nevus
7. Which of the following is a risk factor for development ofsquamous cell
carcinoma
a. Xeroderma pigmentosa
b. Ionizing radiation
c. HPV type 3
d. EBV
8. Which of the following is a predisposing factor for squamous cell carcinoma

a. Vulvar lichen sclerosis


b. Erythema multiform
c. Morphea
9. Immune complex deposition in SLE
a. Dermoepidermal junction
b. Dermal papillae
c. Dermis
d. Perivascular
e. Perineural
10. All of the following is infiltrated by cytotoxic (CD8+ ) T-cells EXCEPT
a. Drug fixed eruption
b. Urticaria
c. GVHD
d. Erythema multiforme
e. Mycosis fungoides
11. Spongiosis is
a. Intercellular edema in the epidermis
b. Intercellular edema in the keratinocytes
c. Edema ofthe dermis
12. Skin disease which has dermal papillae microabscesses and by IF is IgA positive
is associated with
a. Celiac disease [dermatitis herpetiformis]
13. Which of the following is the best way to avoid metastasis of malignant
melanoma
a. Early diagnosis and complete excision
b. Sentinel lymph node
c. Chemotherapy
d. Reduce exposure to UV light
e. Excision oflymph nodes with metastases
14. Female patient had bone marrow transplant, her CBC afterwards was normal
but she developed skin rash and diarrhea, what is the cause
a. GVHD

15. Cell of origin ofMF


a. CD 4
b. CD 8
c. Dendritic cells
16. The most important prognostic factor in melanoma is
a. Clark
b. Breslow
17. A mucocutaneous lesion (small cell carcinoma) which is CK20 positive
a. Merkel cell carcinoma
18. A patient with HIV and skin lesion (Kaposi)
19. A patient with multiple cutaneous nodules which appear and disappear over a
period of time and it's a type of cutaneous lymphoma
a. Lymphomatoid papulosis
b. Sezary syndrome
c. MALT axon B-cell lymphoma
20. Immuno for MF
a. CD 3 + ve, CD 4 + ve, CD 8 ve
b. CD 3 + ve, CD 4+ ve, CD 8 + ve
c. CD 3 ve, CD 4 + ve, CD 8 -ve
21. Which is a mimicker of BCC?
a. Trichoepithelioma
22. Female with multiple dermal nodules showing palisading granulomas
a. Granuloma annulare
23. Skin tumor that is CAM5.2 positive
a. Hidradenoma

24. Which tumor is typically composed of highly pleomorphic spindle cells and
large sometimes multinucleated lipid-laden histiocytic cells that exhibit a
vimentin + , HMB45 -, and MART-1/Melan A immunophenotype?
a. Atypical fibroxanthoma
b. Merkel cell carcinoma
c. Metastatic breast carcinoma
d. Nodular melanoma
25. Which tumor shows perinuclear dot-like positivity for CK20 and is CD56
positive?
a. Atypical fibroxanthoma
b. Merkel cell carcinoma
c. Metastatic breast carcinoma
d. Nodular melanoma
26. Which tumor occasionally occurs in association with basal cell carcinoma and
squamous cell carcinoma ofthe skin?
a. Atypical fibroxanthoma
b. Merkel cell carcinoma

c. Metastatic breast carcinoma


d. Nodular melanoma
27. Which tumor is associated with both prior irradiation and long-standing
lymphedema?
a. Angiosarcoma
b. Epithelioid sarcoma
c. Kaaposi sarcoma
d. Pseudovascular squamous cell carcinoma
e. Spindle cell hemangioma
28. Loss of INI-1 expression is most characteristic ofwhich lesion?
a. Angiosarcoma
b. Epithelioid sarcoma
c. Kaposi sarcoma
d. Pseudovascular squamous cell carcinoma
e. Spindle cell hemangioma
29. Which tumor contains an admixture of thick-walled vessels, cavernous
hemangioma-like zones, and spindled zones containing vacuolated endothelial
cells?
a. Angiosarcoma
b. Epithelioid sarcoma
c. Kaposi sarcoma
d. Pseudovascular squamous cell carcinoma
e. Spindle cell hemangioma
30. Which is a vesiculobullous disease characterized by intercellular deposition of
IgG and C3?
a. Pemphigus vulgaris
31. Which disease usually results in death in utero in males?
a. Incontinentia pigmenti
32. Which entity is associated with CHILD syndrome?
a. Inflammatory linear verrous epidermal nevus
33. Associated with chronic lymphedema in post mastectomy patients
a. Stewart-Treves syndrome

34. Associated with HHV


a. Kaposi sarcoma
35. A low grade malignant cutaneous neoplasm typically presenting in children as
diffuse swelling or intradermal tumor
a. Endovascular papillary angioendothelioma (Dabska tumor)
36. Which lesion characteristically demonstrates Touton-type giant cells?
a. Xanthogranuloma
37. Which tumor shows S100 positivity and positive staining for lysosomal granules
and is ultrastuctually characterized by intracellular septation into membrane
limited compartments?
a. Granular cell tumor
38. Which lesion typically presentsas a very dark small (3 6 mm) uniform macule
on the extremity ofa young patient?
a. Pigmented spindle cell nevus
39. Kamino bodies are a charasteric feature ofwhich lesion?
a. Spitz nevus
40. Which lesion is wedge shaped with fascilces ofspindled melanocytes and
admixed melanophages that infiltrate pilar muscle?
a. Deep penetrating nevus
41. Which tumor may show positive staining for neuroendocrine markers and be
negative for CK20?
a. Metastatic small cell neuroendocrine carcinoma

42. Which lesion typically presents as violaceous flat topped papules with sawtooth epidermal hyperplasia, hypergranulosis, and prominent Civatte bodies?
a. Lichen planus
43. Which lesion is associated with localized scleroderma (morphea)?
a. Lichen sclerosus et atrophicus
44. Which lesion typically shows granular deposits ofimmunoglobulin and
complement along the dermal epidermal junction in sun-exposed clinically
affected skin?
a. Discoid lupus erythematosus
45. Which one ofthe following DNA abnomalities would support a diagnosis of
melanoma over melanocytic nevus?
a. Gains of11p
b. GNAQ gene mutation
c. Multiple DNA gains and losses
d. t(12;22)(q13;q12)
e. V600E BRAF gene mutation
46. Which one ofthe following parameters is used for pathologic tumor staging in
melanoma?
a. level of invasion
b. Mitotic count per mm2
c. Presence ofregression
d. Tumor infiltrating lymphocyts
e. Tumor largest diameter
47. what is the main reason for evaluating BRAF gene status in melanoma?

a. Presence ofV600E BRAF mutation is associated with resistance to


Vemurafenib treatment

b. Presence ofV600E BRAF mutation is associated with tumor response to


Ipilimumab treatment
c. Presence of V600E BRAF mutation is associated with tumor response
to Vemurafenib treatment
d. Presence ofV600E BRAF mutation is associated with tumor response to
Ipilimumab treatment
e. Serves as a diagnostic test for melanoma

3.
1.

Oral Cavity and Oropharynx

Genetics oforal squamous cell carcinoma


a. 19p
b. 17p with p53 mutation
c. P16

2. What is the most common risk factor for oral cavity tumors
a. Smoking
3. Most common site oflaryngeal carcinoma
a. Glottis
b. Epiglottis
c. Supraglottis
4. Which laryngeal tumor is characterized by comedo-type necrosis?
a. Basaloid squamous cell carcinoma
5. Which tumor is characterized by the presence ofa myoepithelial component?
a. Adenoid cystic carcinoma
6. HPV positivity is associated with a more aggressive variation ofbasaloid
squamous cell carcinoma
a. True
b. False

4.
1.

Mandible and Maxilla

Intraosseous ameloblastoma is
a. Locally aggressive tumor
b. Benign
c. Malignant
d. Peripheral is more aggressive

2. Which is false regarding adamantinoma


a. Young age at presentation is bad prognosis
b. CK 8, 18 +
c. Osteofibrous dysplasia like pattern is least common histologic event
d. Common variants include squamoid, tubular, spindle, and basaloid
3. Which is the most common lesion occurring in the jaw showing osteoclast-like
multinucleated giant cells
a. Central giant cell tumor
b. ABC
c. Central giant cell granuloma
4. Which cyst has a high rate ofrecurrence
a. Odontogenic cyst
5. Giant cells are seen in all ofthe following except
a. Adamantinoma
b. ABC
c. Brown tumor
d. Reparative giant cell granuloma

5.
1.

Respiratory Tract
Which type ofpulmonary adenocarcinoma is newly described and shows
positivity for CDX2 and CK20

2.

a.

BAC, mucinous type

b.

BAC, non-mucinous type

c.

Adenocarcinoma goblet cell

d.

Adenocarcinoma signet cell

e.

Mucinous (colloid) carcinoma

Which ofthe following is associated with nasopharyngeal carcinoma


a.

EBV

b.

HPV

c.

CMV

3.

Atypical lung carcinoid is characterized by which of the following

4.

Which tumor occurs exclusively in women ofreproductive age


a. Lymphangioleiomyomatosis
b. Wegner's granulomatosis
c. Good pasture's disease
d. Churg-strauss disease

5.

Interstitial lung disease with temporally heterogenous and low power


"variegated" appearance

6.

Lung shows alveolar hyaline membrane formation without inflammatory


cell infiltrate, what is the diagnosis
a.
Diffuse alveolar damage

7.

8.

9.

10.

BOOP histology
a.

Asteroid body in the bronchus

b.

Loose fibrous tissue in the bronchus and alveoli

Transbronchial biopsy is reliable for all ofthe following EXCEPT


a.

Sarcoidosis

b.

Interstitial lung disease

c.

Lung cancer

d.

Infection

Which interstitial pneumonia is strongly associated with smoking


a.

Usual interstitial pneumonia

b.

Desquamative interstitial pneumonia

c.

Non-specific interstitial pneumonia

HIV positive patient has cough and dyspnea and his lung biopsy showed
intra-alveolar frothy eosinophilic material, which special stain should be
done

11.

a.

PAS

b.

Modified methamin silver

c.

Mucicarmin

Same heading as question 10, what is the organism


a.

12.

P. carinii

The vast majority ofthe bronchoalveolar mucinous carcinoma is classified


now according to the

13.

a.

Invasive mucinous carcinoma

b.

Minimally invasive carcinoma

Which lung adenocarcinoma has lipoid morphology and no capsular,


stromal, or vascular invasion with tall columnar cells and apical mucin
a.

14.

BAC, mucinous type

Which tumor shows loss of 3p


a.

Small cell carcinoma

15.

Multiple pleural nodules and pleural thickening composed ofspindle cells


which are CK positive and Calretinin positive, CD34 and bcl-2 ve

16.

a.

Malignant mesothelioma, sarcomatoid type

b.

Synovial sarcoma

c.

Solitary fibrous tumor

d.

Intrathoracic desmoid

e.

Sarcomatoid carcinoma

Female patient with history of uterine fibroids has developed multiple


pulmonary nodules. Which ofthe following is correct
a.

17.

Both tumors have the same clonality

35 year old female patient with sinusitis and dyspnea, biopsy showed
capillaritis, geographic necrosis, poorly formed granulomas and palisaded
lymphocytes

18.

a.

TB

b.

Sarcoidosis

c.

Aspergillosis

d.

Phinosporidosis

e.

Wegner granulomatosis

Which ofthe following organisms might cause lung lesion with eosinophilic
pneumonia and granuloma palisaded around small central eosinophilic
material in the interstitium with bronchoalveolar destruction

19.

a.

GMSpositive aspergillosis

b.

Giemsa positive leishmaniasis

c.

AFB positive TB

d.

GMSpositive fungi

Patient has cough, his lung shows patchy infiltrate and interstitial
mononuclear cell infiltrate, what is the causative organism

20.

a.

E. coli

b.

Aspergillus

c.

Mycobacterium Tb

d.

Mycoplasma pneumonia

e.

Adenovirus

Lung disease associated with EBV infection

a. Lymphoid (lymphocytic) interstitial pneumonia


b. Follicular bronchitis
c. Lymphangiomyomatosis
d. Lymphomatoid granulomatosis
e. Castleman disease
f.
21.

Lymphoepithelioma-like carcinoma

Which stain is used to differentiate between 1ry and 2ry lung carcinoma
a. TTF1

22.

23.

What is the causative organism ofsinonasal papilloma


a.

HPV

b.

HIV

c.

EBV

d.

RSV

Which ofthe following is true regarding cor pulmonale


a. Hypertrophy of the right side ofthe heart

24.

All ofthe following are true regarding right sided heart failure except
a. Pulmonary congestion
b. Hepatosplenomegaly
c. Peripheral edema
d. Pleural effusion

25.

A patient with pneumonia, glomerulonephritis, and c-ANCA positivity has


which disease
a. Wegner's granulomatosis

26.

Rhinosporidiosis is caused by
a. Spongiorum containing spores

27.

Acute respiratory distress syndrome is associated with which drug


a. Aspirin
b. Cocaine

28.

Which ofthe following tumors is peripheral


a. Adenocarcinoma

29.

Histology of solitary fibrous tumor

30.

Pulmonary aspergillosus is associated with


a. Corpora amylacae
b. Target lesions

31.

What is the mutation seen in small cell carcinoma


a. Deletion of 3p

32.

Histopathology of ARDS

33.

48 year old male patient with history of dyspnea for 12 years. No cough,
no fever, no sputum production. X-ray showed diffuse interstitial fibrosis.
What is the most likely risk factor
a. Inorganic dust particles
b. Organic particles
c. Nitric oxide
d. Smoking
e. Radon gas

34.

Which tumor is HMB-45 positive


a. Lymphangioleiomyomatosis

35.

Patient with idiopathic pulmonary disease, shows temporal heterogenous


"variegated" appearance on low power
a. Usual interstitial pneumonia
b. Desquamative interstitial pneumonia

36.

Primary lung peripheral adenocarcinoma


a. TTF1 + , CK7 + , CDX2 -, p63

37.

Tumor showing multiple pleural nodules or diffuse pleural thickening with


spindle cell proliferation which are positive for CK, variable positivity for
calretenin, and negative for CD34
a. Mesothelioma

38.

Lung mass CDX2 + ve, CK 7 ve, TTF-1 ve


a. Metastatic colon cancer

39.

Extrinsic allergic alveolitis


a. Chronic interstitial fibrosis and non-caseating granulomas
b. Eosinophils in the wall

40.

What is the most common organism in patients with lung abscess


a. Staph aureus

41.

Allergic alveolitis

42.

A 69 year old retired carpenter with recurrent nose bleeds has an


exophytic granular mass arising ove the middle turbinate bone. Which test
will confirm the most likely diagnosis
a. AE1/AE3
b. CD45
c. NSE
d. PAS/D
e. Reticulin

43.

Which is a diffuse interstitial process characterized by a polyclonal


population of B cells and T cells and is most often seen in the setting of

collagen vascular disease or immunodeficiency syndromes


a. Extranodal marginal zone B cell lymphoma
b. Lymphocytic interstitial pneumonia
c. Lymphomatoid granulomatosis
d. Mantle cell lymphoma
e. Nodular lymphoid hyperplasia
44.

Which disorder forms a mass lesion with architectural effacement and is


morphologically well circumscribed, lacks lymphoepithelial lesions or
pleural invasion, has prominent germinal center formation and
interfollicular plasma cells
a. Extranodal marginal zone B cell lymphoma
b. Lymphocytic interstitial pneumonia
c. Lymphomatoid granulomatosis
d. Mantle cell lymphoma
e. Nodular lymphoid hyperplasia

45.

Which disorder is Epstein-barr virus driven and characterized by a mixture


of atypical B cells and non-neoplastic T cells arranged in angiocentric
pattern?
a. Extranodal marginal zone B cell lymphoma
b. Lymphocytic interstitial pneumonia
c. Lymphomatoid granulomatosis
d. Mantle cell lymphoma
e. Nodular lymphoid hyperplasia

46.

Which ofthe following neonatal lung diseases is characterized by nearcomplete lack ofairspace development and is universally fatal in the 1st

hour oflife?
a. Acinar dysplasia
b. Alveolar capillary dysplasia
c. Bronchopulmonary dysplasia
d. Congenital alveolar dysplasia
e. Pulmonary interstitial glycogenosis
47.

Chronic congestive vasculopathy (pulmonary venous hypertension) may


result from all of the following cardiopulmonary lesions, except?
a. Congenital cardiomyopathy with chronic left ventricular failure
b. Mitral valve stenosis
c. Pulmonary vein stenosis
d. Total anomalous pulmonary venous connection
e. Ventricular septal defect

48.

Alveolar capillary dysplasia is caused by mutations or deletions in which of


the following genes?
a. ABCA3
b. FOXF1
c. SFTPB
d. SHH
e. TTF1

49.

How should most tumors with primarily non-mucinous lepidic growth and
a focus ofinvasive carcinoma measuring 5 mm or less be classified in the

IASLC/ATS/ERSclassification scheme
a. Adenocarcinoma, predominantly lepidic type
b. Adenocarcinoma, mixed type
c. Bronchioloalveolar carcinoma
d. Minimally invasive adenocarcinoma
50.

In a case otherwise meeting criteria for minimally invasive


adenocarcinoma (MIA) the diagnosis of MIA is excluded ifwhich of the
following is/are present
a. The invasive component consists of a papillary or micropapillary
pattern
b. Multiple foci ofinvasion less than 5 mm are present but the total
amount of invasive tumor adds up to more than 5 mm
c. Pleural and/or lymphovascular invasion is present
d. Multiple primary tumors are present

51.

The vast majority oftumors previously classified as mucinous


bronchioloalveolar carcinoma will be classified as which ofthe following in
the IASLC/ATS/ERSclassification
a. Adenocarcinoma in situ
b. Minimally invasive carcinoma
c. Invasive mucinous carcinoma
d. Colloid carcinoma

52.

While the capsule ofC neoformans typically stains fuchsia with


mucicarmine stain, capsule-deficient variants may be stained by
a. Alcian blue
b. Fontana-masson
c. Verhoeff-van gieson
d. Masson trichrome

53.

Malakoplakia is
a. A granulomatous-type inflammatory response usually seen in the
urinary tract associated with gram-negative coliform bacilli
b. Is typically associated with the formation ofMichaelis-Gutmann bodies
c. Seen in the lungs associated with R. equi infection
d. All ofthe above
e. None of the above

54.

C neoformans
a. Is a common soil contaminant spread through aerosolized dust
b. Is spread through pigeon droppings
c. Is usually a pathogen in immunocompromised patients but may also
afflict immunocompetent patients
d. Has a world-wide distribution
e. All ofthe above
f.

55.

Non e ofthe above

Which human papillomavirus type is the most common cause ofrecurrent


repiratory papillomatosis
a. HPV 16
b. HPV 18
c. HPV 31
d. HPV 33
e. HPV 11

56.

Which is not typically a feature ofrecurrent respiratory papillomatosis


a. A pushing, non-infiltrative
b. Fibrovascular cores covered in an orderly squamous cell carcinoma

c. Increased mitotic figures and a desmoplastic stroma


d. Presence of mild to moderate squamous dysplasia
e. Multiple exophytic lesions
57.

What is the most common clinical presentation for recurrent respiratory


papillomatosis
a. Cavitary lung lesions seen on imaging
b. Hoarseness or changes in voice
c. Respiratory distress
d. Presence of enlarged cervical lymph nodes
e. Post-obstructive pneumonia

58.

Epithelioid malignant mesothelioma may rarely exhibit clear cell


morphology and may be positive for CD10 and RCCMa in a small number
of cases
a. True
b. False

59.

The presence ofextracellular mucin, which stains positively for alcian blue,
reliably supports a diagnosis of adenocarcinoma over mesothelioma
a. True
b. False

60.

Epithelioid mesotheliomas are typically characterized by bland, uniform


cytologic features but may exhibit anaplastic, signet ring,
lymphohistiocytoid or rhabdoid morphology
a. True
b. False

61.

Which entity is characterized by a nodular lymphoid infiltrate centered on


bronchioles and sharply demarcated from normal lung, showing a

predominance of interfollicular T cells, polytypic B cells and Bcl-2 negative


germinal centers?
a. Extranodal marginal zone lymphoma, MALT type
b. Lymphoid interstitial pneumonia
c. Mantle cell lymphoma
d. Nodular lymphoid hyperplasia
e. SLL
62.

Which lymphoid proliferation typically involves the interstitium ofthe lung


preferentially and is composed of T lymphocytes with relatively few B
cells and polytypic plasma cells?
a. Extranodal marginal zone lymphoma, MALT type
b. Lymphoid interstitial pneumonia
c. Mantle cell lymphoma
d. Nodular lymphoid hyperplasia
e. SLL

63.

Which lesion exhibits spread along bronchovascular bundles with


lymphoepithelial lesions infiltrates interlobular septa and pleura and is
characterized cytologically by an admixture ofsmall lymphocytes with few
admixed large cells monocytoid and plasma cells with monotypic light
chain expression and plasmacytoid cells with Dutcher bodies?
a. Extranodal marginal zone lymphoma, MALT type
b. Lymphoid interstitial pneumonia
c. Mantle cell lymphoma
d. Nodular lymphoid hyperplasia
e. SLL

64.

Which entity is characterized by coarse granular intra-alveolar exudates


which are typically PASpositive and may contain observable cholesterol
clefts?
a. Adenovirus pneumonia
b. CMV pneumonia
c. CMV pneumonia and Pneumocytstis pneumonia
d. Pneumocystis pneumonia
e. Pulmonary alveolar proteinosis
f.

65.

Pulmonary edema

Which entity has a classic histologic pattern of frothy vacuolated intra-

alveolar exudates but may also be associated with a diffuse alveolar


damage interstitial exudates or minimal histologic changes?
a. Adenovirus pneumonia
b. CMV pneumonia
c. CMV pneumonia and PCP
d. PCP
e. Pulmonary alveolar proteinosis
f.
66.

Pulmonary edema

Which entity is characterized by enlarged cells with intranuclear


eosinophilic inclusions and smaller intracytoplasmic inclusions?
a. Adenovirus pneumonia
b. CMV pneumonia
c. CMV and PCP
d. PCP
e. Pulmonary alveolar proteinosis
f.

67.

Pulmonary edema

Which tumor consistently shows nuclear expression ofp63 and is negative


for TTF-1?
a. Atypical carcinoid
b. Large cell neuroendocrine carcinoma
c. Small cell carcinoma
d. Squamous cell carcinoma, poorly differentiated
e. Typical carcinoid

68.

Which neuroendocrine tumor is defined in the WHO classification as having


2 10 mitoses/10 hpfor necrosis?
a. Atypical carcinoid
b. Large cell neuroendocrine carcinoma
c. Small cell carcinoma
d. Squamous cell carcinoma, poorly differentiated
e. Typical carcinoid

69.

Which tumor is characterized by large cells with prominent nucleoli,


extensive infarct-like necrosis, chromogranin expression and mitotic

counts exceeding 10/ 10 hpf?


a. Atypical carcinoid
b. Large cell neuroendocrine carcinoma
c. Small cell carcinoma
d. Squamous cell carcinoma, poorly differentiated
e. Typical carcinoid
70.

Which malignant lesion is by definition non-invasive?


a. Atypical adenomatous hyperplasia
b. Metastatic adenocarcinoma
c. Pulmonary adenocarcinoma, bronchoalveolar type
d. Pulmonary adenocarcinoma, fetal type
e. Pulmonary adenocarcinoma, mixed subtype
f.

71.

Pulmonary adenocarcinoma, mucinous type

Which lesion typically shows glycogen rich glands with supranuclear and
subnuclear vacuoles imparting an endometrioid appearance?
a. Atypical adenomatous hyperplasia
b. Metastatic adenocarcinoma
c. Pulmonary adenocarcinoma, bronchoalveolar type
d. Pulmonary adenocarcinoma, fetal type
e. Pulmonary adenocarcinoma, mixed type
f.

72.

Pulmonary adenocarcinoma, mucinous type

Which histologic pattern ofprimary adenocarcinoma ofthe lung is the


most common?
a. Atypical adenomatous hyperplasia
b. Metastatic adenocarcinoma

c. Pulmonary adenocarcinoma, bronchoalveolar type


d. Pulmonary adenocarcinoma, fetal type
e. Pulmonary adenocarcinoma, mixed subtype
f.
73.

Pulmonary adenocarcinoma, mucinous type

Which entity is typically encountered in immunocompromised individuals


and is associated with Rhodococcus equii infection?
a. Clear cell tumor fthe lung
b. Inflammatory myofibroblastic tumor
c. Malakoplakia
d. Metastatic renal cell carcinoma
e. Mycobacterial pseudotumor
f.

74.

Pulmonary Cryptococcus

Which enity is composed of clear cells which are positive for HMB45?
a. Clear cell tumor ofthe lung
b. Inflammatory myofibroblastic tumor
c. Malakoplakia
d. Metastatic renal cell carcinoma
e. Mycobacterial pseudotumor
f.

75.

Pulmonary Cryptococcus

Which entity may produce a wide spectrum of histologic changes including


granulomas fibrohistiocytic masses histiocytic infiltrates and mucoid

pneumonia?
a. Clear cell tumor ofthe lung
b. Inflammatory myofibroblastic tumor
c. Malakoplakia
d. Metastatic renal cell carcinoma
e. Mycobacterial pseudotumor
f.
76.

Pulmonary Cryptococcus

Which entity is characterized by prominent eosinophils, intra-alveolar


fibrin and macrophages?
a. Eosinophilic pneumonia

77.

Which entity is characterized by intra-alveolar hemosiderin-laden


macrophages and neutrophilic infiltrates located predominantly in the
alveolar septa?
a. Diffuse alveolar hemorrhage with capillaritis

78.

Which entity is characterized by hyaline membrane formation and


sparse/absent inflammatory cell infiltrates?
a. Diffuse alveolar damage

79.

Which ofthe following is considered noninvasive?


a. Adenocarcinoma, primary pulmonary
b. Bronchoalveolar carcinoma, nonmucinous type
c. Metastatic renal cell carcinoma
d. Mucinous (colloid) adenocarcinoma primary oulmonary
e. All ofthe above are invasive

80.

Which ofthe following is typically negative for both TTF1 and CDX2?
a. Bronchioloalveolar carcinoma mucinous type

81.

Which entity is most likely to be TTF1 negative CK7 negative and CDX2
positive?
a. Metastatic colon carcinoma

82.

Which tumor may present as either a localized mass or diffuse pleural


thickening and is composed of spindled cells which show strong positive
staining with bcl2 variable patchy staining for CK and calretenin and
negative for CD34?
a. Synovial sarcoma

83.

Which tumor presents as a large pleural based mass and is composed of


spindled cells in a collagenous background which show positive staining for
CD34 and bcl2 and lack staining with CK and calretenin?
a. Solitary fibrous tumor

84.

Which tumor presents as multiple pleural nodules or diffuse pleural


thickening and is composed of spindled cells which are diffusely and
strongly positive with CK variably positive for calretenin and negative for
bcl2 and CD34?
a. Malignant mesothelioma, sarcomatoid type

85.

Which tumor is composed of cytologically bland tall columnar cells with


apical mucin growing in a purley lepidic patternwithout areas ofstromal,
pleural, or vascular invasion?
a. Bronchoalveolar carcinoma, mucinous type

86.

Which tumor is a recently described variant of mucinous pulmonary


adenocarcinoma which shows positive staining with CK20 and CDX2 in the
majority of cases?
a. Adenocarcinoma, goblet cell type

87.

Which tumor is characterized by a fibrous walled cyst lined by a


cytologically bland mucinous epithelium?
a. Mucinous cystadenoma

88.

Which tumor is characterized by mixed blastomatous and sarcomatous


features?
a. Pleuropulmonary blastoma

89.

Which entity is characterized by enlarged cells with eosinophilic


intracytoplasmic inclusions which may be arranged in a paranuclear
distribution and no intranuclear inclusions?
a. Respiratory syncytial virus pneumonia

90.

Which enity is characterized by cells without significant enlargement


containing basophilic nuclear inclusions which obscure the nuclear
membrane (smudge cells) and no cytoplasmic inclusions?
a. Adenovirus pneumonia

91.

Which entity is characterized by enlarged frequently multinucleated cells


with eosinophilic nuclear inclusions and no cytoplasmic inclusions?
a. Herpes simplex pneumonia

92.

Which neoplasm is most commonly seen in the upper respiratory tract?


a. Extramedullary plasmacytoma

93.

Biphasic malignant melanoma may have all the below characteristics


except
a. Intracytoplasmic mucin

94.

From the following list the single best criterion for separating biphasic
pulmonary blastoma from biphasic synovial sarcoma
a. Supranuclear and subnuclear vacuoles in the epithelial component

95.

Which is true regarding biphasic pulmonary blastoma?


a. May have areas ofchondrosarcoma, rhabdomyosarcoma or
osteosarcoma

96.

Which ofthe following is true regarding diffuse parenchymal amyloidosis


in the lung?
a. The histologic appearance is essentially indistinguishable from
pulmonary involvement by systemic light chain disease
b. It is never associated with underlying systemic amyloidosis
c. It is almost always causes severe respiratory impairment
d. Most cases are familial or associated with hemodialysis
e. In most cases the type ofamyloid deposited is AA

97.

Which disease is characterized by diffuse pulmonary fibrosis without


temporal heterogeneity or hyaline membrane formation?
a. Non-specific interstitial pneumonia (NSIP), fibrosing type

98.

Which disease is characterized by patcy interstitial fibrosis which shows


temporal heterogeneity in the form offibroblastic foci?
a. Usual interstitial pneumonia

99.

Which pulmonary lesion is characterized by solid cell nests, furiform tumor


cells with peripheral palisading and a CD56 negative CK 5/6 positive
phenotype?
a. Basaloid carcinoma

100.

Which tumor typically shows prominent necrosis and increased mitotic


activity with strong membranous immunopositivity for CD56 and dot like
cytoplasmic staining for low molecular weight CK?
a. Small cell carcinoma

101.

Which tumor is combined with keratinizing squamous carcinoma in a high


percentage of cases?
a. Basaloid carcinoma

102.

Which lesion is associated with tuberous sclerosis complex and is typically


negative for HMB45?
a. Multifocal micronodular pneumocyte hyperplasia

103.

which lesion is associated with smoking and contains S100 positive cells?
a. Langerhans cell histiocytosis

104.

Which ofthe following immunohistochemical markers is most specific for


mesothelail origin?
a. Calretenin

105.

Which type ofmesothelioma has the best prognosis?


a. Multicystic

106.

Which ofthe following tumors has the poorest prognosis?


a. Bronchoalveolar carcinoma, type 3 (with central sclerosis)

107.

Which ofthe following immunohistochemical staining patterns is


characteristic of primary pulmonary adenocarcinoma?
a. CK 20 -, CK 7 + , TTF1 +

108.

Which ofthe following is considered most important in assessing the


prognosis of bronchioloalveolar carcinoma?
a. Extent of tumor spread

109.

A balanced translocation involving the EWSgene on chromosome 22 is


seen in
a. PNET

110.

Antibodies to the MIC2 gene product are seen in all except


a. Ewing's sarcoma
b. Neuroblastoma

c. Acute lymphoblastic lymphomas and leukemias


d. Wilm's tumor
e. PNET
111.

A feature distinguishing Ewing's sarcoma from PNET is


a. More favourable prognosis

112.

Which lesion is associated with an abnormal reduction in weight and/or


volume of the lung
a. Pulmonary hypoplasia

113.

Which lesion results in an orederly increase in size and/or number in the


structural components ofthe pulmonary lobules
a. Pulmonary hyperplasia

114.

Which lesion is a hamartoma associated with the tuberous sclerosis


complex
a. Micronodular pneumocyte hyperplasia

115.

Which has intracytoplasmic inclusions only


a. Respiratory syncytial virus pneumonia

116.

Which has intracytoplasmic and intranuclear inclusions


a. Measles pneumonia

117.

Which has intrabronchiolar neutrophilic exudate with plasma cell rich


chronic inflammation in bronchial wall
a. Mycoplasma pneumonia

118.

Which ofthe following pulmonary infections is characterized by


necrotizing vascultits?
a. Pseudomonas pneumonia

119.

Granulomatous inflammation is commonly associated with which ofthe


following?
a. Cryptococcus

120.

Which ofthe following statements is true?


a. Viral culture is more sensitive than histologic examination of tissue in

diagnosing CMV pneumonia


b. Legionella pneumonia is caused only by legionella pneumophila species
c. Pulmonary tularemia is characteristically associated with acute
necrotizing vasculitis
d. Legionella bacteria are easily identified in tissue sections stained with
Gram's stain
e. Actinomycosis commonly stains with an AFB
121.

Which lesion is characterized by positive staining with smooth muscle


markers and HMB45?
a. Lymphangioleiomyomatosis

122.

Which lesion is characterized by mutations ofthe tuberous sclerosis genes


TSC1 or TSC2?
a. Lymphangioleiomyomatosis

123.

Which lesion may involve the lymph nodes or viscera, is associated with
HHV8, is positive for endothelial markers and is negative for HMB45?
a. Kaposi sarcoma

124.

which tumor is characterized by an immature mesenchymal component


and beta-catenin mutations?
a. Carcinosarcoma
b. Pleomorphic carcinoma
c. Pulmonary blastoma
d. Squamous cell carcinoma

125.

which tumor is defined in the current WHO classification as a biphasic


tumor with heterologous differentiation such as bone, cartilage or skeletal
muscle?

a. Carcinosarcoma
b. Pleomorphic carcinoma
c. Pulmonary blastoma
d. Squamous cell carcinoma

126.

Pleomorphic carcinoma and carcinosarcoma may harbpr EGFR and KRAS


mutation.

a. True
b. False

6.
1.

Mediastinum

Patient with solitary mediastinal mass showed hyalinized lymphoid tissue and
abundant plasma cells
a. Castleman's
b. Lymphoplasmacytic
c. DiGeorge

2. 20 year old male with mediastinal lymphoblastic lymphoma, the cells will most
likely be
a. T cells
b. Bcells
3. What is positive in thymic carcinoma
a. CD5
4. Which immunohistochemical study helps to distinguish thymic from non-thymic
carcinoma
a. CD5
b. CK
c. CK7
d. EMA
e. CEA
5. Which of the following lesions commonly occurs in the posterior mediastinum
a. Bronchogenic cyst
b. Lymphoma
c. Thymoma
d. Sarcoidosis
e. Schwannoma
6. Which of the following thymic lesions is commonly associated with myasthenia
gravis
a. Thymic follicular hyperplasia
b. Thymoma

c. Thymitis
7. Most common mediastinal lymphoma in children
a. Lymphoblastic lymphoma
b. Non-hodgkin
c. Diffuse large B cell
d. Mantle zone
8. Which of the following presents as a thymic mass
a. Lymphoblastic lymphoma
9. Patient with DiGeorge most likely will present to the pediatrician with
a. Infantile hypocalcemia
b. Recurrent infections
10. Which tumor is poorly circumscribed, comprised of bland spindle cells and
expresses B-catenin?
a. Desmoid tumor
b. Malignant mesothelioma, desmoplastic type
c. Schwannoma
d. Solitary fibrous tumor
e. Synovial sarcoma
11. Which tumor may be associated with hypoglycemia or hypertrophic pulmonary
osteoarthropathy?
a. Desmoid tumor
b. Malignant mesothelioma, desmoplastic type
c. Schwannoma
d. Solitary fibrous tumor
e. Synovial sarcoma
12. Which tumor may express CK and bcl-2 but is usually negative for CD34 and is
additionally characterized by t(x;18)?
a. Desmoid tumor
b. Malignant mesothelioma, desmoplastic type
c. Schwannoma
d. Solitary fibrous tumor
e. Synovial sarcoma

13. Which tumor is characterized by positive staining for CD5 and CD117
a. Follicular dendritic cell sarcoma
b. Nodular sclerosis classical Hodgkin lymphoma
c. Primary mediastinal large B cell lymphoma
d. Spindle cell thymoma
e. Thymic carcinoma
14. Which tumor is typically defined by positive staining for CD21 and CD35/23
a. Follicular dendritic cell sarcoma
b. Nodular sclerosis classical Hodgkin lymphoma
c. Primary mediastinal large B cell lymphoma
d. Spindle cell thymoma
e. Thymic carcinoma

15. Which tumor is characterized by CD30 and CD20 positive cells with clear
cytoplasm in a background ofdelicate interstitial fibrosis
a. Follicular dendritic cell sarcoma
b. Nodular sclerosis classical Hodgkin lymphoma
c. Primary mediastinal large B cell lymphoma
d. Spindle cell thymoma
e. Thymic carcinoma
16. Which tumor exhibits consistent nuclear positivity for B catenin?
a. Desmoid tumor
b. Low grade fibromyxoid sarcoma
c. Malignant peripheral nerve sheath tumor
d. Myxofibrosarcoma
e. Myxoid liposarcoma
f.

Schwannoma

g. Solitary fibrous tumor


17. Which tumor lacks the alternating areas ofmyxoid and fibrous areas but is
consistently positive for CD34?
a. Desmoid tumor
b. Low grade fibromyxoid sarcoma
c. Malignant peripheral nerve sheath tumor
d. Myxofibrosarcoma
e. Myxoid liposarcoma
f.

Schwannoma

g. Solitary fibrous tumor


18. Which tumor is characterized by alternating areas offibrous and myxoid zones,
curvilinear blood vessels, alternations in FUS gene and absence of lipoblasts?
a. Desmoid tumor
b. Low grade fibromyxoid sarcoma
c. Malignant peripheral nerve sheath tumor
d. Myxofibrosarcoma
e. Myxoid liposarcoma
f.

Schwannoma

g. Solitary fibrous tumor


19. Which of the following lesions is MOST commonly encountered in the anterior
mediastinum?

a. Neurofibroma
b. Paraganglioma
c. Thymoma
d. Schwannoma
e. Liposarcoma
20. In the mediastinum, the hemangiopericytomatous vascular pattern is unique to
spindle cell thymomas.
a. True
b. False
21. Thymic carcinomas are the only thymic epithelial neoplasm capable of
biologically aggressive behavior.
a. True
b. False
22. Which of the following statements is incorrect?
a. Mitotic activity, nuclear pleomorphism, and the presence ofnecrosis are
poorly correlated with clinical behavior
b. Paragangliomas can be differentiated from most epithelial tumors based
upon their negativity for CK and EMA and positivity for neuroendocrine

markers
c. Paragangliomas can be functional (catecholamine-secreting) or nonfunctional
d. Paravertebral paragangliomas are more likely to be functional than aortic
body paragangliomas
e. The presence of multiple tumors along the vertebrae in the thorax and
abdomen is indicative ofmetastasis
23. Mitotic activity is diagnostic ofmalignancy in paragangliomas
a. True
b. False
24. Which of the following associations with paraganglioma is incorrect
a. Carney syndrome
b. Carney triad
c. MEN II
d. Neurofibromatosis type 1 (von-Recklinghausen disease)
e. Von-hippel landau syndrome
25. Which of the following lesions is LEAST likely encountered in the anterior
mediastinum?
a. Germ cell tumor
b. Mediastinal (thymic) large B cell lymphoma
c. Schwannoma
d. Thymic cyst
e. Thymoma
26. Capsular invasion in thymic epithelial tumors is diagnostic of thymic carcinoma
a. True
b. False

27. Which of the following associations is most correct?


a. Myasthenia gravis and thymic follicular hyperplasia
b. Nodular sclerosis variant ofHodgkin lymphoma and CD45 expression
c. Seminomas and elevated serum AFP
d. Spindle cell thymoma and cytologic atypia
e. Thymomas and immunophenotypically aberrant T cells
28. Which entity is characterized by positive staining for CD20 and bcl2?
a. Angiofollicular hyperplasia (Castleman disease)
b. Follicular lymphoma
c. Lymphocyte-predominant thymoma
d. Plasmacytoma
e. Reactive follicular hyperplasia
f.

Regressing follicular hyperplasia

29. Which entity has a preponderance of CD5 positve lymphoid cells suggesting an
abnormal proliferation of CD5 positive lymphocytes stimulated by specific
lymphokinases as its possible histiogenesis?
a. Angiofollicular hyperplasia (Castleman disease)
b. Follicular lymphoma
c. Lymphocytes-predominant thymoma
d. Plasmacytoma
e. Reactive follicular hyperplasia
f.

Regressing follicular hyperplasia

30. Which entity may have Hassal's corpuscles and large cells positive for CK?
a. Castleman disease
b. Follicular lymphoma
c. Lymphocytes-predominant thymoma
d. Plasmacytoma
e. Reactive follicular hyperplasia
f.

Regressing follicular hyperplasia

31. Which neoplasm exhibits a paracortical pattern ofgrowth in lymph nodes,


frank cytologic atypia, CK -, S100 + , HMB45 and ultrastructurally prominent
cell processes lacking desmosomes?
a. Follicular dendritic cell tumor/sarcoma

b. Interdigitating dendritic cell sarcoma


c. Large B cell lymphoma
d. Large cell neuroendocrine carcinoma
e. Malignant melanoma
f.

Thymoma

32. Which tumor exhibits syncytial aggregates of benign appearing epithelioid cells
admixed with lymphocytes, prominent perivascular spaces, and a CK + , S100 -,
synaptophysin immunophenotype?
a. Follicular dendritic cell tumor/sarcoma
b. Interdigitating dendritic cell sarcoma
c. Large B cell lymphoma
d. Large cell neuroendocrine carcinoma
e. Malignant melanoma
f.

Thymoma

33. Which neoplasm typically shows tumor cells in whorls and fascicles admixed
with lymphocytes, expresses CD21, CD35 and clusterin (CK and HMB45 -) and
ultrastructurally shows prominent desmosome-rich cell processes?
a. Follicular dendritic cell tumor/sarcoma
b. Interdigitating dendritic cell sarcoma
c. Large B cell lymphoma
d. Large cell neuroendocrine carcinoma
e. Malignant melanoma
f.

Thymoma

34. Which tumor occurs more commonly in the middle or posterior mediastinum is
positive for neuroendocrine markers but is typically negative for CK?
a. Paraganglioma
35. Which tumor occurs in the anterior mediastinum is positive for neuroendocrine
markers and CK has a low mitotic rate and may be associated with Cushing

syndrome?
a. Thymic carcinoid
36. Which tumor occurs as an anterior mediastinal mass shows lobular growth with
admixed lymphocytes and is positive for keratin while being negative for
chromogranin and synaptophysin?
a. Thymoma
37. Which entity often arises from pleura and is richly collagenized?
a. Carcinoma with thymic like elements (CASTLE)
b. Germ cell tumor
c. Hodgkin lymphoma
d. Metastatic carcinoma
e. Non-hodgkin lymphoma
f.

Solitary fibrous tumor

g. Thymic carcinoid
h. Thymoma
38. Which condition occurs in multiple microscopic variants and may be associated
with autoimmune disorders?
a. Thymoma
39. Which lesion is usually anatomically associated with thyroid tissue?
a. Carcinoma with thymic like elements (CASTLE)
40. Vascular proliferation and interfollicular or diffuse proliferation ofintermediate
size atypical T lymphocytes with clear cytoplasm and prominent mitotic activity
a. Angioimmunoblastic t cell lymphoma
41. Effacement ofthe nodal architecture by mixed cell proliferation with scattered
atypical large cells with polylobated nuclei
a. Hodgkin lymphoma
42. Prominent interfollicular proliferation of blood vessels and small lymphocytes
obliteration oflymph node sinuses and abnormal follicles with regressive

germinal centers and onion-scaling lymphocytes


a. Castleman disease
43. Which entity is the most common primary mediastinal germ cell tumor?
a. Mature cystic teratoma
44. Which entity includes a sarcomatous or carcinomatous component associated
with disorganized mature tissues?
a. Teratoma with malignant components
45. Which enitity includes primitive neuroepithelail tubules among disorganized
mature tissue?
a. Immature teratoma
46. Which of the following statements about thymic carcinoid is true?
a. It may be associated with symptomatic neuromuscular abnormalities
b. It is usually encapsulated and exhibits fibrous trabeculae
c. It is associated with paraneoplastic phenomena in 90% ofcases
d. It is commonly associated with the complete carcinoid syndrome
47. The lymphocytes involved in thymic follicular hyperplasia
a. CD20 and CD21 + and TdT negative
48. The most importatnt prognostic indicator for thymoma is
a. Clinicopathologic stage
49. Which of the following is ost likely in carcinoid ofthymic origin?
a. Immunoreactivity for chromogranin and CK and identification of
neurosecretory granules by EM
b. immunoreactivity for LCA and CD20
c. immunoreasctivity for placental alkaline phosphatase and leu7 and negative
CK stain
d. immunoreactivity for chromogranin negative CK and identification of
neurosecretory granules by EM

e. immunoreactivity for CD15 and CD30 and negative LCA


50. MEN are most likely associated with which ofthe following mediastinal tumors?
a. Carcinoid tumor
51. By definition invasive thymoma is a thymic epithelial tumor with cytologically
malignant cells
a. True
b. False

7.
1.

Thyroid and Parathyroid

Chernobyl nuclear disaster showed increase in which tumor in children


a. Thyroid cancer

2. Which variant of PTC is seen in children


a. Solid variant
3. Which thyroid tumor shows strong CK19 positivity
a. PTC
b. Follicular carcinoma
c. Follicular adenoma
d. Hurthle cell neoplasm
4. Which entity is histologically characterized by diffuse follicular hyperplasia with
prominent papillary architecture absence ofresidual normal thyroid
parenchyma follicular epithelial cells with round and uniform nuclei minimal to
absent colloid production and the presence ofan associated lymphocytic cell
infiltrate
a. Grave's disease
b. deQuervian
c. Reidel
5. Genetic abnormality of Medullary thyroid carcinoma
a. RET proto-oncogene
6. Which of the following variants of PTC has poor prognosis
a. Columnar cell variant
7. Secondary hyperparathyroidism is due to
a. Chronic renal failure
8. Which thyroid tumor is most likely to undergo clear cell change
a. Hurthle cell tumors

9. Unlike Hashimoto's thyroiditis, De Quervian is likely to have


a. Complete recovery
10. Anaplastic thyroid cancer
a. Metastasis at the time ofdiagnosis
b. A third ofthe cases are associated with well-differentiated tumor
11. 45 year old woman presents with a progressively enlarging mass in the
neck.the patient denies family history o fthyroid disease. FNA shows medullary
carcinoma. To which clinical group does this patient belong?
a. A sporadic case ofmedullary carcinoma lacking C cell hyperplasia
b. A familial case ofmedullary carcinoma without an associated
endocrinopathy
c. MEN 2a
d. MEN2b
12. The majority of medullary thyroid carcinomas associated with MEN2a and
MEN2b contain mutations in which proto-oncogene?
a. RAS
b. PTEN
c. RET
d. P53
e. B-catenin
13. Which staining pattern is most consistent with medullary thyroid carcinoma?
a. TTF1-, calcitonin-, LMWCK + , CEA+
b. TTF1+ , calcitonin+ , LMWCK -, CEAc. TTF1-, calcitonin+ , LMWCK + , CEA+
d. TTF1+ , calcitonin-, LMWCK + , CEA-

14. CASTLE can be distinguished from primary or metastatic squamous cell


carcinoma ofthe thyroid
a. Membranous CD5 and CD117 positivity
b. Epstein-barr virus positivity
c. Close resemblance of CASTLE to lymphoepithelioma-like carcinoma ofthe
thymus
d. P63 positivity
e. Lymph node involvement
15. The immunohistochemical profile ofCASTLE includes
a. Keratin and p63 positivity
b. CD5 and CD117 positivity
c. Chromogranin and synaptophysin positivity
d. Absence ofCD20/CD45 + lymphocytes
e. Presence ofCD1a/TdT + small lymphocytes in the stroma
16. CASTLE shares with lymphoepithelioma-like carcinoma ofthe nasopharynx all
ofthe following except
a. Dense lymphoplasmacytic stromal infiltrates
b. Propensity for lymph node metastases
c. Good response to treatment
d. Uniform population of keratin positive cells with prominent nucleoli
e. Association with Epstein-Barr virus
17. Which entity has a propensity for hematogenous spread?
a. Follicular adenoma
b. Follicular carcinoma
c. PTC, classical type
d. PTC, columnar cell type
e. PTC, cribriform-morula type
18. Which entity is associated with familial adenomatous polyposis (FAP)?
a. Follicular adenoma
b. Follicular carcinoma
c. PTC, classical type
d. PTC, columnar cell type
e. PTC, cribriform-morula type
19. The most aggressive clinical course is seen with which variant ofPTC?

a. Follicular adenoma
b. Follicular carcinoma
c. PTC, classical type
d. PTC, columnar cell type
e. PTC, cribriform-morula type
20. The finding of a sclerotic process involving the thyroid with bland nuclear
features and lacking vascular occlusion by spindle cells is most compatible with
this entity?
a. Angiosarcoma
b. Malignant lymphoma
c. Papillary carcinoma
d. Riedel thyroiditis
e. Undifferentiated (anaplastic) carcinoma
21. Which entity shares overlap ofimmunohistochemical markers and clinical
course with undifferentiated carcinoma and frequently is composed of

epithelioid cells?
a. Angiosarcoma
b. Malignant lymphoma
c. Papillary carcinoma
d. Riedel thyroiditis
e. Undifferentiated carcinoma
22. Which of the above tumors is characterized by extensive vascular plugging
with malignant spindled cells that shows the majority of which 70% positive
staining with p53?
a. Angiosarcoma
b. Malignantlymphoma
c. Papillary carcinoma
d. Riedel thyroiditis
e. Undifferentiated carcinoma
23. Which entity is histologically characterized by the presence ofa discrete mass
showing prominent papillary architecture and a constellation ofcytologic
alterations that include enlarged nuclei with irregularities in size and shape
dispersed to ground glass appearing nuclear chromatin nuclear crowding
nuclear grooves and intranuclear inclusions
a. Papillary carcinoma
24. Which entity is histologically characterized by the presence ofan encapsulated
mass with capsular invasion and angioinvasion and cells with hyperchromatic
round and regular nuclei
a. Follicular carcinoma
25. Which lesion is encapsulated, usually cystic, and contains papillae oriented
toward the center ofthe lesion and includes follicles in some of the papillae?
a. Follicular nodule with papillary hyperplasia
26. Which of the following is not true ofpapillary carcinoma?
a. Cervical lymph node metastasis is present in less than 5% at diagnosis
27. Which of the following is required for the diagnosis ofPTC?
a. Typical nuclear features
28. A thyroid lesion thought to have a viral causation

a. Subacute thyroiditis
29. A fibrous lesion characterized by keloid-like bands that may extend into
cervical soft tissue
a. Riedel thyroiditis
30. A fibrous lesion purportedly associated with an unusual variant of
mucoepidermoid carcinoma
a. Hashimoto thyroiditis fibrous variant
31. Monoclonal CEA positivity with a positive family history is supportive ofwhich
diagnosis?
a. Medullary thyroid carcinoma
32. Which tumor is characterized by a multifocal growth pattern with subepithelial
cell balls protruding into thyroid follicles?
a. Metastatic neuroendocrine carcinoma
33. Which tumor displays the following immunoreactivity pan neuroendocrine
positivity and S100 positive sustentacular cells
a. Paraganglioma
34. An 11 year old girl with a 2.5 cm circumscribed centrally cystic thyroid nodule.
Histology showed an encapsulated nodule composed of papillae and follicles
with stromal edema and round polarized nuclei
a. Papillary hyperplastic nodule
35. An 8 year old girl deafgirl with a large goiter. Her developmental milestones,
which were similar to two older siblings, indicated she was "slow". The goiter
was composed of encapsulated and nonencapsulated, variably sized nodules.
Both nodular and nonnodular thyrid areas contained atypical nuclei
a. Dyshormonogenetic goiter

36. Which thyroid lesion is characterized histologically by a lymphocytic infiltrate in


the thyroid gland with germinal center formation oxyphilic metaplasia of
thyroid follicular cells follicular colonization absence of extension outside the
thyroid gland and immunoreactivity with both B cell and T cell markers?
a. Chronic lymphocytic thyroiditis
37. Which thyroid lesion is characterized by a lymphocytic infiltrate in the thyroid
gland coupled with laboratory evidence of hyperthyroidism, immunoglobulin
deposition within the thyroid, thymic enlargement, and almost invariable
presence ofthyroid stimulating stimulating antibodies
a. Grave's disease
38. Which thyroid lesion is characterized by a lymphocytic infiltration with follicular
colonization extension of the infiltrate beyond the thyroid capsule into
perithyroidal soft tissue, immunoreactivity with CD20 but not CD45RO, and
light chain restriction?
a. Marginal zone lymphoma ofMALT type
39. Oxyphil adenomas of the parathyroid glands differ from chiefcell adenoma in
which of the following?
a. Lareger size
b. More follicular pattern
c. Lower serum calcium
d. Decreased intracellular fat
e. All of the above
40. Parathyroid carcinoma differs from parathyroid adenoma by
a. Invasion ofneighboring structures
41. The frequency of hyperparathyroidism in MEN syndromes
a. MEN I> MEN II
42. Follicular variant of PTC has a clear long-term survival advantage over
conventional PTC
a. True
b. False

43. Presence ofelongated branching follicles filled with hypereosinophilic colloid is


sufficient to establish a diagnosis of FVPTC
a. True
b. False
44. Major criteria for the diagnosis of FVPTC include a follicular pattern of growth
lined by cells with enlarged nuclei, clearing ofthe nuclear chromatin,
longitudinal nuclear grooves, and intranuclear pseudoinclusions
a. True
.bF alse
45. Focal lymphocytic thyroiditis is an early form ofHashimoto thyroiditis
a. True
b. False
46. The fibrous variant ofHashimoto thyroiditis is associated with multifocal
(systemic) fibrosclerosis
a. True
b. False
47. The combination of goiter, hypothyroidism, and circulating anti-thyroglobulin
and/or antithyroid peroxidase antibodies is diagnostic of Hashimoto thyroiditis
a. True

b. False

8.
1.

Gastrointestinal Tract

Pseudomembranes, flask-shaped ulcer, histiocyte-like structures with


hemophagocytosis
a. Amoeba

2. Whipple triad
3. Which is the primary site for a tumor with the following characteristics: Signet
ring morphology, positive for: MUC2, CDX2, MUC5AC, CK20; and negative
for:MUC1, PSA, TTF-1, CK7
a. Ling
b. Prostate
c. Pancreas
d. Colon
e. Breast
4. Gardner syndrome is associated with which ofthe following
a. Elastofibroma
b. Dermatofibroma
c. Spindle cell lipoma
d. Nuchal fibroma
e. Fibrolipoma
5. Which of the following is least likely to be associated with Gardner syndrome
a. Spindle cell lipoma
b. Fibrolipoma
6. In the recent update ofthe TNM classification of colon cancer, which ofthe
following is not present anymore

a. Tumor size
b. Liver metastasis
c. Number oflymph nodes
d. Muscularis propria invasion
e. Measurement ofthe distance ofinvasion from the muscularis propria to
the peri-colic fat
7. Favourable location of carcinoid
a. Appendix
b. Colon
c. Stomach
8. Bland looking spindle cell lesion which is locally aggressive and presents with
late metastases
a. GIST
b. Leiomyosarcoma
c. Low grade fibromyxoid sarcoma
9. Spindle cell lesion which shows bland cells in edematous myxoid stroma
arranged around blood vessels with eosinophils
a. GIST
b. Inflammatory fibroid polyp
c. Inflammatory myofibroblastic tumor
d. Leiomyosarcoma
10. The commonest location ofa peptic ulcer is
a. Gastric antrum

b. 2nd part of the duodenum


c. 1st part ofthe duodenum
d. GEJ
11. CDX2 is negative in
a. Colon cancer
b. Carcinoid ofappendix
c. Stomach adenocarcinoma
d. Ampulla ofvater adenocarcinoma
e. Cholangiocarcinoma
12. The most common complication ofpeptic ulcer
a. Perforation
b. Bleeding
c. Malignant transformation
d. Stenosis/ obstruction
13. In lymphocytic gastritis, the intraepithelial lymphocytes are ofwhich type
a. CD8 T cells
b. CD4 T cells
c. CD5 T cells
d. B cells
14. A gastric polyp in the pyloric area is composed inflammatory cells mainly
eosinophils along with myofibroblast proliferation
a. Hyperplastic polyp
b. Inflammatory polyp
c. Fundic gland polyp
d. Inflammatory fibroid polyp
15. What histologic feature can be used to distinguish Crohn's from ulcerative
colitis
a. Skip lesions
b. Granulomas
16. Colonic adenocarcinoma most likely arises from
a. Villous adenoma
17. Which of the following is a risk factor for gastric carcinoma

18. Signet ring carcinoma ofthe stomach presents as


a. Diffuse thickening of the wall
19. Carcinoid tumor
a. Low grade malignant tumor ofneuroendocrine cells
20. A baby with projectile vomiting
a. Pyloric stenosis
21. What is the most common gastric polyp
a. Hyperplastic polyp
b. Fundic gland polyp
c. Adenomatous polyp
22. Which test may be helpful in a patient with gastric MALT
a. Urea breath test (????)
23. Loss of E-cadherin function is the key ofthe development ofwhich type of
cancer
a. Diffuse gastric cancer with stromal signet ring cell type carcinoma
b. GIST
c. Gastric adenocarcinoma, intestinal type
d. Squamous cell carcinoma ofesophagus
24. Anal carcinoma
a. Positive for CK7, CK20 and B72.3
b. Positive for GCDFP-15
c. Positive for ER and PR
d. Mutation ofp53 and c-myc
25. Most common location for gangliocytic paraganglioma
a. Distal duodenum
b. Distal esophagus
c. Fundic gastric mucosa
d. Pylorus and antrum
26. Which of the following is most common in females
a. Collagenous colitis

b. Ulcerative colitis
c. Lymphocytic colitis
27. Patient with resection ofcolon cancer presents with mesenteric mass with a
white cut surface without necrosis or hemorrhage
a. Fibromatosis
b. Angiosarcoma
28. CDX2 is + vein all except
a. Carcinoid
29. By definition, signet ring carcinoma
a. > 50% signet ring
b. > 50% mucinous
30. Question about carcinoid prognosis
31. The most common cause ofintestinal obstruction
a. Volvulus
b. Neoplasm
c. Intussusception
d. Hernia
e. Adhesions
32. A filling defect on a barium examination ofthe GIT means
a. The lumen is dilated or there is outpouching
b. Something protruding into the lumen displaces the barium
c. Both
d. Neither
33. Each of the following applies to Hirschsprung disease, except
a. Intestinal obstruction
b. Absence ofganglion cells in myenteric plexus
c. Treated by removal ofdistended segment ofbowel
d. Toxic megacolon as complication
e. Narrow more distal segment exhibits characteristic deficiency of
development

34. Which congenital abnormality has onset at 2 4 weeks with a strong male
predominance?
a. Pyloric stenosis
b. Intestinal atresia
c. Meckel diverticulum
d. Hirschsprung disease
e. Annular pancreas
35. An 8 month old has episodes ofcolic with blood in his stool
a. Pyloric stenosis
b. Intestinal atresia
c. Meckel's
d. Hirschsprung
36. Congenital lesion that usually presents with vomiting in the 1stthree days oflife
a. Intestinal atresia
37. Small intestinal type mucosa in a gastric biopsy is most likely due to
a. Chronic gastritis
b. Congenital heterotropia
c. Precancerous dysplasia
d. Metastatic carcinoma
e. Benign neoplasm
38. Absence ofganglion cells is a diagnostic feature of
a. Hirschsprung
39. The most common cause ofupper GIbleeding
a. Esophageal varices
b. Gastric carcinoma

c. Peptic ulcer
d. Gastritis
40. Mucosal necrosis
a. Pseudomembranous colitis
41. Mechanism ofpseudomembranous colitis
a. Elaboration ofenterotoxin following colonization ofthe colon
b. Ingestion ofpreformed toxin
c. Invasion and destruction ofmucosa by organism
d. Host immune response to the organism
42. Primary carcinoma is least common in
a. Esophagus
b. Stomach
c. Small intestine
d. Colon
43. Carcinoid syndrome
44. Complication ofhiatal hernia
a. Esophagitis
45. Esophageal varices
a. In the submucosa
46. Multiple recurrent ulcers in the proximal and distal duodenum for several
years are likely to be associated with
a. Islet cell adenoma ofthe pancreas
b. Gastrin secreting tumors
c. Heavy aspirin intake
d. Smoking
e. Excessive bile secretion

47. Feature of Zollinger-Ellison syndrome


a. Hypoglycemic attack
b. Obesity
c. Gastric hyperchlorhydria
d. DM
e. Fainting spells
48. Commonest site of peptic ulcer
a. Lesser curvature at antral body junction
b. Anterior wall at duodenal verge
c. Greater curvature in mid-antrum
d. Gastro-esophageal junction
49. Stress ulcer
a. Extensive burns
50. Rectal polyp in a child is most likely
a. Juvenile polyp
51. Which type of polyp is most likely in a patient with watery mucoid diarrhea
a. Juvenile polyp
b. Adenomatous polyp
c. Villous adenoma
d. Carcinoma
52. A bulky tumor ofthe rectum exhibits a delicate, velvety, easily bleeding
surface and no pedical with a base as wide as the apex
a. Inflammatory polyp
b. Villous adenoma
c. Mucinous carcinoma
d. Lymphoid polyp
e. Leiomyoma
53. What percent of villous adenoma has cancer at the time oftheir removal
a. 10%
b. 30%
c. 50%
d. 70%

54. Which best characterizes Peutz-Jeghers polyps?


a. Solitary, hamartomatous, not premalignant
b. Multiple, hamartomatous, not premalignant
c. Solitary, neoplastic, premalignant
d. Multiple, neoplastic, not premalignant
55. Which best predicts the biological behavior ofcolon cancer?
a. Age of the patient
b. Duration ofsymptoms
c. Region ofthe colon affected
d. Diameter ofthe cancer
e. Lymph node metastasis
56. Colon cancer commonly metastasizes to
a. Liver
b. Lung
c. Vertebral column
d. Small intestine
e. Kidney
57. Adenocarcinoma ofthe ascending colon is more likely than sigmoid to
a. Encircle the bowel, causing stricture or obstruction
b. Be bulky
58. Which of the following is least likely to undergo malignant transformation
a. Ulcerative colitis
b. Juvenile polyp
c. Villous adenoma
d. Polyp ofgardner's syndrome
59. What is the commonest cancer of the anus?
a. Squamous cell carcinoma
b. Adenocarcinoma
60. What is the most likely diagnostic histologic feature if the radiology of the
stomach shows gastric ulceration with a Hampton's line?
a. Granulomatous inflammation and giant cells

b. Ulceration, chronic inflammation, and fibrosis


c. Anaplastic glands with invasion
d. Hamartomatous polyp
e. Diffuse lymphocytic infiltration
61. Which of the following predisposes to squamous cell carcinoma of the
esophagus?
a. Achalasia
62. Villi distended by histiocytes
a. Whipple's disease
63. Which entity is characterized by mutations in KIT or PDGFRA and is usually
positive for DOG 1 by immunohistochemistry
a. Epithelioid GIST
b. Mixed adenoneuroendocrine carcinoma
c. Neuroendocrine carcinoma
d. Neuroendocrine tumor
64. A tumor with monotonous neuroendocrine morphology shows strong
synaptophysin positivity by immunohistochemistry, no mitotic activity and a

Ki67 index 4%. What is the best diagnosis?


a. Epithelioid GIST
b. Mixed adenoneuroendocrine carcinoma
c. Neuroendocrine carcinoma WHO grade 3
d. Neuroendocrine tumor WHO grade 1
e. Neuroendocrine tumor WHO grade 2
65. In which entity should the pathologist count mitotic figure in 50 HPF as part of
risk stratification
a. Epithelioid GIST
b. Neuroendocrine tumor WHO grade 1
c. Neuroendocrine tumor WHO grade 2
d. All of the above

66. This lesions consists ofcytologically bland spindle cells loosely arranged in an
edematous myxoid stroma that contains numerous eosinophils. The spindle
cells concentrically whorl around thin-walled vessels
a. GIST
b. Inflammatory fibroid polyp
c. Inflammatory myofibroblastic tumor
d. Leiomyoma
e. Leiomyosarcoma
67. This lesion is typically composed of long, well-oriented fascicles of elongate
spindle cells exhibit monotonous mitotically inactive cigar-shaped nuclei and
may exhibit extensive intratumoral hyalinization
a. GIST
b. Inflammatory fibroid polyp
c. Inflammatory myofibroblastic tumor
d. Leiomyoma
e. Leiomyosarcoma
68. A 61 year old man presents with iron deficiency anemia and a 6.5 cm gastric
mass. Histologically, the mass exhibits foci ofcoagulative necrosis and is
composed ofspindle and epithelioid cells some exhibiting signet ring type
morphology. The proliferative index is approximately 20% and tumor cells
express CD34 and DOG1. Tumor cells do not express muscle actin, CD117, or
pan CK and they are mucicarmine negative. What is the diagnosis
a. GIST
b. Inflammatory fibroid polyp
c. Inflammatory myofibroblastic tumor
d. Leiomyoma
e. Leiomyosarcoma
69. Which mesenteric lesion tends to have histologic variability from patient to
patient but typically contains areas of fibrosis, fat necrosis and/or chronic
inflammation?

a. GIST
b. Low grade fibromyxoid sarcomas
c. Mesenteric fibromatosis
d. Sclerosing mesenteritis
e. Solitary fibrous tumor
70. Which bland appearing lesion is locally aggressive and can develop late
metastases?
a. GIST
b. Low grade fibromyxoid sarcomas
c. Mesenteric fibromatosis
d. Sclerosing fibromatosis
e. Solitary fibrous tumor
71. Which mesenteric lesion often involves the bowel wall and shows nuclear
positivity for B-catenin by immune?
a. GIST
b. Low grade fibromyxoid sarcomas
c. Mesenteric fibromatosis
d. Sclerosing fibromatosis
e. Solitary fibrous tumor
72. Which of the following histologic findings are characteristic ofsclerosing
mesenteritis?
a. Atypical stromal cells with hyperchromatic enlarged irregular nuclei
b. Fibrosis chronic inflammation and fat necrosis
c. Infiltration into the adjacent bowel wall
d. Storiform or fascicular growth pattern of spindle cells

e. Myxoid change and keloid type collagen


73. Which of the following is correct about sclerosing mesenteritis?
a. Usually presents in childhood or young adulthood
b. Most cases present with intestinal obstruction
c. SMA often positive
d. Association with systemic IgG4 disease is well-established
e. Surgery is the treatment ofchoice in most cases
74. Which of the following features favor mesenteric fibromatosis over sclerosing
mesenteritis?
a. Fat necrosis
b. Infiltration into adjacent bowel wall
c. Intralesional lymphoid aggregates
d. Membranous staining with b-catenin
e. Dystrophic calcification
75. Which of the following best describes lymphocytic and collagenous colitis
a. Distorted mucosal architecture with variability in size shape and spacing of
glands
b. Preserved mucosal architecture inflamed lamina propria and increased
intraepithelial lymphocytes
c. Fringed fuzzy coating at the luminal side ofthe mucosa with no associated
inflammation or architectural disarray
d. Superficial mucosal necrosis with atrophic crypts hyalinization ofthe lamina
propria and features ofregeneration
76. Which special stain can be used to distinguish lymphocytic colitis from
collagenous colitis in equivocal cases
a. Trichrome
b. Warthin-starry
c. GMS
d. Gram
77. Which entity is typically associated with chronic watery diarrhea and normal or
near-normal endoscopic mucosal appearance during colonoscopy
a. Ulcerative colitis
b. Ischemic colitis
c. Intestinal spirochetosis

d. Collagenous colitis
78. Mucinous and signet ring cell carcinomas together account for approximately
what percentage of colorectal adenocarcinomas
a. 1%
b. 10%
c. 20%
d. 25%
79. Mucinous colorectal adenocarcinoma is associated with which one ofthe
following
a. A near 100% likelihood ofhereditary non-polyposis colorectal cancer
b. A near 100% likelihood ofhigh microsatellite instability
c. An excellent response to chemotherapy
d. A better prognosis compared to signet ring cell carcinoma
80. Which of the following is/are required for classification of a tumor as a signet
ring cell carcinoma
a. More than 50% ofthe tumor comprised ofmucin
b. More than 50% ofthe tumor cells are signet ring cells
c. High microsatellite instability
d. A family pedigree consistent with HNPCC
81. Pseudomembranes may form in this disease process but will have a paucity of
neutrophils
a. Amoebic colitis
b. Clostridium difficile pseudomembranous colitis
c. GVHD

d. Inflammatory bowel disease with pseudomembrane formation


e. Ischemic colitis with pseudomembrane formation
f.

Radiation colitis

g. Typhilitis
82. In which disease does examination ofthe colon reveal pseudomembranes with
siderophages and fibrosis oflamina propria?
a. Amoebic colitis
b. Clostridium defficile pseudomembranous colitis
c. GVHD
d. Inflammatory bowel disease with pseudomembrane formation
e. Ischemic colitis with pseudomembrane formation
f.

Radiation colitis

g. Typhilitis
83. In which disease does examination of the colon reaveal pseudomembranes
with flask-shaped ulceration containing histiocytoid structures that exhibit
erythrophagocytosis and a strong cytoplasmic PASreaction?
a. Amoebic colitis
b. Clostridium difficile pseudomembranous colitis
c. GVHD
d. Inflammatory bowel disease with pseudomembrane formation
e. Ischemic colitis with pseudomembrane formation
f.

Radiation colitis

g. Typhilitis
84. Which of the following feature favors radiation colitis over ulcerative colitis?
a. Cryptitis
b. Crypt architectural distortion
c. Lamina propria fibrosis
d. Perivascular hyalinization
85. Which of the following is a feature common to both active ulcerative colitis and
Crohn disease?
a. Cryptitis and crypt abscess

b. Discontinuous involvement of colon


c. Transmural inflammation
d. Well formed epithelioid granuloma
86. Creeping fat is a feature of which ofthe following
a. Crohn disease
b. Infectious colitis
c. Radiation colitis
d. Ulcerative colitis
87. The tumor that would most consistently demonstrate CD117 expression is
a. GIST
b. Hodgkin lymphoma
c. Liposarcoma
d. Mesenteric fibromatosis
e. Sclerosing mesenteritis
88. The entity characterized by inflammation and fat necrosis in early phases, and
a predominance of fibrosis as the process matures is
a. GIST
b. Hodgkin lymphoma
c. Liposarcoma
d. Mesenteric fibromatosis
e. Sclerosing mesenteritis

89. A bland fibroblastic proliferation in the mesentery with involvement of the


intestinal muscularis propria would be typical of
a. GIST
b. Hodgkin lymphoma
c. Liposarcoma
d. Mesenteric fibromatosis
e. Sclerosing mesenteritis
90. Immunoproliferative small intestinal disease typically occurs in asymptomatic,
older males.
a. True
b. False
91. Large B cell lymphoma and plasmacytoma can be reliably distinguished by the
use of antibodies to CD138, CD20, and CD79a.
a. True
b. False
92. CD117 expression may be seen in malignant melanoma and plasma cell
neoplasms as well as GIST.
a. True
b. False
93. In the USapproximately 5% of gastric carcinomas are diagnosed in patients
younger than 40 years
a. True
b. False
94. Patients with familial adenomatous polyposis coli carry a germline mutation in
the APC gene and are at significant risk for developing gastric carcinoma
a. True
b. False
95. Patients with signet ring cell carcinomas ofthe stomach can show hereditary
characteristics due to germline mutations in the CDH1 gene, which codes for

the E-cadherin protein.


a. True
b. False
96. Which of the following lesions shows nuclear expression of B catenin, negative
staining for S100 and variable positivity for KIT, smooth muscle actin and
desmin?
a. GIST
b. Leiomyoma
c. Mesenteric fibromatosis
d. Schwannoma
97. All of the following typically show KIT positive staining on
immunohistochemistry except
a. Malignant melanoma
b. Mastocytosis
c. Schwannoma
d. Seminoma
98. Which of the following is correct about GISTs?
a. Epithelioid GIST are more common in the small intestine

b. Imatinib (gleevec) therapy is not effective in tumors that are KIT negative
by immunohistochemistry
c. Immunohistochemical expression ofdesmin is common
d. Small intestinal GISTs are more likely to be malignant than gastric GISTs
99. Celiac disease predisposes to small bowel adenocarcinoma, which can occur
even after a long period of gluten free diet
a. True
b. False
100.

Celiac disease associated small bowel adenocarcinoma is characterized

by proximal location, high frequency of defective mismatch repair proteins,


and better patient survival, compared to small bowel adenocarcinoma of
patients whoc are not gluten sensitive.
a. True
b. False
101. Compared to the general population, there is no increased risk ofsmall bowel
adenocarcinoma in patients with Crohn disease
a. True
b. False
102.

Which ofthe following is true regarding radiation-induced colitis?

a. Granulomas may be present


b. Perivascular hyalinization helps distinguish it from Crohn disease
c. Radiation induced colitis is usually an acute event following radiation
d. Stricture formation and fistulae are not characteristic

103.

Which ofthe following statemnts regarding Crohn disease is FALSE?

a. Deep fissuring ulcers and fistula formation are seen in Crohn disease and
are not seen in ulcerative colitis
b. Non-caseating granulomas are seen in 60-90% of mucosal biopsy samples
c. The presence of small bowel involvement typically helps distinguish Crohn
disease from ulcerative colitis
d. Transmural lymphoid inflammation is typical ofthis entity
104.

Which site represents the commonest location of luminal GI

involvement by sarcoidosis?
a. Esophagus
b. Large bowel
c. Small bowel
d. Stomach
105.

Which ofthe following tumors typically lacks expression CDX-2?

a. Periampullary adenocarcinoma
b. Colorectal adenocarcinoma
c. Cholangiocarcinoma
d. Gastric adenocarcinoma
106.

Patients with familial adenomatous polyposis carry a germline mutation

in the APC gene and are at significant risk for developing gastric carcinoma
a. True
b. False
107.

Patients with PJsyndrome have a genetic defect in chromosomal

region 19p13 and are at significant risk for developing gastric carcinoma
a. True
b. False
108.

In the US approximately 5% of gastric carcinomas are diagnosed in

patients younger than 40 years.


a. True
b. False
109.

Which tumor typically is positive for CD34 and CD117?

a. Alveolar soft part sarcoma

b. GIST
c. Hemangiopericytoma/solitary fibrous tumor of soft tissues
d. Leiomyosarcoma
e. Perivascular epithelioid cell tumor (PEComa)
110. Which diagnosis implies the most aggressive natural history?
a. Carcinoid
b. Goblet cell carcinoid
c. Mixed carcinoid adenocarcinoma
d. Tubular carcinoid
111. Which is the most reliable feature for the light microscopic distinction
between tubular carcinoid and mixed endocrine-adenocarcinoma?
a. Chromogranin expression
b. Cytologic, especially nuclear, atypia
c. Cytoplasmic mucin
d. Desmoplastic response
112. Which tumor typically involves distal ileum, occurs with equal frequency in
both sexes with a peak in the sixth and seventh decades, and classically has an

insular pattern of growth?


a. Carcinoid tumor
113. Which entity has the most striking female predisposition?
a. Collagenous colitis
b. Lymphocytic colitis
c. Ischemic colitis
d. Ulcerative colitis
e. Neutrophilic enterocolitis
114. Pseudomembrane formation may be evident in which ofthe following diseases?
a. Ischemic colitis
b. Clostridium difficile associated colitis
c. Amebic colitis
d. All of the above
115. Stool culture is the most expedient preferred method for confirmation of
clostridium difficile associated colitis
a. True
b. False
116. Which of the following molecular events has been most specifically associated
with decreased cellular adhesion and diffuse infiltration characteristic ofsignet
ring carcinoma?
a. E cadherin inactivation
117. Which primary site is favored by the following immunohistochemical profile
exhibited by a metastatic signet ring carcinoma: CDX2 + , MUC1 -, MUC2 + ,
MUC5AC + , CK7 -, CK20+ , TTF -, PSA -?
a. Colon
118. Which of the following statemtns is true ofcolorectal signet-ring carcinoma?
a. The most common histologic variant of carcinoma involving the cecum
b. Occurs in older age group than other histologic variants ofcolorectal
carcinoma
c. In advanced disease is more often associated with peritoneal dissemination
than with liver metastases
d. Has a better prognosis adjusted for stage than non-mucinous carcinoma of
the colorectum

e. Is typically associated with a diffuse infiltrating scirrhous appearance


119. Which tumor is most likely to demonstrate defective DNA mismatch repair?
a. Colonic adenocarcinoma
120.

Wich tumor is a typical component ofhereditary nonpolyposis

colorectal cancer (Lynch) syndrome in women?


a. Endometrial carcinoma
121. The presence of defective DNA mismatch repair in gastrointestinal malignancy
is associated with an improved prognosis compared to the same type of
malignant neoplasm with intact DNA mismatch repair function
a. True
b. False
122.

The most common neoplasm ofthe middle esophagus typically

associated with long-standing chronic esophagitis


a. Squamous cell carcinoma
123.

Pedunculated esophageal lesion that most commonly arises in the

upper third of the esophagus predominantly occurs in men and is lined by


squamous epithelium
a. Giant fibrovascular polyp
124.

Typically a solitary exophytic esophageal lesion that occurs in middle-

aged men with the majority located in the lower esophagus


a. Squamous papilloma
125.

This condition is more common in the elderly, is typically acute and

segmental and features superficial mucosal necrosis as the hallmark histologic


findings
a. Ischemic colitis
126.

The fulminant phase ofchronic ulcerative colitis may be associated with

which one ofthe following


a. Fissuring ulceration of the colon
127.

The major short term implication of distinguishing Crohn colitis from

chronic ulcerative colitis at the time ofcolectomy for fulminant phase of colitis

a. Selection ofappropriate surgical reconstruction


128.

which ofthe following is most likely to be associated with development

ofhemolytic uremic syndrome?


a. Hemorrhagic colitis due to e. coli
129.

In a patient whoc is suspected to have CMV colitis which of the

following assays performed on biopsy tissue or stool is least likely to confirm


the diagnosis?
a. Stool viral culture
130.

A colorectal biopsy performed on an immunosuppressed patient with

colitis demonstrates intranuclear inclusions predominantly affecting the surface


epithelium including goblet cells; the inclusions are amphophilic ofcrescent or
targetoid shape and completely fill the affected nuclei. Which of the following
are they most likely to represent?
a. Adenovirus
131. Which lesion can arise in the setting ofpernicious anemia as a response to
hypergastinemia
a. Carcinoid tumor
132.

Which lesion is associated with EBV infection?

a. Lymphoepithelioma-like carcinoma
133.

Which entity is most commonly associated with duodenal ulcers?

a. Active chronic gastritis, H. pylori associated


134.

Which entity is associated with anti-intrinsic factor antibodies?

a. Chronic atrophic gastritis


135.

Which entity is associated with non-steroidal anti-inflammatory agents?

a. Regenerative gastropathy

136.

Which lesion is a benign nonneoplastic mucosal excrescence

characterized by cystically dilated glands?


a. Fundic gland polyp
137.

Which lesion is most strongly associated with peripheral eosinophilia?

a. Eosinophilic gastroenteritis
138.

Which is a polypoid esophageal neoplasm composed ofCK positive

spindle cells with or without a light microscopically identifiable squamous


component
a. Spindle cell carcinoma
139.

Which tumor exhibits strong immunnoreactivity for chromogranin and

many neurosecreoty granules by EM


a. Carcinoid
140.

Pseudomembranes or pseudomembrane-like structures may be seen

in all ofthe following entities except


a. Neutropenic colitis
b. Collagenous colitis
c. Clostridium difficile pseudomembranous colitis
d. Ischemic colitis
e. Hemorrhagic colitis syndromes
141. Clostridium difficle is the most frequently identified enteric pathogen
associated with antibiotic-associated diarrhea or colitis
a. True
b. False
142.

Which may feature eosinophilia and atypical lymphoid cells

a. Peripheral T cell lymphoma


143.

Which may feature edema and a relatively pure population of

eosinophils with minimal tissue damage?


a. Eosinophilic gastritis
144.

Which will feature moderate eosinophilia admixed with tryptase-

positive mononuclear cell population by immunoperoxidase staining?

a. Mast cell disease


145.

Which ofthe following is true?

a. In crohn's disease, biopsies ofendoscopically normal mucosa may display


colitis
b. Collagenous colitis occurs more often in men than in women
c. Approximately 50% of patients with chronic diarrhea and normal
endoscopic findings have either lymphocytic colitis or collagenous colitis
d. Diffuse mucosal lymphocytosis or lymphoplasmacytosis is common in GVHD
e. NSAID are associated with diffuse neutrophilic cryptitis
146.

Intraepithelial lymphocytosis is a prominent feature of colon biopsies in

all of the following except


a. Collagenous colitis
b. Chronic ulcerative colitis
c. Lymphocytic colitis

d. Refractory sprue
e. Brainerd diarrhea
147.

In individuals with chronic diarrhea and no HIV infection, the most

common biopsy finding is?


a. Histologically normal mucosa
148.

Esophagitis associated with inflammation ofthe oral cavity and ocular

and genital areas in characteristic of which entity?


a. Behcet syndrome
149.

Which infectious agent commonly involves epithelial cells, endothelial

cells, smooth muscle and ganglion cells ofthe GIT?


a. CMV
150.

Germline mutation in the STK11/LKB1 gene have been recognized as a

cause of
a. Peutz-Jeghers syndrome
151. The association of adenomatous polyposis with osteomas and soft tissue
fibromas is called
a. Gardner syndrome
152.

Classically, the lifetime risk ofcolorectal carcinoma in patients with APC

gene mutation-associated polyposis is


a. 100%
153.

Which gene is often hypermethylated in colonic medullary carcinomas?

a. MLH1
154.

Which ofthe following is required for a diagnosis ofcolonic high-grade

large cell neuroendocrine carcinoma?


a. Demonstration ofneuroendocrine differentiation by immunohistochemistry

155.

Which is not a characteristic feature ofcolon cancers that are

microsatellite unstable?
a. Diffuse chromogranin expression
156.

Which entity is characterized by multiple cysts in the bowel wall that

are often lined by histiocytes and giant cells with associated acute mucosal
necrosis?
a. Pneumatosis (cystoides) intestinalis related to ischemic bowel disease
157.

Which entity is characterized by cyst-like spaces that contain foamy

macrophages and are lined by endothelial cells?


a. Intestinal lymphangiectasia
158.

Which entity is associated with mucosal polyps and submucosal cysts

that are lined by benign intestinal epithelium?


a. Enteritis cystica profunda
159.

Which one of the following features strongly favors Crohn colitis over

ulcerative colitis?
a. Chronic inflammation in biopsies from upper GIT
b. Diffuse colonic involvement
c. Perianal disease
d. Presence ofprimary sclerosing cholangitis
160.

Which ofthe following infections can closely mimic Crohn disease by

showing ileocolonic involvement fissuring ulcers granulomas and transmural


inflammation?
a. Yersinia
161. Which of the following features favor Crohn over mycobacterium TB infection?
a. Granulomas in draining lymph nodes
b. Ileocolic involvement
c. Large and confluent granulomas

d. Longitudinal ulcers and cobblestoning


e. Prominent necrosis
162.

Which ofthe following is false ofgoblet carcinoids

a. Immunohistochemical stains are positive for CEA


b. Metastasis to the ovary can resemble poorly differentiated adenocarcinoma.
c. The growth pattern in the appendix is submucosal
d. They are more common in males
e. They have both glandular and endocrine elements
163.

GCC always express neuroendocrine markers(ie. Synaptophysin,

chromogranin,CD56).
a. True
b. False
164.

Which ofthe following findings is more suggestive of an ovarian

primary than a metastasis?


a. Bilaterality
b. Expansile pattern of growth
c. Nodular pattern of invasion
d. Tumor on ovarian surface
e. Young age at presentation

9.
1.

Salivary Glands

What is the most common location ofpolymorphous low grade


adenocarcinoma
a. Parotid glands
b. Submandibular glands
c. Submental glands
d. Palate

2. Which of the following is the most common location for minor salivary gland
tumors
a. Oral cavity and palate
3. What is the commonest salivary gland lesion in HIV patients
a. Epidermoid cyst
b. Lymphoepithelial cyst
c. Lymphoepithelial-like carcinoma
4. Which salivary gland tumor most commonly infiltrates nerves?
a. Adenoid cystic carcinoma
5. Which is the most common tumor occurring in the minor salivary glands and
the submandibular gland?
a. Adenoid cystic carcinoma
b. Mucoepidermoid carcinoma
c. Acinic cell carcinoma
6. Which tumor of the salivary gland does not have myoepithelial cell layer
a. Canalicular adenoma
b. PLGA

c. Basal cell adenoma


d. Adenoid cystic carcinoma
e. Pleomorphic adenoma
7. Most common malignant salivary gland tumor in children
a. Mucoepidermoid carcinoma
b. Acinic cell carcinoma
8. Which tumor, after mucoepidermoid carcinoma, is the 2nd most common
malignant salivary gland tumor in children?
a. Acinic cell carcinoma
b. Adenocarcinoma, NOS
c. Granular cell tumor
d. Metastatic thyroid carcinoma
e. Normal salivary gland
9. Which tumor will show strong, diffuse S100 and CD68 reactivity?
a. Acinic cell carcinoma
b. Adenocarcinoma, NOS
c. Granular cell tumor
d. Metastatic thyroid carcinoma
e. Normal salivary gland
10. As general rule acinic cell carcinoma ofthe major salivary glands has a better
prognosis than acinic cell carcinoma of the minor salivary glands
a. True
b. False
11. Which tumor is most prone to early recurrence and poor long term survival?
a. Carcinoma ex pleomorphic adenoma
12. Which is the most common tumor arising within the parapharyngeal space?
a. Pleomorphic adenoma
13. Which is the most common malignant tumor ofthe minor salivary glands?
a. Mucoepidermoid carcinoma

10. Liver and Gallbadder


1.

The hepatic cell responsible for most forms ofhepatic fibrosis is the
a. Ito (Stellate) cell
b. Hepatocyte
c. Kupffer cell
d. Portal tract fibroblast

2. What is the difference between alcoholic and non-alcoholic steato-hepatitis


a. Central vein obliteration
3. What is the hyaline globule found in alcoholic hepatitis
4. Which of the following results in ductopenia
a. Primary biliary cirrhosis
b. Extrabiliary atresia
c. Hemochromatosis
5. Sclerosing cholangitis is associated with which ofthe following tumors
6. Which of the following features favors hepatoblastoma over adenoma
7. Which of the hepatic lesion has no gender bias
a. HCC, fibrolamellar variant
b. FNH
c. Bile duct hamartoma
d. Hepatic adenoma
e. Regenerative nodular hyperplasia
8. Cholangiocarcinoma is seen in
a. Caroli disease
9. Which of the following shows ground glass nuclei
a. HBV

b. HCV
10. Obstruction ofvenous blood flow
a. Budd chiari syndrome
b. Crigler najjar syndrome
11. What is the most common cause ofextrahepatic biliary obstruction
a. Schistosomiasis
b. Criggler najjar syndrome
c. Budd chiari syndrome
12. All of the following are features ofextrahepatic obstruction EXCEPT
a. Bile duct proliferation
b. Bile duct with inspissated bile
c. Neutrophilic infiltration
d. Portal tract edema
e. Hepatocyte and canaliculi without bile
13. Nutmeg liver is associated with
a. Chronic passive congestion
b. Portal vein thrombosis
14. 3 year old boy has fever with sore throat, his mother gave him aspirin, the
child became comatose and his liver enzymes are elevated, what is the
diagnosis
a. Reye's syndrome
15. 30 year old male developed dementia, hemibalismus, and elevated liver
enzymes, CT scan showed liver cirrhosis, what other findings can be found
a. Antimitochondrial AB
b. Kyser Fleisher ring in the cornea
16. Which of the following stains can help distinguish Gucher cells from pseudogaucher cells
a. Alcian blue
b. PAS
c. Prussian blue
d. PAS-D

17. Which of the following is a risk factor for the development of hepatic
angiosarcoma
a. Vinyl chloride
b. Asbestos
18. Autosomal recessive disorder with no cysts grossly but hepatic microscopic
cysts identified
a. Congenital hepatic fibrosis
b. Caroli disease
c. Caroli syndrome
d. Choledocal cyst
19. Which lesion is a diffuse nodular transformation ofthe entire liver due to
heterogenous blood flow?
a. Focal nodular hyperplasia
b. Fibrolamellar HCC
c. Hepatic adenoma
d. Nodular regenerative hyperplasia
20. Which of the following hepatic tumor's prognosis depends on histology and
mitoses
a. Teratoma
b. Hepatoblastoma

c. Poorly differentiated rhabdomyosarcoma


d. Mesenchymal
21. Which is the most common virus to cause chronic hepatitis after an acute
hepatitis episode
a. HCV
b. HBV
c. HAV
d. HEV
e. EBV
22. Which of the following is true
a. The most common cause ofcryptogenic cirrhosis is NASH
23. What is true regarding primary sclerosing cholangitis
a. Associated with ulcerative colitis
24. What causes liver congestion with hepatic vein occlusion by collagen (venoocclusive disease)
25. Liver with central scar and proliferation ofarteries
a. Focal nodular hyperplasia
26. Patient with history ofCML who underwent bone marrow transplantation
presents with increased liver enzymes and obliteration ofhepatic vein radical
with centrilobular congestion

a. Portal vein thrombosis


27. OCP use is associated with
a. Hepatic adenoma
b. HCC
c. Bile duct adenoma
d. Focal nodular hyperplasia
28. Which liver cancer has the best prognosis
a. Fibrolamellar variant
29. Cirrhotic liver with copper deposition in periportal hepatocytes
a. Alcohol abuse
b. Hepatitis B
c. Hepatitis C
d. Bile duct obstruction
e. Alpha 1 antitrypsin deficiency
f.

Hemochromatosis

30. What is the commonest liver tumor


a. Cavernous hemangioma
31. Question about Criggler-Najjar syndrome
32. Which is the commonest form of gallstone?
33. What is the pathogenesis ofWilson disease
34. Male patient with ulcerative colitis. Radiology showed beading in the liver. The
biopsy of the liver will show which ofthe following
a. Onion skin fibrosis ofthe bile ducts
35. Female patient with high AMA and biopsy ofthe liver showing granulomas
a. Primary biliary cirrhosis
36. What would you see in a liver biopsy of a patient having constrictive
pericarditis
a. Sinusoidal dilation
b. Macro-regenerative nodules

c. Bile duct proliferation


37. Which is the most common to present with childhood jaundice past infancy
a. Bile duct hamartoma
b. Mesenchymal hamartoma
c. Choledochal cyst
d. Biliary atresia
38. All of the following can cause ductular reaction except
a. HCV
b. Massive necrosis
c. Bile duct obstruction
d. Chronic hepatitis B
e. Caroli disease or syndrome
39. GVHD ofliver, lymphocytes attack
a. Intrahepatic bile ducts
b. Veins
c. Hepatocytes
40. Cholangiocarcinoma is associated with
a. Primary sclerosing cholangitis
41. Bone marrow transplant patient now came with weight gain after 6 months.
Liver biopsy shows congested lobules with glycogen rich cells and obstructed

hepatic vein
a. Veno-occlusive disease
b. Hepatic vein thrombosis
42. Hepatic tumor which presents with heart failure
a. Infantile hemangioendothelioma
b. Hepatoblastoma
c. Rhabdomyosarcoma
d. Undifferentiated sarcoma
43. Biliary atresia differs from congenital hepatic fibrosis by
a. Ductular proliferation
b. Duct plate malformation
44. The 3 characteristic histologic features of fibrolamellar HCC are
a. Eosinophilic granular hepatocytes, prominent hepatocyte nucleoli and
lamellar fibrosis
b. Eosinophilic granular hepatocytes, lamellary fibrosis and abundant mitoses
c. Lamellar fibrosis, pseudoglandular formation and eosinophilic granular
hepatocytes
d. Mucin production, lamellar fibrosis and pseudoglandular formation
e. Tumor necrosis, prominent hepatocyte nucleoli and lamellar fibrosis
45. Local regional metastasis, local invasion and regional LN metstasis are
contraindications to liver resection in fibrolamellar HCC
a. True
b. False
46. The presence of a centraql scar in fibrolamellar carcinoma seen by CT will
raise the possibility ofwhich other entity
a. Focal nodular hyperplasia
b. Metastatic adenocarcinoma
c. Metastatic melanoma
d. Scirrhous variant of conventional HCC
47. Which of the following best describes angiosarcoma
a. Multiple tumor nodules alternating with blood filled cavities
b. Spindle cell tumor cells with intracytoplasmic capillary lumens
c. Trabecular arrangements of malignant endothelial cells

d. Tumor nodules composed ofmalignant spindled cells associated with a


fibrotic stroma
48. Which entity has been associated with arsenic exposure
a. Angiosarcoma
b. Epithelioid hemangioendothelioma
c. Metastatic sarcoma
d. Poorly differentiated HCC
49. All of the following statements are associated with epithelioid
hemangioendothelioma ofthe liver except
a. EHE is typically associated with bilateral hepatic lobe involvement
b. EHE may express both CD31 and keratin
c. Females are more frequently affected by EHE as compared to males
d. Surgical resection is not effective treatment for EHE
50. Which of the following features favors mesenchymal hamartoma over infantile
hemangioma
a. Branching bile duct structures
b. Cavernous vascular channels
c. Entrapped hepatocytes and bile ducts
d. Fibromyxoid stroma
e. Immunoreactivity for CD31
51. Which of the following is correct about undifferentiated embryonal sarcoma?
a. Cytogenetic changes similar to mesenchymal hamartoma
b. Diffuse expression of desmin and myogenin on immunohistochemistry

c. Mild nuclear pleomorphism


d. PASpositive diastase sensitive globules in cytoplasm and stroma
e. Peak incidence in 0 3 year age group
52. Which of the following features favor mesenchymal hamartoma over mixed
epithelial-mesenchymal subtype of hepatoblastoma
a. Absence ofbile duct elements
b. Glandular elements with mucinous epithelium
c. Markedly elevated AFP
d. Normal appearing hepatocytes
e. Osteoid formation in stroma
53. The presence of periportal PASpositive intracytoplaasmic globules with PIZZ
present on isoelectric focusing is diagnostic for
a. Alpha 1 antitrypsin deficiency
b. Wilson disease
c. Hepatitis C
d. Primary biliary cirrhosis
54. The most common cause ofPAS positive intracytoplasmic globules adjacent to
the central vein is
a. Hepatocellular carcinoma
b. Centrilobular congestion
c. Alpha 1 antitrypsin deficiency
d. Hepatitis C
55. There is a high prevalence of what viral infection associated with alpha 1
antitrypsin deficiency
a. Epstein-barr
b. Hepatitis B
c. Hepatitis E
d. Hepatitis A
56. Which of the following best describes focal nodular hyperplasia of the liver?
a. Multiple hepatic nodules ranging from 1 to mm in size
b. Large nodule 1-3 cm in size composed ofclonal hepatocytes without
associated portal tracts

c. Well-demarcated nodule near the liver capsule with a central fibrosed


region
d. Well-circumscribed nodule measuring up to 3 cm in a background of
cirrhosis
57. Which entity has been associated with a risk of malignant transformation
a. Hepatic adenoma
b. Focal nodular hyperplasia
c. Nodular regenerative hyperplasia
d. Large benign regenerative nodule
58. Which entity is typically associated with oral contraceptive use?
a. Hepatic adenoma
b. Nodular regenerative hyperplasia
c. Focal nodular hyperplasia
d. Large regenerative nodule
59. Which of the following markers is useful in identifying Mallory-Denk hyaline?
a. CK 8/18
b. Congo red
c. Inhibin
d. Mucicarmine
e. P53
f.

Trichrome

60. Which of the following drugs is most likely to be a cause ofphospholipidosis?


a. Acetaminophen
b. Amiodarone
c. Metronidazole

d. Nicotine
e. NSAIDS
61. Which of the following findings can be seen in alcoholic liver disease but not in
non-alcoholic fatty liver disease?
a. Central vein obliteration
b. Cirrhosis
c. Mallory-Denk hyaline
d. Panlobular macrovesicular steatosis
e. Periportal fibrosis
62. Which of the following statements is correct about hepatoblastoma
a. Crowded fetal pattern typically shows mitoses > =20/HPF
b. Macrotrabecular patterntypically shows 3-5 cells thick trabeculae
c. Immature fibrous tissue, cartilage and osteoid are the most common
mesenchymal elements in the mixed epithelial mesenchymal subtype
d. Diffuse glutamine synthetase on immunohistochemistry distinguishes it
from HCC
e. Absence ofglypican 3 on immunohistochemistry distinguishesit from HCC
63. Which of the following argues against the diagnosis of undifferentiated
embryonal sarcoma
a. Occurrence in 6 10 year age group
b. Marked nuclear pleomorphism with rhabdomyoblast-like
c. Entrapped hepatocytes and bile ductules at periphery
d. PAS-diastase positive globules in cytoplasm and stroma
e. Diffuse expression of myogenin by immunohistochemistry
64. Which of the following favors hepatoblastoma over hepatocellular adenoma
a. Age above 10 years
b. Glycogen storage disease
c. Elevated serum AFP

d. Androgen therapy for Fanconi anemia


e. Large tumor cells resembling mature hepatocytes
65. Which of the following features can be helpful in distinguishing cytoplasmic
inclusions in GSD IV and Lafora disease
a. Inclusions ofLafora disease are pale staining ofbasophilic
b. Inclusions ofLafora disease are digested with diastase
c. Inclusions ofGSD IV area coarsely stained with colloid iron
d. Inclusions ofGSD IV resist digestion with pectinase
66. Which of the following conditions is not associated with swollen hepatocytes
loaded with glycogen
a. Urea cycle enzyme defects
b. Fibrinogen storage disease
c. Glycogenic hepatopathy
d. High dose steroids
67. Which of the following statements is correct about glycogen storage diseases
a. Swollen hepatocytes are common in GSD, type 0
b. Adenomas occur in GSD, type I
c. Lysosomal glycogen in seen in GSD, type III
d. Ground glass inclusions are seen in GSD, type IX
68. Which of the following is true about the use ofHep Par 1 in the diagnosis of
hepatocellular carcinoma (HCC)?
a. Not useful in distinction from renal cell carcinoma
b. Sensitivity is high (> 80%) in poorly differentiated HCC
c. Sensitivity is low (< 50%) in clear cell HCC
d. Strong expression can occur in metastatic adenocarcinoma from lung and
stomach
69. Which of the following immunohistochemical results favors renal cell carcinoma
metastatic to liver over hepatocellular carcinoma?
a. CK 7 and 20 ve
b. Hep Par 1 + ve
c. Moc-31 ve
d. PAX-2 + ve
70. Which of the following is true about making an immunohistochemical diagnosis

ofHCC?
a. AFP has high sensitivity but low specificity
b. Cytoplasmic reactivity with polyclonal CEA is specific for hepatocellular
differentiation
c. Glypican-3 has lower sensitivity for poorly differentiated HCC than Hep Par
-1
d. MOC-31 expression is seen in a small minority of HCC
71. Which of the following is the most common form ofhereditary liver disease in
adults?
a. Alpha-1 antitrypsin deficiency
b. Glycogen storage disease type IV
c. Hereditary hemochromatosis
d. Hereditary hypofibrinogenemia
e. Wilson disease
72. Which of the following USabnormalities is characteristic of alpha-1 antitrypsin
deficiency?
a. Dilated endoplasmic reticulum containing amorphous material

b. Electron-dense cytoplasmic material with a "starburst" pattern


c. Filamentous viral particles
d. Lysosomal storage disease
73. Which of the following therapeutic options is most commonly used for treating
liver disease associated with alpha-1 antitrypsin deficiency?
a. Anti-viral medication
b. Copper supplementation
c. Enzyme replacement therapy
d. Iron chelation
e. Liver transplantation
74. Which of the following is not true about acute liver failure due to herpes
simplex virus hepatitis?
a. Can occur in immunocompetent adults
b. Rapidly progressive course
c. Marked lymphoplasmacytic inflammation
d. Extensive necrosis
75. Which of the following statements about acetaminophen toxicity is not correct?
a. It is the most common cause offulminant hepatic failure in the US
b. Chronic alcohol use can enhance hepatotoxicity of acetaminophen
c. Necrosis most often occurs in non-zonal distribution
d. Histological distinction from ischemic necrosis can be difficult

76. All the following are true about Wilson disease except
a. Presentation after 50 years ofage is uncommon
b. 24 hour urinary copper is markedly elevated in fulminant disease
c. Normal ceruloplasmin level rules out the diagnosis
d. Quamtitative copper level from the paraffin block is the most accurate
method ofassessing hepatic copper
77. Which lesion is the most common benign tumor of the liver?
a. Angiosarcoma
b. Cavernous hemangioma
c. Epithelioid hemangioendothelioma
d. Focal nodular hyperplasia
e. Peliosis hepatis
78. Which lesion is characterized by tumor cells with an epithelioid appearance and
cytoplasmic vacuoles that may contain RBCs?
a. Angiosarcoma
b. Cavernous hemangioma
c. Epithelioid hemangioendothelioma
d. Focal nodular hyperplasia
e. Peliosis hepatis
79. Which lesion is charactgerized by the presence ofa central scar in which
malformed arteries are found?
a. Angiosarcoma
b. Cavernous hemangioma
c. Epithelioid hemangioendothelioma
d. Focal nodular hyperplasia
e. Peliosis hepatis
80. Immunohistochemical workup of this case using CK stains only may lead to a
misdiagnosis

a. True
b. False
81. Angiosarcoma is associated with environmental carcinogens such as thorotrast,
vinyl chloride, and arsenic compounds in > 95% ofcases
a. True
b. False
82. Nuclear atypia, capsule penetration, necrosis, and the number ofmitoses,
predict prognosis in epithelioid hemangioendothelioma
a. True
b. False
83. Which non-cirrhotic condition may be associated with portal hypertension and
is often seen in patients with hematologic and immunologic disorders
a. Focal nodular hyperplasia
b. Hepatic adenoma
c. Hepatocellular carcinoma, fibrolamellar type
d. Macroregenerative nodule
e. Nodular regenerative hyperplasia
84. Which tumor is usually seen in young in young non-cirrhotic patients contains
cytoplasmic pale bodies and has aggressive behavior but carries a more
favourable prognosis than its related variants occurring in cirrhotic liver?
a. Focal nodular hyperplasia
b. Hepatic adenoma
c. HCC, fibrolamellar type
d. Macroregenerative nodule
e. Nodular regenerative hyperplasia
85. Which lesion most often occurs in young women taking oral contraceptive
agents and when large carries a risk oflife-threatening hemoperitoneum?
a. Focal nodular hyperplasia
b. Hepatic adenoma

c. HCC, fibrolamellar type


d. Macroregenerative nodule
e. Nodular regenerative hyperplasia
86. Which of the following is most likely to show false negative staining for mast
cells?
a. Alcian blue staining performed on an air-dried touch prep
b. Giemsa staining performed on formalin fixed paraffin embedded tissue
c. Immunohistochemical staining for human tryptase performed on an airdried touch prep
d. Toluidine blue staining performed on an alcohol fixed touch prep
87. Which of the following is not considered a diagnostic criterion ofsystemic
mastocytosis?
a. Anatomical distribution of mast cell aggregates
b. Immunohistochemical staining ofmast cells
c. Morphologic features of mast cells
d. Presence ofadditional clonal hematopoietic disorder/malignancy
e. Serum total tryptase level
88. Which is the cause of non-cirrhotic portal hypertension?
a. Hepatic veno-occlusive disease
b. Hereditary hemochromatosis
c. Sarcoidosis
d. Schistosomiasis
e. Systemic mastocytosis
89. Hepatocellular lesion in which cytoplasmic pale bodies are most frequently
identified.

a. Fibrolamellar HCC
b. Focal nodular hyperplasia
c. HCC, well-differentiated
d. Oncocytic carcinoid tumor
e. Scirrhous HCC
90. Hepatocellular lesion that is most frequently associated with a centeral scar
containing large vessels.
a. Fibrolamellar HCC
b. Focal nodular hyperplasia
c. HCC, well-differentiated
d. Oncocytic carcinoid tumor
e. Scirrhous HCC
91. Hepatocellular lesion that lacks a fibrous stromal component.
a. Fibrolamellar HCC
b. Focal nodular hyperplasia
c. HCC, well-differentiated
d. Oncocytic carcinoid tumor
e. Scirrhous HCC
92. Which is least likely to be associated with Mallory hyaline?
a. Cirrhosis due to autoimmune hepatitis
93. Which can be regarded as a model ofprotein conformational disease?
a. Cirrhosis due to alpha-1 antitrypsin deficiency
94. Which is related to mutations of the SERPINA gene on chromosome 14?
a. Cirrhosis due to alpha-1 antitrypsin deficiency
95. Which condition does not feature perisinusoidal connective tissue deposition?
a. Epithelioid hemangioendothelioma
96. Which non-neoplastic entity typically involves the entire hepatic acinus (ie
nonzonal)?
a. Amyloidosis
97. Which entity is not associated with portal hypertension?
a. Amyloidosis

b. Cirrhosis due to steatohepatitis


c. Epithelioid hemangioendothelioma
d. Venous outflow obstruction with extensive fibrosis (cardiac cirrhosis)
98. Which of the following is not true about herpes simplex virus hepatits?
a. Can affect immunocompetent adults
b. Often follows a rapidly progressive course
c. Histological findings are often indistinguishable from adenoviral hepatitis
d. Marked lymphoplasmacytic inflammation is common
e. Necrosis is often extensive and non-zonal
99. Which of the following statements about acetaminophen toxicity is not correct?
a. It is the most common cause offulminant hepatic failure in the US
b. Chronic alcohol use can enhance hepatotoxicity of acetaminophen
c. Necrosis most often occurs in a non-zonal distribution
d. Histological distinctionfrom ischemic necrosis can be difficult
e. Sinusoidal dilatation and congestion can be seen in the centrizonal areas
100.

All ofthe following are true about Wilson disease except?

a. Presentation after 50 years ofage is uncommon


b. 24 hour urinary copper is markedly elevated in fulminant disease
c. Normal ceruloplasmin levels can be seen in around 10% ofcases
d. Absence ofcopper on histochemical stains rules out Wilson disease
e. Quantitative copper levels from the paraffin block is the most accurate
method ofassessing hepatic copper

101. Which is almost invariably associated with autosomal dominant polycystic


kidney disease?
a. Polycystic liver disease
102.

Which shows cystic dilation ofinterlobar and segmental bile ducts,

often presents with recurrent cholangitis and microscopically does not show
evidemce ofcongenital hepatic fibrosis
a. Caroli disease
103.

Which lesion most commonly consists of a solitary tumor due to

aberrant blood flow?


a. Focal nodular hyperplasia
104.

Which condition is most likely to present as bleeding from esophageal

varices?
a. Congenital hepatic fibrosis
105.

Which condition may be associated with small scars that resemble

corpora albicantia?
a. Congenital cystic (polycystic) disease ofthe liver
106.

In which malignant hepatic tumor may mitoses and histology be

prognostic determinants?
a. Hepatoblastoma
107.

Which hepatic mass may be the cause ofheart failure?

a. Mesenchymal hamartoma
108.

Which hepatic mass arising in the biliary tree most typically causes

obstruction ofbile?
a. Rhabdomyosarcoma
109.

A CK7 -, CK20+ gland forming cancer involving the liver favors which

ofthe following diagnoses?


a. Colorectal adenocarcinoma, metastatic

110. Which of the following is considered to be a risk factor for gallbladder


carcinoma?
a. Salmonella typhi carrier state
111. Which of the following conditions is most strongly associated with gallbladder
adenocarcinoma?
a. Porcelain gallbladder
112. Which of these lesions is benign demonstrates ovarian-like stroma and occurs
almost exclusively in females?
a. Hepatobiliary cystadenoma
113. Which of these lesions is thought to arise from the embryonic foregut and
shows differentiation towards bronchial structures?
a. Ciliated forgut structure
114. Which of these lesions is commonly associated with inflammation and biliary
calculi?
a. Caroli disease
115. In which tumor are pale bodies frequently identified within the cytoplasm of
the hepatocyte?
a. Fibrolamellar hepatocellular carcinoma
116. Which lesion is most likely to be associated with cavernous hemangiomas of
the liver?
a. Focal nodular hyperplasia

117. In which lesion does a solitary central artery with high flow and absent portal
vein account for its characteristic radiologic and pathologic appearance?
a. Focal nodular hyperplasia
118. Which hepatic neoplasm most commonly arises within large bile ducts?
a. Embryonal RMS
119. Which liver tumor occurs in the 5 20 year old age group contains
pleomorphic spindle cells displays eosinophillic inclusions and has a
characteristic myxoid background?
a. Embryonal (undifferentiated) sarcoma
120.

Which entity usually shows negative viral serology with positive

antinuclear antibodies (ANA) and moderately elevated ALT without evidence of


significant biliary damage?
a. Autoimmune hepatitis
121. Which entity usually shows negative viral serology with positive
antimitochondrial antibody (AMA) mildly elevated ALT and significant evidence
ofbiliary damage?
a. Primary biliary cirrhosis
122.

Which entity is usually diagnosed by the radiographic demonstration of

beaded bile ducts in the absence ofother disease?


a. Primary sclerosing cholangitis
123.

Chronic rejection of the liver typically manifests as

a. Loss ofsmall bile ducts with an obliterative arteriopathy


124.

Acute rejection of the liver may be distinguished from chronic hepatits

by
a. The presence of endotheliitis
b. The presence of mixed portal inflammation (lymphocytes, plasma cells,
eosinophils) as opposed to a purely lymphocytic infiltrate
c. The lack of piecemeal necrosis
d. Extensive duct damage
e. All of the above
125.

How long after liver transplantation does chronic rejection typically

occur?
a. Within one year
126.

Which ofthe following hereditary diseases affecting the liver may

present in adulthood?
a. Alpha-1 antitrypsin deficiency
b. Wilson's disease
c. Hemochromatosis
d. Hereditary amyloidosis
e. All of the above
127.

While hepatic involvement in most forms ofsystemic amyloidosis is

common clinical evidence ofliver dysfunction is usually not significant


a. True
b. False
128.

Which is a congenital liver abnormality that involves both lobes and is

associated with bile duct stones, cholangitis and renal cysts?


a. Caroli's disease
129.

Which liver abnormality shows preferential involvement of the left lobe,

intrahepatic bile duct stones and cholangitis?


a. Recurrent pyogenic cholangitis (oriental cholangiohepatitis)
130.

Which liver abnormality most often arises in patients with inflammatory

bowel disease?
a. Primary sclerosing cholangitis

131. Which is the most common malignant tumor ofthe gallbladder?


a. Well-moderately differentiated adenocarcinoma
132.

All ofthe following are associated with an increased risk ofgallbladder

carcinoma except
a. Cholelithiasis
b. Adenomatous pancreaticobiliary duct union
c. GERD
d. Sclerosing cholangitis
e. Native American heritage
133.

In which hepatic lesion is elevation ofserum alpha fetoprotein most

common?
a. HCC, NOS
134.

Which hepatic lesion is most strongly associated with cirrhosis?

a. HCC, NOS
135.

Alpha 1 antitrypsin deficiency is often associated with hepatic

ductopenia
a. True
b. False
136.

Histologic findings on liver biopsy will differentiate the underlying

etiology in the majority ofcases ofchronic biliary disease.


a. True
b. False
137.

Most patients with cystic fibrosis will develop chronic biliary disease

a. True
b. False
138.

Ductal plate malformation is a bile duct abnormality which is a feature

of
a. Alagille syndrome
b. Congenital hepatic fibrosis
c. Cystic fibrosis
d. Extrahepatic biliary atresia
e. All of the above

139.

Hepatorenal fibrocystic diseases include all ofthe following except

a. Alagille syndrome
b. Autosomal dominant polycystic kidney disease
c. Autosomal recessive polycystic kidney disease
d. Meckel syndrome
e. Nephronophthisis
140.

A typical feature ofextrahepatic biliary atresia which is helpful in

distinguishing it from congenital hepatic fibrosis is


a. Association with ciliopathies
b. Reactive ductular proliferation
c. Ductal plate malformation
d. Hepatocellular swelling and giant cell transformation
e. PKHD1 gene mutation
141. Which entity is characterized by hepatosplenomegaly neurological dysfunction
skeletal disorders and enlarged macrophages infiltrating the liver spleen brain
and bone marrow which have a fibrillary tissue paper appearance to the
cytoplasm?
a. Gaucher disease
b. Malakoplakia
c. Niemann-Pick disease
d. Rosai Dorfman disease
e. Tay-Sachs disease
142.

Which entity is diagnosed using a biochemical test for sphingomyelinase

activity?
a. Gaucher disease
b. Malakoplakia
c. Niemann-Pick disease
d. Rosai-Dorfman disease
e. Tay-Sachs disease
143.

Which entity is primarily seen in middle aged women and is

characterized by a positive antimitochondrial antibody, elevated alkaline

phosphatase and elevated IgM?


a. Primary biliary cirrhosis
144.

Which entity is associated with endoplasmic reticulum stress secondary

to accumulation of abnormally folded protein?


a. Alpha-1 antitrypsin deficiency
145.

Which ofthe following features is more characteristic offibrolamellar

carcinoma compared to conventional hepatocellular carcinoma?


a. Pale bodies
146.

Which ofthe following correctly describes the clinical impact of

distinguishing fibrolamellar carcinoma and conventional hepatocellular


carcinoma?
a. More aggressive approach in lymph nodes dissection in fibrolamellar
carcinoma
147.

Which ofthe following argues against a diagnosis of fibrolamellar

carcinoma?
a. Diffuse chromogranin expression
148.

Which entity is characterized by isolated increased lipid uptake/storage

in macrophages in the lamina propria ofthe gallbladder?

a. Cholesterolosis
149.

Which entity is characterized by the accumulation ofglucocerebroside

in cells including tissue macrophages, 2ry to a deficiency ofB-glucosidase?


a. Gaucher disease
150.

Which condition is characterized by accumulation of lipofuscin (ceroid)

granules within macrophages?


a. Xanthogranulomatous cholecystits
151. Which entity is characterized by a mutation in the ATP7B protein encoded for
on chromosome 13?
a. Wilson disease
152.

Which ofthe following best describes undifferentiated embryonal

sarcoma ofliver?
a. Multiple tumor nodules formed by small undifferentiated cells.
b. Primitive small tumor cells, resembling fetal liver.
c. Polymorphus spindle, ovoid, and anaplastic tumor cells with
intracytoplasmic globules.
d. Tumor nodules composed ofmalignant spindled cells associated with
cytoplasmic vacuoles

153.

Which entity shares 19q abnormalities with some cases ofembryonal

sarcoma, suggesting a possible shared pathway?


a. Bile duct hamartoma
b.

Hepatobiliary rhabdomyosarcoma

c.

Hepatoblastoma

d.

Mesenchymal hamartoma

154.

Positive staining for which IHC markers can be seen in undifferentiated

embryonal sarcoma of the liver(UESL)?


a. Cytokeratin

b. Desmin
c. Vimentin
d. All of the above
e. None of the above

11.
1.

Pancreas

Pancreatic multicystic tumor with ovarian stroma in the cyst wall


a. Mucinous cystic neoplasm

2. Which of the following pancreatic lesion is associated with VHL


a. Pancreatic serous cystadenoma
b. Mucinous cystadenoma
3. Pancreatic osteoclastic giant cell tumor
a. CD68 positive, and CK negative
b. CD68 positive, CK positive
c. CD68 negative, CK negative
d. CD68 positive and EMA positive
4. Which pancreatic tumor is CK negative
a. Solid pseudopapilallry tumor
b. Pancreatoblastoma
c. Acinic cell carcinoma
d. Endocrine tumor of pancreas
e. Intraductal papillary mucinous cystadenoma
5. Patient with confusion and stupor precipitated by fasting and relieved by food
most likely has
a. Insulinoma
6. A female with chronic painful diarrhea, recurrent duodenal ulcers, and
increased gastic acid output probably has
7. Question about chronic pancreatitis

8. Intraductal pancreatic mucinous neoplasm


a. Occurs in the main pancreatic duct
b. More common in females than males
c. More often multifocal
d. Has ovarian-like stroma
9. Which of the following pancreatic tumors has mutations in k-ras, CDKN2a and
loss of DPC4
a. Pancreatic adenocarcinoma (NOS)
10. Which of the following is the pathogenesis ofDM II
a. Increase ofthe peripheral resistence of insulin
b. Loss ofbeta cells
c. Chronic renal failure
d. Amyloid deposition
11. Which of the following is the pathogenesis ofDM I
a. Loss ofislet cells
12. A large for gestational age baby presents with hyperglycemia and is born of a
diabetic mother, which is he most likely to have
a. Nesidioblastosis
13. All of the following are prognostic factors for pancreatic endocrine neoplasms
except
a. Invasion ofadjacent peripancreatic tissue, age of the patient, and extent of
the disease
b. Vimentin
c. Synaptophysin
d. GFAP
14. A 60 year old patient dies, his pancreas shows atrophy ofacini, fibrosis, spacing
ofislet cells, and secretions in ducts. What's the possible cause

a. Alcohol
b. Cholelithiasis
15. The histological difference in the pancreas oftype I and typeIIDM
a. In type I DM, there is no amyloid deposition
16. Which tumor is commonly associated with mutations in exon 3 ofthe betacatenin gene?
a. Acinar cell carcinoma
b. Ductal adenocarcinoma
c. Pancreatic pseudocyst
d. Pancreatoblastoma
e. Solid-pseudopapillary neoplasm
f.

Neuroendocrine tumor

17. Which tumor typically occurs in patients under 10 years ofage and displays
squamoid corpuscles on microscopy?
a. Acinar cell carcinoma
b. Ductal adenocarcinoma
c. Pancreatic pseudocyst
d. Pancreatoblastoma
e. Solid-pseudopapillary neoplasm
f.

Neuroendocrine tumor

18. Which tumor stains strongly for chromogranin and displays a membranous
staining pattern with CD99?
a. Acinar cell carcinoma
b. Ductal adenocarcinoma
c. Pancreatic pseudocyst
d. Pancreatoblastoma
e. Solid-pseudopapillary neoplasm
f.

Neuroendocrine tumor

19. Which of the following is correct about PanNET

a. Cytologically bland tumors may show lymph node metastasis


b. LG and HG NE carcinoma are distinguished by size
c. Most insulinoma are metastatic at presentation
d. Most tumors larger than 2 cm are malignant
20. Which immunostain panel is useful in diagnosing solid pseudopapillary tumor
a. Amylase, lipase, trypsin
b. B-catenin, E-cadherin, PR
c. ER, somatostatin, CD10
d. Keratin, chromogranin, Mib1
21. Pancreatoblastoma is exclusively seen in children and thus would not have to
be considered in the differential diagnosis ofthis case
a. True
b. False
22. Which of the following entities is most commonly seen in young children?
a. Acinar cell carcinoma
b. Pancreatic endocrine tumor
c. Pancreatoblastoma
d. Solid pseudopapillary neoplasm
e. Well-differentiated ductal adenocarcinoma
23. Which entity is rare in men?
a. Acinar cell carcinoma
b. Pancreatic endocrine tumor
c. Pancreatoblastoma
d. Solid pseudopapillary neoplasm
e. Well-differentiated ductal adenocarcinoma
24. Which adult tumor is immunohistochemically positive for pancreatic enzymes
trypsin and chymotrypsin?
a. Acinar cell carcinoma
b. Pancreatic endocrine tumor
c. Pancreatoblastoma
d. Solid pseudopapillary neoplasm
e. Well-differentiated ductal adenocarcinoma
25. Which entity is characterized by multiple cysts lined by low-cuboidal epithelial

cells with clear cytoplasm and can be seen in association with the VHL
syndrome
a. Intraductal papillary mucinous neoplasm
b. Lymphoepithelial cyst
c. Metastatic renal cells carcinoma
d. Microcystic serous cystadenoma
e. Mucinous cystic neoplasm
f.

Pancreatic ductal adenocarcinoma

26. Which entity is almost entirely seen in women and is characterized by cystic
spaces lined by mucinous cells with a surrounding ovarian type stroma

a. Intraductal papillary mucinous neoplasm


b. Lymphoepithelial cyst
c. Metastatic renal cell carcinoma
d. Microcystic serous cystadenoma
e. Mucinous cystic neoplasm,
f.

Pancreatic ductal adenocarcinoma

27. Which cystic entity is often filled with keratinaceous debris


a. Intraductal papillary mucinous neoplasm
b. Lymphoepithelial cyst
c. Metastatic renal cell carcinoma
d. Microcystic serous cystadenoma
e. Mucinous cystic neoplasm
f.

Pancreatic ductal adenocarcinoma

28. Which of the entities is most commonly seen in children?


a. Acinar cell carcinoma
b. Pancreatic endocrine tumor
c. Pancreatoblastoma
d. Solid pseudopapillary neoplasm ofthe pancreas
e. Well-differentiated ductal adenocarcinoma
29. Which entity is more common in women?
a. Acinar cell carcinoma
b. Pancreatic endocrine tumor
c. Pancreatoblastoma

d. Solid pseudopapillary neoplasm ofthe pancreas


e. Well-differentiated ductal adenocarcinoma
30. Which adult tumor is immunohistochemically positive for pancreatic enzymes
trypsin and chymotrypsin?
a. Acinar cell carcinoma
b. Pancreatic endocrine tumor
c. Pancreatoblastoma
d. Solid pseudopapillary neoplasm ofthe pancreas
e. Well-differentiated ductal adenocarcinoma
31. This asymptomatic tumor frequently occurs as a pancreatic tail mass in young
women
a. Acinar cell carcinoma
b. Pancreatic endocrine neoplasm
c. Pancreatoblastoma
d. Poorly differentiated pancreatic adenocarcinoma
e. Solid pseudopapillary neoplasm
32. CD56 is often considered a neuroendocrine marker. In addition to pancreatic
endocrine tumor, this tumor frequently shows strong CD56 immunoreactivity.
a. Acinar cell carcinoma
b. Pancreatic endocrine neoplasm
c. Pancreatoblastoma
d. Poorly differentiated pancreatic adenocarcinoma
e. Solid pseudopapillary neoplasm

33. The squamoid corpuscle is a characteristic feature ofthis tumor.


a. Acinar cell carcinoma
b. Pancreatic endocrine neoplasm
c. Pancreatoblastoma
d. Poorly differentiated pancreatic adenocarcinoma
e. Solid pseudopapillary neoplasm
34. Which of the tumors occurs almost exclusively in women and is notable for a
peri-cystic stromal proliferation termed ovarian type stroma
a. Intraductal papillary mucinous carcinoma, non-invasive
b. Mucinous carcinoma arising in an intraductal papillary mucinous neoplasm
c. Mucinous cystic neoplasm
d. Pancreatic ductal adenocarcinoma, usual type
e. Serous microcystic adenoma
35. This tumor is associated with mutations in Kras, p16, TP53, and loss ofDCP4
a. Intraductal papillary mucinous carcinoma, non-invasive
b. Mucinous carcinoma arising in an intraductal papillary mucinous neoplasm
c. Mucinous cystic neoplasm
d. Pancreatic ductal adenocarcinoma, usual type
e. Serous microcystic adenoma
36. In the setting ofIPMN with invasive carcinoma, mucinous differentiation is
associated with a better prognosis
a. True
b. False
37. Which immunostains would be most helpful in the differential diagnosis of
lymphoepithelail cyst of the pancreas and pancreatic lymphangioma
a. CD3, CD20, and factor VIII
b. CD45, CD68, and factor VIII
c. Factor VIIIand HMB45
d. Factor VIIIand CD3
e. Factor VIII, CD31, and CK
38. Which of the following statements regarding pancreatic cysts is correct
a. The epithelium in dermoid cyst of the pancreas never shows evidence of
mucinous or sebaceous differentiation

b. The epithelium in mucinous cystic neoplasm and intraductal pancreatic


mucinous tumor are typically ofthe transitional type
c. Like lymphoepithelail cyst ofthe salivary gland, lymphoepithelail cyst of the
pancreas is associated with HOV disease
d. Pancreatic pseudocyst is devoid ofan epithelial lining
39. In lymphoepithelial cyst ofthe pancrease, all ofthe following may be seen
except
a. Clusters offoamy macrophages
b. Extensive sebaceous differentiation
c. Foci of fat necrosis
d. Keratin granulomas
e. Lymphoepithelial nests in the lymphoid tissue
40. Which multicystic tumor is positive for CD31?
a. Lymphangioma
b. Metastatic renal cell carcinoma
c. Microcystic serous cystadenoma
d. Mucinous cystic neoplasm
e. Serous cystadenocarcinoma
41. Which tumor is associated with von-Hippel-Lindau syndrome, can have VHL
gene mutations, and is a primary pancreatic neoplasm?
a. Lymphangioma
b. Metastatic renal cell carcinoma
c. Microcystic serous cystadenoma
d. Mucinous cystic neoplasm
e. Serous cystadenocarcinoma
42. Which multicystic tumor contains ovarian type stroma in the cyst walls?
a. Lymphangioma
b. Metastatic renal cell carcinoma
c. Microcystic serous cystadenoma
d. Mucinous cystic neoplasm

e. Serous cystadenocarcinoma
43. Which is not a neoplasm?
a. Cystic islet cell neoplasm
b. Intraductal papillary mucinous neoplasm
c. Mucinous cystic neoplasm (mucinous cystadenoma)
d. Pancreatic pseudocyst
e. Serous cystadenoma
f.

Solid pseudopapillary neoplasm ofthe pancreas

44. Which enitity features ovarian type spindle cell stroma which is often positive
for estrogen and progesterone receptors?
a. Cystic islet cell neoplasm
b. Intraductal papillary mucinous neoplasm
c. Mucinous cystic neoplasm
d. Pancreatic pseudocyst
e. Serous cystadenoma
f.

Solid pseudopapillary neoplasm ofthe pancreas

45. Frozen section analysis ofthe pancreatic duct margin is most important for
which entity?
a. Cystic islet cell neoplasm

b. Intraductal papillary mucinous neoplasm


c. Mucinous cystic neoplasm
d. Pancreatic pseudocyst
e. Serous cystadenoma
f.

Solid pseudopapillary neoplasm ofthe pancreas

46. Which lesion is thought to arise from the pancreatic duct system and is often
multifocal?
a. Intraductal papillary mucinous tumor
b. Mucinous cystadenocarcinoma
c. Pancreatic acinar cell carcinoma
d. Pancreatic ductal adenocarcinoma
e. Pancreatic endocrine carcinoma
f.

Pancreatoblastoma

g. Solid pseudopapillary tumor


47. Which lesion occurs in young women and may have a "pseudo" cystic
appearance on gross examination?
a. Intraductal papillary mucinous tumor
b. Mucinous cystadenocarcinoma
c. Pancreatic acinar cell carcinoma
d. Pancreatic ductal carcinoma
e. Pancreatic endocrine carcinoma
f.

Pancreatoblastoma

g. Solid pseudopapillary tumor


48. The grading of pancreatic endocrine tumor is based on hidtologic features
alone
a. True
b. False
49. Which cystic tumor commonly extends along ductal structures within the
pancreas?
a. Intraductal papillary mucinous neoplasm
50. Which tumor shows proliferation ofdysplastic epithelium with excess mucin
production involving the main pancreatic duct, associated with chronic

pancreatitis
a. Intraductal papillary mucinous tumor
51. Which is an agrressive tumor typically consists ofinfiltrating glands embedded
in dense desmoplastic stroma forming an ill-defined mass in the pancreatic
head
a. Ductal adenocarcinoma
52. Which tumor is associated with both clinical and immunohistochemical features
ofpancreatic enzyme secretion?
a. Acinar cell carcinoma
53. Which tumor is a low grade malignant neoplasm most likely to be mistaken for
a pancreatic pseudocyst?
a. Solid-pseudopapillary tumor
54. Which lesion is most commonly seen in young children and characterized by
squamoid corpuscles?
a. Pancreatoblastoma
55. Which lesion is most commonly associated with subcutaneous fat necrosis?
a. Acinar cell carcinoma
56. Which lesion is seen most often in patients with Zollinger Ellison syndrome?
a. Pancreatic endocrine tumor
57. Which is characterized histologically by PAS positive diastase resistant
cytoplasmic granules?
a. Acinar cell carcinoma

58. Which of the following is correct about well-differentiated pancreatic


endocrine tumors?
a. Most insulinomas are malignant
b. The distinction between low and high grade neuroendocrine carcinoma is
based on vascular and perineural invasion
c. Tumors lacking nuclear atypia can show lymph node metastasis
d. Tumors larger than 2 cm are considered malignant
59. Which of the following immunohistochemical staining results is characteristic of
solid pesudopapillary tumor?
a. Nuclear staining with PR
60. Which of the following immunohistochemical staining results is correct about
the diagnosis of pancreatic tumors
a. Chromogranin expression is characteristic ofsolid pseudopapillary tumors
b. Loss ofmembrane E-cadherin staining is characteristic ofacinar cell
carcinoma
c. Nuclear accumulation of beta catenin is characteristic ofneuroendocrine
tumors
d. Trypsin expression is characteristic ofpancreatoblastoma

12. Adrenal Gland


1.

Immunohistochemistry ofadrenocortical carcinoma

2. Zellballen architecture is characteristic ofwhat lesion


a. Paraganglioma
b. Schwannoma
c. Chordoma
3. Waterhouse-Friderichsen syndrome is due to
a. Meningococcal infection
4. All of the following are secondary causes ofhypertension except
(hypertension is seen in all of the following endocrine disease except)
a. Addison's disease
b. Cushing
c. Con's
d. Acromegaly
5. The most common adrenal malignancy in adults is
a. Metastatic tumors
b. Pheochromocytoma
c. Adrenocortical carcinoma
d. Neuroblastoma
6. Adrenocortical carcinoma is difficult to distinguish from which tumor
a. Renal cell carcinoma
b. Pheochromocytoma
7. A neuroblastoma confined to the adrenal gland does not cross the midline. It
grossly totally excised, but microscopic margins are positive. There is no lymph

node metastasis. Bilateral bone marrow biopsies and aspirates are negative for
tumor and there is no distant metastasis. What is the stage?
a. Stage 1
b. Stage 2a
c. Stage 3
d. Stage 4
e. Stage 4s
8. Which of the following is a poor prognostic feature for neuroblastoma?
a. Duplication of1p
b. FISHfor MYCN- 2 copies
c. FISHfor MYCN- 16 copies
d. Patient age less than 6 months
e. Stage 4s
9. Which of the following would be classified as unfavorable histology?
a. 4 month old boy- neuroblastoma, poorly differentiated, low MKI
b. 8 month old girl- neuroblastoma, differentiating, intermediate MKI
c. 12 month old girl- neuroblastoma, poorly differentiated, high MKI
d. 2 year old boy- neuroblastoma, differentiating, low MKI
e. 7 year old boy- ganglioneuroblastoma, intermixed type
10. Chromogranin distinguishes pheochromocytoma from adrenocortical neoplasm
a. True
b. False
11. The SDHB mutation is almost always associated with benign behavior in
pheochromocytoma
a. True
b. False

12. Carney's triad is an inherited condition


a. True
b. False
13. In which tumor is numerous blasts and myeloid percursors identified?
a. Extramedullary hematopoiesis
b. Extramedullary myeloid tumor
c. Lipoma
d. Liposarcoma
e. Meylolipoma
f.

Neuroblastoma

14. Which tumor is composed ofmature adipose tissue admixed with benign
hematopoietic elements?
a. Extramedullary hematopoiesis
b. Extramedullary myeloid tumor
c. Lipoma
d. Liposarcoma
e. Meylolipoma
f.

Neuroblastoma

15. Which tumor is most often associated with N-myc amplification?


a. Extramedullary hematopoiesis
b. Extramedullary myeloid tumor
c. Lipoma
d. Liposarcoma
e. Meylolipoma
f.

Neuroblastoma

16. The three most important histologic features used in differentiating adrenal
cortical carcinoma from adrenal cortical adenoma are mitotic activity of > 5 /

50 hpf atypical mitoses and venous invasion


a. True
b. False
17. Tumor weight is a reliable measure means ofdetermining ifan adrenal cortical
tumor is benign or malignant
a. True
b. False
18. Adrenal cortical carcinomas are usually positive for both synaptophysin and
chromogranin by immunohistochemistry
a. True
b. False
19. Age and tumor histology correlate strongly with prognosis ofneuroblastoma
a. True
b. False
20. Ifneuroblastoma metastasizes to marrow, liver, and skin the prognosis is poor.
a. True
b. False
21. The nodular macroscopic feature ofneuroblastoma suggests that one slide
alone is not suffiecient for diagnosis classification and determination of
prognosis
a. True
b. False
22. Which condition may arise as a consequence ofmaturation or differentiation of
another entity of the following?
a. Adrenal medullary hyperplasi
b. Adrenal pseudocyst
c. Ganglioneuroma
d. Metastatic carcinoma
e. Neuroblastoma
f.

Pheochromocytoma

23. Which entity is among the most common childhood neoplasms?


a. Neuroblastomas

24. Which condition may be accompanied by signs ofhypercorticalism?


a. Adrenal cortical adenoma
25. Which neoplasm shows positive immunohistochemical positivity for
chromogranin and synaptophysin?
a. Pheochromocytoma
26. Age, tumor histology and MYCN expression correlate strongly with prognosis
in differentiating neuroblastoma.
a. True
b. False
27. Differentiating neuroblastoma is likely to demonstrate binding ofantibody
CD99
a. True
b. False
28. The nodularmacroscopic features ofdifferentiating neuroblastoma suggest
that one slide alone is sufficient for diagnosis classification and prognosis
a. True
b. False
29. Which benign tumor commonly presents as an incidental adrenal/periadrenal
mass?
a. Myelolipoma
30. In the absence ofmetastatic disease, malignancy in pheochromocytoma can be
predicted by:
a. Vascular invasion
b. Abnormal mitotic figures and necrosis
c. Nuclear pleomorphism
d. Pigmentation
e. None ofthe above
31. Which adrenal lesion is almost always associated with hepatosplenomegaly and
abnormal bone marrow findings?
a. Extramedullary hematopoiesis

32. Which lesion is most commonly lcated in the adrenal gland, shows an
admixture ofmature adipose tissue and hematopoietic elements and is most
often an incidental asymptomatic finding?
a. Myelolipoma
33. Which tumor is characterized by sustetacular pattern ofS100 positive
immunostaining?
a. Pheochromocytoma
34. Which malignant neoplasm is characterized by the following immunoprofile:
Melan A + , vimentin + , inhibin + , and calretinin + ?
a. Adrenal cortical carcinoma

13. Urinary Tract


1.

What is the similarity between membranous and post-strept


glomerulonephritis

2. The majority of patients with post-infectious glomerulonephritis will have


a. Complete recovery
b. RPGN
c. MPGN
d. Membranous GN
3. Patient with hemoptysis and renal failure with crescentric proliferative
glomerulonephritis and linear IgG and to lesser extent C3
a. Good pasture
4. Patient with hyperlipidemia, hypoalbuminemia, and proteinuria, has
a. Nephrotic syndrome
5. All of the following are associated with nephrotic syndrome EXCEPT
a. Proteinuria
b. Hypoalbuminemia
c. Hypertension
d. Hyperlipidemia
6. Genetics ofclear cell renal cell carcinoma
a. Loss of3p
7. What immunohistochemistry is useful to differentiate between
angiomyolipoma and clear cell renal cell carcinoma
8. Papillary renal necrosis is associated with
a. Diabetes
b. Myeloma cast
c. Acute interstitial pyelonephritis

9. Common cause of acute pyelonephritis


10. Patient with renal transplant on H&E shows arteritis and tubulitis
a. Acute rejection
b. Chronic rejection
c. Acute tubular necrosis
d. GVHD
11. Which of the following renal tumors shows desmoplastic reaction
a. Collecting duct carcinoma
12. Which of the following neoplasms shows soft tissue invasion and capsular
invasion but still isn't considered to have aggressive behavior
a. Angiomyolipoma ofthe kidney
b. Oncocytoma
13. The commonest site for malakoplalia in the urinary tract is
a. Urinary bladder
b. Renal pelvis
14. Which tumor is associated with VHL
a. Clear cell renal cell carcinoma
b. Papillary renal cell carcinoma
15. What is the microscopic description of rapidly progressive glomerulonephritis
a. Crescent formation
b. Subepithelial IgG deposits
16. Which of the following is least likely to be associated with hematuria
a. Rapidly progressive glomerulonephritis
b. Minimal change disease

c. Membranoproliferative glomerulonephritis
17. To differentiate between angiomyolipoma and leiomyoma
a. HMB45
18. Regarding renal neoplasms which ofthe following is true
a. VHL in a minority ofcases
b. Chromophobe arises from intercalated cells oftubular epithelium
c. Papillary has a worse prognosis than clear cell carcinoma
19. Which of the following features is seen in post-streptococcal
glomerulonephritis
a. Neutrophilic infiltration (suppurative glomerular inflammation)
b. Linear capillary IgG
c. Glomerular microthrombi
d. Subendothelial immune complex deposition by LM
e. Subendothelial immune complex deposition by EM
f.

Double contour

g. Spikes by silver stain


20. Subendothelial dense deposits
a. Membranoproliferative
21. Renal vein thrombosis is associated with
a. DM
b. Wegner's
c. Focal segmental glomerulosclerosis
d. Membranous
e. Membranoproliferative
22. Patient being treated for hypertension now BP 250/150 with increased urea
and creatinine. What are the findings in the kidney?
a. Hyperplastic arteriopathy (arteriosclerosis)
b. Renal vein thrombosis
c. Renal artery stenosis
d. Monckiberg calcinosis
23. What immunocomplex is seen with rapidly progressive glomerulonephritis

24. Which best describes the immune of angiomyolipoma


a. HMB45, Melan A +
b. SMA, caldesmon +
c. CD34, WT1 +
d. Keratin +
25. Large staghorn stone obstructing the renal pelvis, the calyces are dilated and
covered by pink-white material, what is the most likely diagnosis
a. Xanthogranulomatous pyelonephritis
b. Interstitial nephritis
26. Chromosome 11 abnormality
a. Wilm's tumor
27. Renal disease showing by IF a paramesangial immune complex deposition
a. IgA nephropathy
b. FSGS
c. MPGN-I
d. Diabetic glomerulosclerosis
28. Male patient with sudden onset of malignant hypertension (240/190) will have
which of the following renal complication
a. Acute infarction
b. Crescent formation
c. Acute GN
d. Glomerulosclerosis
e. Amyloid deposition
29. Which tumor is characterized by perivascular spindle cells that are SMA and
HMB45 positive
a. Angiomyolipoma

30. Which of the following presents with nephritic syndrome and shows granular
IgG and complement deposition along the capillary wall
a. Membranous GN
b. DM
c. MPGN
d. Amyloidosis
31. Kidney mass showing scar and cells arranged in nests
a. Oncocytoma
32. Xanthogranulomatous pyelonephritis is likely to be caused by
a. E. coli and proteus
33. A renal tumor with gain of chromosome3q, 7, and 17 and loss ofthe Y
chromosome
a. Papillary renal cell carcinoma
34. In post-streptococcal glomerulonephritis
a. Linear deposition
b. Fine granular deposition
c. Coarse granular deposition
35. A patient presents with renal failure. His US showed bilateral renal cysts at the
corticomedullary junction:
a. PKD
b. Acquired cystic disease
c. Medullary cystic disease
36. Sarcomatoid renal cell carcinoma grossly shows
a. Hemorrhage
b. Cysts
c. White firm lesion
37. Which is lost in SLE
a. C5
b. IgM
c. IgA
38. EM of post infectious GN

a. Subepithelial humps
39. Which type of E. coli causes hemolytic uremic syndrome?
a. Toxogenic E. coli
40. What is the commonest renal tumor in a child < 2 months old?
a. Wilm's tumor
b. Clear cell sarcoma
c. Rhabdoid tumor
41. Which immuohistochemical panel below is likely to be positive in oncocytoma
a. CD10, RCC, racemase

b. CK7, Ksp-cadherin
c. KIT, S100A1, Pax2
d. Vimentin, CD10, racemase
42. Which of the following is not typical ofchromophobe renal cell carcinoma
a. Abundant mitochondria
b. Binucleation
c. Colloidal iron positivity
d. Perinuclear halos
e. Solid growth of neoplastic cells
43. A renal tumor composed ofcells with abundant granular cytoplasm
demonstrates immunoreactivity with antibodies against S100A1 or PAX2. The
likely diagnosis
a. Chromophobe renal cell carcinoma
b. Epithelioid angiomyolipoma
c. Oncocytoma
d. Papillary renal cell carcinoma
44. What is the classification ofa urothelial carcinoma with deep muscularis propria
invasion with extension into the prostatic urethra without stromal invasion?
a. T2a
b. T2b
c. T3a
d. T4a
e. T4b
45. Urothelial carcinoma with the presence ofintercellular mucin is sufficient
criteria to diagnose urothelial carcinoma with glandular differentiation
a. True
b. False
46. Which prostate specific immunohistochemical stain has a finely dotted
perinuclear cytoplasmic staining pattern?
a. PSA
b. Prostein
c. P63
d. PSMA
e. HMWCK

47. Immunoreactivity to ehich type ofantibody can aid in differentiating


angiomyolipoma from leiomyosarcoma
a. CK AE1/AE3
b. Desmin
c. EMA
d. HMB45
e. SMA
48. Which of the following statements regarding renal angiomyolipomas in
tuberous sclerosis patients versus the general population not true
a. The incidence is much higher in tuberous sclerosis patients
b. The mean age at diagnosis is lower in tuberous sclerosis patients
c. The tumor is more often bilateral in tuberous sclerosis patients
d. Renal failure due to renal angiomyolipoma is common in tuberous sclerosis
patients
e. Renal angiomyolipomas metastasis is more common in tuberous sclerosis
patients
49. Which histologic feature has been shown to correlate with more aggressive
behavior?
a. Clear cell change ofthe smooth muscle cells
b. Epithelioid cytology of the smooth muscle cells
c. Presence ofmultinucleated giant cells
d. Prominent vascular component
e. Vascular thrombosis
50. Which is the most common renal neoplasm in children 2 months of age
a. Clear cell sarcoma
b. Congenital mesoblastic nephroma
c. Rhabdoid tumor
d. Wilms tumor

e. Leiomyoma
51. Cellular mesoblastic nephromas often exhibit trisomy 11. Which other tumor
harbors a chromosomal abnormality ofchromosome 11
a. Clear cell sarcoma
b. Congenital mesoblastic nephroma
c. Rhabdoid tumor
d. Wilms tumor
e. Leiomyoma
52. Loss of INI1 nuclear staining is associated with which pediatric tumor
a. Clear cell sarcoma
b. Congenital mesoblastic nephroma
c. Rhabdoid tumor
d. Wilms tumor
e. Leiomyoma
53. Loss of INI1 is seen with which other tumor besides renal medullary carcinoma
a. Collecting duct carcinoma
b. Metastatic adenocarcinoma
c. Renal cell carcinoma, unclassified
d. Renal medullary carcinoma
e. Rhabdoid tumor
f.

Urothelial carcinoma with glandular features involving the renal pelvis

54. Patients with this type oftumor typically have sickle trait or rarely sickle cell
anemia

a. Collecting duct carcinoma


b. Metastatic adenocarcinoma
c. Renal cell carcinoma, unclassified
d. Renal medullary carcinoma
e. Rhabdoid tumor
f.

Urothelial carcinoma with glandular features involving the renal pelvis

55. This tumor is usually positive for HMCK, EMA, CK7, ULEX-1 and INI1
a. Collecting duct carcinoma
b. Metastatic adenocarcinoma
c. Renal cell carcinoma, unclassified
d. Renal medullary carcinoma
e. Rhabdoid tumor
f.

Urothelial carcinoma with glandular features involving the renal pelvis

56. Which of the following is true regarding the diagnosis ofangiomyolipoma


a. A non-infiltrative pattern ofgrowth helps distinguish AML from renal cell
carcinoma
b. Splenic and lymph node deposits of AML should always be considered
metastases from either renal or hepatic primary tumors
c. The polygonal-shaped smooth muscle cells in hepatic AML can be mistaken
for HCC
d. The presence of cells resembling lipoblasts help distinguish AML from
liposarcoma
57. Which of the following statements regarding angiomyolipoma is false?
a. Approximately halfof all cases of AML are associated with tuberous
sclerosis

b. Atypia in epithelioid variants ofAML is often confused with


leiomyosarcoma
c. Most cases of AML in the liver are associated with tuberous sclerosis
d. The presence of bilateral AMLs is highly suggestive of tuberous sclerosis
58. Which pair ofimmunostains is MOST likely to help differentiate a fatpredominant angiomyolipoma from a lipoma or well-differentiated liposarcoma?
a. C-kit and Melan-A
b. Desmin and S100
c. EMA and MITF
d. S100 and HMB45
59. Which tumor shows significant clinical and histologic overlap with cystic
nephroma?
a. Angiomyolipoma
b. Cystic partially differentiated nephroblastoma
c. Metanephric adenofibroma
d. Mixed epithelial and stromal tumor
e. Multilocular cystic renal cell carcinoma
f.

Solitary fibrous tumor

60. Which tumor is seen predominantly in children under the age of2?
a. Angiomyolipoma
b. Cystic partially differentiated nephroblastoma
c. Metanephric adenofibroma
d. Mixed epithelial and stromal tumor
e. Multilocular cystic renal cell carcinoma
f.

Solitary fibrous tumor

61. Which tumor is characterized by cysts lined by flattened, cuboidal, or hobnail


apithelium and septa containing ovarian-like stroma?
a. Angiomyolipoma
b. Cystic partially differentiated nephroblastoma
c. Metanephric adenofibroma
d. Mixed epithelial and stromal tumor
e. Multilocular cystic renal cell carcinoma
f.

Solitary fibrous tumor

62. Which tumor is characterized by positive AMACR and CK7 immunostaining and

trisomy 7 and 17?


a. Clear cell papillary renal cell carcinoma
b. Clear cell renal cell carcinoma
c. Multilocular renal cell cell carcinoma
d. Papillary renal cell carcinoma
e. Renal carcinomas associated with Xp11.2
63. Which tumor is positive for nuclear TFE3 immunostaining?
a. Clear cell papillary renal cell carcinoma
b. Clear cell renal cell carcinoma
c. Multilocular renal cell carcinoma
d. Papillary renal cell carcinoma
e. Renal carcinomas associated with Xp11.2
64. Deletions ofthe VHL gene on chromosome 3p25 are seen in which entity?
a. Clear cell papillary renal cell carcinoma
b. Clear cell renal cell carcinoma
c. Multilocular renal cell carcinoma
d. Papillary renal cell carcinoma
e. Renal carcinomas associated with Xp11.2
65. Which entity is peanut lectin + , Ulex europaeus + , RCC -, and CD10 -?
a. Chromophobe renal cell carcinoma

b. Collecting duct carcinoma


c. Conventional clear cell carcinoma
d. Papillary renal cell carcinoma
e. Papillary urothelial carcinoma
66. Which entity characteristically shows karyotype abnormalities trisomy 7,
trisomy 17 and loss of chromosome Y?
a. Chromophobe renal cell carcinoma
b. Collecting duct carcinoma
c. Conventional clear cell carcinoma
d. Papillary renal cell carcinoma
e. Papillary urothelial carcinoma
67. Papillary renal cell carcinoma has an overall better survival rate than
conventional clear cell renal cell carcinoma
a. True
b. False
68. Which entity is characterized by HMB45 + , MiTF + , keratin -, EMA
phenotype?
a. Chromophobe renal cell carcinoma
b. Conventional renal cell carcinoma
c. Epithelioid angiomyolipoma
d. Metastatic HCC
e. Renal oncocytoma
69. Which entity is characterized ultrastructually by cytoplasmic membrane bound
vesicles and shows diffuse granular cytoplasmic staining with Hale's colloidal

iron stain?
a. Chromophobe renal cell carcinoma
b. Conventional renal cell carcinoma
c. Epithelioid angiomyolipoma
d. Metastatic HCC
e. Renal oncocytoma
70. Which entity is characterized by a central scar with a nested growth pattern
and round nuclear contours
a. Chromophobe renal cell carcinoma
b. Conventional renal cell carcinoma
c. Epithelioid angiomyolipoma
d. Metastatic HCC
e. Renal oncocytoma
71. Which of the following structures accounts for the granularity of the
cytoplasm in renal oncocytoma
a. Abundant microvesicles
b. Iron pigment
c. Lysosomes
d. Mitochondria
e. Rhomboid crystals
72. Which of the following features of chromophobe renal cell carcinoma is ost
useful in distinguishing it from oncocytomas?
a. Absence ofvimentin expression

b. Collecting duct immunophenotype


c. Diffuse strong cytoplasmic staining pattern with Hale's colloidal iron
d. Finely granular cytoplasm
e. Presence ofmitochondrial DNA abnormalities
73. Which of the following is a recurrent cytogenetic abnormality in oncocytoma?
a. Loss ofchromosome 3p
b. Rearrangement ofchromosome 11q12-13
c. Trisomy 12
d. Trisomy 16
e. Trisomy 20
74. Renal tumors containing melanin pigment are diagnostic ofmelanoma
a. True
b. False
75. The diagnostic immunoprofile ofclassic angiomyolipoma is positive for HMB45
and SMA and negative for S100 and keratin?
a. True
b. False
76. Monotypic epithelioid angiomyolipoma are potentially malignant neoplasms
a. True
b. False
77. Which condition shows non-specific changes of interstitial fibrosis and tubular
atrophy?
a. Acute cellular allograft rejection
b. Acute tubular necrosis
c. Chronic allograft nephropathy
d. Cyclosporine/tacrolimus toxicity
e. Post-transplant lymphoproliferative disorder
78. Which condition is graded according to severity oftubulitis or arteritis?
a. Acute cellular allograft rejection
79. Which condition shows a predominantly B cell infiltrate?
a. Post transplant lymphoproliferative disorder

80. Which of the following is true ofthe immunophenotype ofrenal cell carcinoma?
a. Both CK 7 and CK 20 are usually positive in clear cell renal cell carcinoma
b. RCC antigen is expressed in nearly 100% ofclear cell renal cell carcinoma
c. Wimentin is negative in most cases ofchromophobe renal cell carcinoma
d. CD10 is negative in most cases ofpapillary renal cell carcinoma
81. Which of the following is true about immunohistochemical distinction of clear
cell renal cell carcinoma and adrenocortical carcinoma?
a. Expression ofvimentin favors clear cell renal cell carcinoma
b. Chromogranin and synaptophysin are generally positive in adrenocortical
carcinoma
c. CK is expressed in clear cell renal cell carcinoma but is usually not
expressed in adrenocortical carcinoma
d. Expression ofmelan A and inhibin is often seen in both tumors
82. Which tumor most likely demonstrates gains ofchromosome 7 and 17?
a. Carcinoid
b. Metanephric adenomafibroma
c. Metanephric adenoma
d. Papillary renal cell carcinoma
e. Wilm's tumor
83. Which tumor is most likely to show abnormalities of chromosome 1?
a. Carcinoid
b. Metanephric adenofibroma
c. Metanephric adenoma
d. Papillary renal cell carcinoma
e. Wilm's tumor
84. Which tumor would typically show an immunophenotype ofEMA CK7 AMACR
positive and WT1 negative?
a. Carcinoid
b. Metanephric adenofibroma
c. Metanephric adenoma
d. Papillary renal cell carcinoma
e. Wilm's tumor
85. Phenacetin abuse and Balkan nephropathy are associated with a greater risk
ofdelepoing upper urinary tract urothelial carcinoma than bladder urothelial

carcinoma
a. True
b. False
86. The most common presenting symptom for renal pelvic urothelial carcinoma is
flank pain
a. True
b. False
87. Upper urinary tract urothelial carcinoma may occur as part ofthe hereditary
nonpolyposis colorectal carcinoma syndrome.
a. True
b. False
88. Which tumor possesses rhomboid shaped renin granules on EM?
a. Juxtaglomerular cell tumor
89. Which tumor typically expresses HMB45 in neoplastic cells?
a. Angiomyolipoma
90. Which tumor frequently has psammoma bodies in the stroma?
a. Metanephric adenoma
91. Which are the most important prognostic features ofWilm's tumor?
a. Stage
b. Histology

c. Age and MYC N expression


d. A & B
e. A, B, & C
92. Which of the following antigens may react in Wilm's tumor?
a. Vimentin and perhaps CK
93. Which of the following tumors are likely to be considered in the differential
diagnosis ofblastemal-rich Wilm's tumor?
a. Primary renal neuroblastoma and peripheral PNET
b. Malignant rhabdoid tumor ofkidney
c. Clear cell sarcoma ofkidney
d. Congenital mesoblastic nephroma
e. B, C, and D
94. Xanthogranulomatous pyelonephritis and renal cell carcinoma may easily be
distinguished from each other by several laboratory and radiographic studies
a. False
95. Features that will favor a diagnosis ofXPN over clear cell RCC include?
a. A zonal pattern ofinflammation with central acute inflammation clusters of
foamy histiocytes fibrosis and giant cells
96. Which papillary lesion can be diagnosed cytologically in urine by the presence
of3D clusters ofdysplastic urothelial cells with altered polarity?
a. Papillary urothelial carcinoma, low grade
97. Which lesion is not biologically aggressive but is associated with recurrences
and carries a risk for progression i.e. invasion?
a. Papillary urothelial neoplasm oflow malignant potential
98. Which neoplasm is characterized by slender delicate filiform processes on the
mucosal surface and tight cell clusters in the subepithelial connective tissue

with prominent retraction artifact resembling angiolymphatic invasion?


a. Urothelial carcinoma with micropapillary components
99. Which renal tumor is composed of fat cells with hyperchromatic nuclei and
spindle cells within fibrous bands, and is immunoreactive for S100 and negative
for HMB45?
a. Well-differentiated liposarcoma
100.

Which tumor characteristically contains pleomorphic epithelioid cells

and spindle cells immunoreactive for keratin and epithelial membrane antigen?
a. Sarcomatoid renal cell carcinoma
101. For which tumor is the presence in regional lymph nodes, renal vein/vena cava
extension or involvement ofperirenal tissue not indicative of malignant
progression?
a. Angiomyolipoma
102.

Loss ofchromosomes 1, 2, 10,13, 6, 21, and 17 are characteristic ofwhich

entity?
a. Chromophobe renal cell carcinoma
103.

Which entity is characterized by interstitial nephritis, often patchy, with

tubular epithelial cell nuclear inclusions?


a. Polyoma (BK) virus
104.

Which entity can have interstitial nephritis with tubulitis, glomerular

endothelial injury and/or vascular endarteritis or fibrinoid necrosis?


a. Acute cellular rejection
105.

Which entity has an expansile atypical lymphoplasmacytic proliferation

and is associated with EBV infection?


a. Post-transplant lymphoproliferative disorder

106.

Which tumor lacks solid expansile nodules and is composed entirely of

multiloculated cysts lined by hobnail cells and thin-walled septa containing


fibrous connective tissue?
a. Cystic nephroma
107.

Which tumor has solid areas as well as clustering ofcysts into a

multilocular mass and contains estrogen receptor positive positive spindle cells?
a. Mixed epithelial and stromal tumor
108.

Which tumor contains vascularized nests ofclear cells and an atypical

mitotically active spindle cell component?


a. Sarcomatoid renal cell carcinoma
109.

Immunohistochemistry is generally helpful in the differential diagnosis

ofspindle cell proliferations ofthe urinary bladder


a. True
b. False
110. Which of the following typically occurs following vesical trauma?
a. Postoperative spindle cell nodule
111. Pseudosarcomatous reactive myofibroblastic proliferation may be histologically
indistinguishable from
a. Postoperative spindle cell nodule and pseudosarcomatous desmoplastic
reaction to urothelial carcinoma
112. An exophytic lesion characterized by compact haphazard fascicles of
mitotically active cytologically bland spindle cells that are CK, vimentin, MSA,
and desmin + , but EMA

a. Postoperative spindle cell nodule


113. A polypoid myxoid tumor composed of mitotically active, atypical spindle cells
that are CK + , EMA + and vimentin +
a. Sarcomatoid urothelial carcinoma
114. A myxoid mural mass with granulation tissue-type vascularity a polymorphous
chronic inflammatory infiltrate and widely spaced cytologically bland spindle
cells that are MSA + , vimentin + , and CK
a. Inflammatory pseudotumor
115. Which tumor has a characteristic diffuse linear membranous staining pattern
for CD99?
a. Primitive neuroectodermal tumor
116. Which tumor predominates in males has a propensity for bone metastasisand
rarely at extrarenal sites?
a. Clear cell sarcoma
117. Which tumor is composed entirely of CD34 + spindle cells and demonstrates
intratumoral angiodysplasia "onion skinning" around entrapped tubules and
heterologous differentiation?
a. Metanephric stromal tumor
118. A cellular tumor with focaly hyalinized stroma composed of tightly packed
small round tubules lined by cells with small uniform cytologically bland nuclei
and scant pale staining cytoplasm
a. Metanephric adenoma
119. A tumor having a nested and archipelaginous pattern and containing a uniform
population ofcells with low grade nuclear features granular eosinophilic
cytoplasm and patchy weak peri-membranous staining with Hale's colloidal iron
a. Renal oncocytoma
120.

A tumor having a sheet-like and trabecular architecture and containing

cells with grade 2 nuclear features granular eosinophilic cytoplasm and

uniformly strong diffuse reticular cytoplasmic staining with Hale's colloidal iron
a. Chromophobe renal cell carcinoma, eosinophilic variant
121. A putative bengn precursor lesion of low grade papillary urothelial neoplasms
a. Papillary urothelial hyperplasia
122.

A papillary lesion that can be diagnosed cytologically in urine specimens

by the presence of3D clusters ofdysplastic urothelial cells with altered


polarity
a. Papillary urothelial carcinoma, low grade
123.

A lesion which is not biologically aggressive but is associated with

recurrence in 40 50 percent ofcases and carries a risk for progression with


each recurrence having a 10 -20 percent change of being invasive
a. Papillary urothelial neoplasm oflow malignant potential
124.

All ofthe following are true about sarcomatoid renal cell carcinoma

except
a. Survival rate usually less than one year
b. High rate of extrarenal spread and metastasis at time ofsurgery
c. HMB45 is usually positive
d. Usually arises in the renal cortex
e. May have foci ofosteosarcoma or chondrosarcoma
125.

All ofthe following are true about angiomyolipoma except

a. HMB45 is usually positive


b. Strong association with tuberous sclerosis
c. Clinical findings (flank pain and hematuria) similar to renal cell carcinoma
d. Most common renal tumor ofearly infancy
e. May be multifocal aand bilateral but rarely malignant

126.

Classic Wilm's tumor has which of the following characteristics?

a. Triphasic histology with epithelial blastemal and mesenchymal components


b. Prognosis favorable with greater than 80% cure rate
c. Stage based on extent oftumor, extent ofresection, and presence of
metastasis
d. Association with dysmorphic syndromes
e. All of the above
127.

Malignant rhabdoid tumor ofthe kidney has which of the following

characteristics?
a. Favorable prognosis with greater than 50% cure rate
b. Immunohistochemical profile identical to RMS
c. Association with primary or metastatic CNS neoplasms
d. Association with VHL
e. Tendency for metastasis to bone
128.

Clear cell sarcoma ofthe kidney has which of the following

characteristics?
a. Deceptively bland histologic appearance
b. Arcading vascular pattern
c. Tendency for metastasis to bone
d. Peak incidence between 6 months and 2 years of age
e. All of the above
129.

Which tumor typically displays loss of INI1 immunoexpression?

a. Rhabdoid tumor
130.

Which tumor is characterized by diffuse CK positivity positive CD10 and

negative HepPar1?
a. Clear cell renal cell carcinoma
131. Which of the following features is required for diagnosis ofanaplasia in Wilm's
tumor?
a. Abnormal multipolar mitotic figures

b. Marked nuclear enlargement (nuclear size 3x that ofthe surrounding


tumor cells)
c. Nuclear hyperchromasia
d. Any of the above features
e. All of the above features
132.

Which ofthe following renal tumor predisposition syndromes is

characterized by gonadal dysgenesis and nephropathy due to mesangial


sclerosis?
a. Denys-Drash syndrome
133.

This pediatric renal tumor showed pre-operative rupture ofthe renal

capsule and extension oftumor into the perirenal fat. Imaging studies showed
no evidence of tumor in the contralateral kidney and no evidence of
metastatic tumor outside ofthe abdomen and pelvis. What is the pathologic
stage of this tumor?
a. Stage III
134.

Which is positive for nuclear TFE3 staining?

a. Renal carcinomas associated with Xp11.2 translocations


135.

Which tumor is characterized by the presence ofcalcium oxalate

crystal deposition?
a. Acquired cystic disease-associated renal cell carcinoma
136.

A malignant spindle cell tumor of the kidney that contains an in situ

epithelial component in the renal pelvis is best classified as?


a. Sarcomatoid urothelial carcinoma
137.

A malignant spindle cell neoplasm with no associated epithelial

component and a CD10/SMC + immunoprofile, most likely represents which

entity?
a. Leiomyosarcoma
138.

Which staining pattern is most consistent with chromophobe RCC?

a. Vimentin -, Hales colloidal iron + (diffuse),CK7+ (diffuse), CD10 -, CD117


+
b. Vimentin -, Hales colloidal iron + (focal), CK7 + (focal),CD10 + , CD117 +
c. Vimentin -, Hales colloidal iron -, CK7 ,CD10 -, CD117
d. Vimentin + , Hales colloidal iron -, CK7 -,CD10 -, CD117

139.

Which histologic feature has been associated with metastases and local

recurrence?
a. Abundant microvesicles on ultrastructural examination
b. Binucleation
c. Fuhrman nuclear grade
d. Sarcomatoid transformation

140.

Which ofthe following statements regarding hereditary chromophobe

RCC is NOT true?


a. Aggressive behavior has been associated with hereditary form of this
tumor
b. Birt-Hogg-Dube syndrome is linked to FLCN gene mutation mapped to
chromosome 17p11.2
c. Birt-Hogg-Dube syndrome is an autosomal dominant condition characterized
by benign cutaneous tumors, pulmonary cysts, and multifocal bilateral renal
cancer
d. The most common renal cell tumor seen in Birt-Hogg-Dube syndrome is the
oncocytic hybrid tumor

14. Male Reproductive System


1.

Which is the most radiosensitive testicular tumor

2. What is the commonest testicular tumor in children


a. Yolk sac tumor
b. Embryonal carcinoma
c. Seminoma
3. All of the following are true regarding PSA except
a. PSA is cancer specific not organ specific
4. What is the most important prognostic factor in prostatic carcinoma
a. Gleason score
b. Perineural invasion
5. Which of the following is a risk factor for testicular tumor
a. Cryptorchidism
6. A child underwent removal ofundescended testis, then he developed
paratesticular swelling, what is the most likely cause
a. Hydrocele
b. Hematocele
c. Spermatocele
d. Spermatic cord infarction
e. Torsion
7. In an older patient which testicular lesion is he most likely to develop
a. Spermatocytic seminoma
b. Orchitis
c. Torsion
d. Choriocarcinoma

e. Embryonal carcinoma
8. Regarding Tb of the epididymis which ofthe following is true
a. Almost always primary involvement
9. Prostatic squamous metaplasia is associated with which ofthe following
a. Benign prostatic hyperplasia
10. Yolk sac tumor
a. Increased serum AFP
11. The most common paratesticular tumor in children
a. Spindle cell variant ofembryonal rhabdomyosarcoma
12. What is the best immunostain for embryonal carcinoma?
a. CD30
b. CEA
c. C-kit
d. Hcg
e. PLAP
13. Which tumor is characterized morphologically by malignant glandular and
mesenchymal elements and is frequently associated with metastases and poor
prognosis?
a. GIST
b. Leiomyosarcoma
c. Sarcomatoid carcinoma
d. Stromal sarcoma
e. Stromal tumor of uncertain malignant potential
14. Which tumor is characterized by diffuse immunoreactivity for CD117?
a. GIST
b. Leiomyosarcoma
c. Sarcomatoid carcinoma
d. Stromal sarcoma
e. STUMP
15. Which tumor is characterized by the following histologic features and
immunophenotype: hypercellular, hyperchromatic spindle cell lesion with mild

to moderate cytologic atypia, no mitoses, no necrosis, and vimentin+ , CD34+ ,


PR+ ?
a. GIST
b. Leiomyosarcoma
c. Sarcomatoid carcinoma
d. Stromal sarcoma
e. STUMP
16. Low stage primary testicular lymphoma has a favorable prognosis
a. True
b. False
17. Cryptorchidism predisposes to development oftesticular lymphoma
a. True
b. False
18. Most primary testicular lymphomas are large B-cell and of germinal center
type.
a. True
b. False
19. Testicular tumors of adrenogenital syndrome are usually unilateral as opposed
to leydig cell tumors which are usually bilateral

a. True
b. False
20. Leydig cell hyperplasia is commonly seen in atrophic testes
a. True
b. False
21. Important features of malignant leydig cell tumors include: size > 5 cm, an
infiltrative margin, cytologic and nuclear atypia, necrosis, lymphovascular
invasion and increased mitotic rate (> 5 mitoses/10 HPF)
a. True
b. False
22. Which tumor typically shows a proliferation ofbland spindle-shaped cells with a
noticeable amount of adipose tissue and thick bundles of brightly eosinophilic
collagen?
a. Aggressive angiomyxoma
b. Angiomyofibroblastoma-like tumor ofthe genital tract (cellular
angiofibroma)
c. Solitary fibrous tumor
d. Spindle cell lipoma
23. Which tumor shows a proliferation of bland spindle-shaped cells and numerous
small and medium sized vessels within an edematous to fibrous stroma?
a. Aggressive angiomyxoma
b. Angiomyofibroblastoma like tumor of the genital tract (cellular
angiofibroma)
c. Solitary fibrous tumor
d. Sindle cell lipoma
24. Which tumor shows a proliferation of bland spindle shaped cells dispersed
among keloid type collagen bundles and a hemangiopericytoma like vascular

pattern
a. Aggressive angiomyxoma
b. Angiomyofibroblastoma like tumor of the genital tract (cellular
angiofibroma)
c. Solitary fibrous tumor
d. Spindle cell lipoma
25. Which adult testicular germ cell tumor characteristically has the following
immunohistochemical profile: PLAP + , CD117 + , CD30 -, AFP -, and inhibin -?
a. Choriocarcinoma
b. Embryonal carcinoma
c. Seminoma
d. Sertoli cell tumor
e. Yolk sac tumor
26. Which adult testicular germ cell tumor characteristically has the following
immunohistochemical profile: AFP + , AE1/AE3 + , CD30 and OCT 3/4 -?
a. Yolk sac tumor
27. Which adult testicular germ cell tumor characteristically has the following
immunohistochemical profile: PLAP + , OCT 3/4 + , CD30 + , AE1/AE3 + , inhibin -?
a. Embryonal carcinoma
28. Spermatocytic seminoma is the only testicular germ cell tumor that does not
occur at an extra-gonadal site
a. True
b. False
29. Which of the following is true regarding spermatocytic seminoma?
a. One of the most common germ cell tumors in adolescence

b. Commonly associated with IGCNU


c. One of the most common germ cell tumors in cryptorchid testes
d. Generally has an excellent prognosis except in the rare circumstance when
it is associated with a sarcomatous component
30. Seminoma classic type may be differentiated from spermatocytic seminoma by
all of the following criteria except
a. Seminoma classic type is commonly associated with IGCNU and other germ
cell types
b. The neoplastic cells in seminoma classic type commonly have abundant
cytoplasmic glycogen and are usually diffusely positive for OCT3/4
c. The neoplastic cells ofseminoma classic type are characteristically a
polymorphous population of small, medium, and large cells
d. Seminoma classic type commonly has a prominent lymphocytic infiltrate
and occasional granulomatous stromal reaction
31. Which is the most common testicular neoplasm in patients over 60 years of
age?
a. DLBL
32. Which is the most common pure testicular germ cell neoplasm?
a. Seminoma
33. Which germ cell tumor is most consistently immunohistochemically positive for
CD30?
a. Emryonal carcinoma
34. Which testicular tumor is more likely to occur in prepubertal children than
adults?
a. Mature teratoma
35. Adenocarcinoma occurring in a teratoma is designated:
a. Teratoma with malignant transformation
36. A PNET is most likely to occur in:
a. Immature teratoma
37. An important morphologic feature differentiating classic seminoma from
spermatocytic seminoma or Sertoli cell tumor is that classical seminoma is

associated with intratubular germ cell neoplasia ofthe unclassified type.


a. True
b. False
38. The delicate fibrovascular stroma with lymphocytic infiltrate as seen in classic
seminoma is not seen in other testicular germ cell tumors or Sertoli cell tumor.
a. True
b. False
39. An important difference between classic seminoma, yolk sac tumor, and
embryonal carcinoma is that seminoma is negative for PLAP, podoplanin, and
CD117, but positive for AE1/AE3 and CAM5.2
a. True
b. False

15. Female Reproductive System


1.

Uterine tumor with tubuloglandular growth, high ER, PR expression and PTEN
mutation
a. Endometrioid
b. Papillary serous carcinoma
c. Clear cell carcinoma

2. Unopposed estrogen causes which type of endometrial cancer


a. Endometrioid
3. Ovarian tumor which is PLAP positive, CK positive, CD 30 positive, Oct 4
positive, CD99 positive, and CD117 negative
a. Embryonal carcinoma
b. Yolk sac tumor
c. Dysgerminoma
4. Primary ovarian tumor showing transitional, squamous, and undifferentiated
carcinoma with nests ofnormal transitional epithelium. What should it be
designated as
a. Malignant brunner tumor
b. Transitional cell carcinoma
5. Ovarian tumor with pseudoprecocious puberty
a. Juvenile graanulosa cell tumor
6. Bilateral ovarian tumor in young which has spread by the time of diagnosis
a. Small cell carcinoma, hypercalcemic type
b. Adult granulosa cell tumor
c. Sex cord tumor with annular tubules
d. Clear cell carcinoma
e. Serous carcinoma
7. Which is the most common malignant germ cell tumor in young females

a. Dysgerminoma
b. Immature teratoma
c. Choriocarcinoma
d. Granulosa cell tumor
8. Which of the following tumors shows BRAF and KRAS mutations and arise from
benign neoplasms
a. Endometrioid carcinoma
b. Serous carcinoma, low grade
c. Serous carcinoma, high grade
d. Clear cell carcinoma
e. Ovarian endometrioid carcinoma
9. Ovarian tumor with BRCA-1 mutation
a. Clear cell carcinoma
b. Serous carcinoma, high grade
c. Serous carcinoma, low grade
d. Endometrioid carcinoma
e. Mucinous carcinoma
10. A tumor which does NOT recur after complete excision, and on H&E shows
epithelioid cells and has a tendency to grow around blood vessels
a. Angiomyofibroblastoma
b. Epithelioid sarcoma
c. Aggressive angiomyxoma
d. Leiomyosarcoma
11. Which tumor in the endocervix is considered low grade and has good
prognosis
a. Endocervical adenocarcinoma, usual type
b. Endometrioid adenocarcinoma ofthe uterus involving the cervix
c. Papillary serous adenocarcinoma of the uterus
d. Villoglandular papillary adenocarcinoma of the cervix

12. Which of the following is true regarding uterine papillary serous


adenocarcinoma
a. Has BRCA1
b. Has P53 mutation
c. Occurs with unopposed estrogen
d. Has KRASmutation
e. Occurs in women in the 2nd-3rd decades oflife
13. Which ovarian tumor occurs in younger women and causes virilization
symptoms
a. Steroid-secreting tumor
b. Sertoli-leydig cell tumor
14. Which ovarian tumor occurs in post-menopausal women, is bilateral and has
paraneoplastic hormonal effect
a. Small cell carcinoma ofthe ovary, pulmonary type
b. Small cell carcinoma ofthe ovary, hypercalcemic type
15. Grading of immature teratoma depends on
a. Neuroepithelium component
16. Which uterine tumor is typically located subserosally, most frequently is
circumscribed, usually occurs in women of reproductive age and is calretenin
and thrombomodulin positive?
a. Adenomatoid tumor
17. A woman shows vulvar thinning of the epidermis with dermal fibrosis and
perivascular inflammatory infiltrate (lichen sclerosis et atrophicus), which ofthe
following is true
a. Increase risk for squamous cell carcinoma
18. Which ovarian tumor shows P53, BRAF1 and BRAF2
a. High grade serous carcinoma
19. Polypoid mass showing malignant stromal cells cuffing glands
a. Adenosarcoma
b. Low grade stromal sarcoma
c. MMMT

20. Which of the following might affect ovarian cancer staging


a. Capsular invasion
21. A female patient with vulvar lesion, locally aggressive, has myxoid background
with stellate shaped stromal cells positive for SMA and multiple thick walled
blood vessels
a. Aggressive angiomyxoma
b. Angiomyofibroblastoma
22. Female patient has an ovarian tumor showing tubular structures, positive for
GFAP, NSE, S100, and neurofilament
a. Immature teratoma
23. Which is the most common germ cell tumor in females
a. Mature cystic teratoma
b. Dysgerminoma
c. Yolk sac tumor
24. Which tumor is associated with endometriosis
a. Clear cell carcinoma ofovary
b. Endometroid carcinoma, secretory type
25. Female patient with large placenta, diffuse villous edema, and fetal hydrops
a. Hydrops placentalis
b. Hydrops fetalis
c. Choriocarcinoma
d. Partial mole
e. Glycogen storage disease

26. Which of the following organisms is the cause of normal vaginal PH


a. Lactobacillus acidophilus
27. 2ry squamous cell carcinoma most commonly occurs in
a. Mature teratoma
28. PJsyndrome is associated with which ovarian tumor
a. Sex cord stromal tumor with annular tubules
29. All are associated with ectopic pregnancy except
a. Adenomatoid tumor
b. IUD
c. Infection
d. Salpingitis isthmica nodosa
30. Cystic papillary uterine mass with atypia and WT=1 positivity
31. Which lesion causes or presents with hyperestrogenism
a. Adult granulosa cell tumor
32. What is the most common source for metastatic carcinoma to the ovary
a. GI
33. A 47 year old woman has a 10 cm cyst in the left ovary and moderately raised
levels of CA125. Histology ofthe cyst shows a borderline serous tumor. The
omentum shows psammoma bodies and small papillary serous epithelial
structures on the surface and embedded within the fat (haphazardly arranged
and surrounded by granulation tissue and inflammatory infiltrate). Peritoneal
washing contains fragments ofbland serous epithelium. What is the diagnosis?
a. Borderline serous tumor with endosalpingiosis
b. Borderline serous tumor with non-invasive desmoplastic implants
c. Primary peritoneal adenocarcinoma of serous type with synchronous
ovarian borderline tumor

d. Stage 1 serous cystadenocarcinoma with benign serous implants


e. Stage 3 serous cystadenocarcinoma ofovary with peritoneal spread
34. Which of the following statements about molar pregnancy is true?
a. By immune, p57 is typically positive in partial moles and negative in
complete moles
b. Complete hydatidiform mole is typically associated with a complete and
well-formed fetus
c. Partial hydatidiform mole typically shows no associated fetal parts
d. The placental karyotype ofcomplete hydatidiform mole is triploid
e. The placental karyotype ofpartial hydatidiform mole is typically diploid
35. Which of the following lesions is a consequence of fetal thrombotic
vasculopathy?
a. Complete hydatidiform mole
b. Hemorrhagic endovasculitis
c. Mesenchymal dysplasia
d. Partial hydatidiform mole
e. Villitis ofunknown etiology
36. Which of the following genetic syndromes is associated with placental
mesenchymal dysplasia?
a. Beckwith-Wiedmann syndrome
b. Down syndrome
c. Gonadal dysgenesis
d. Trisomy 18
e. Turner syndrome mosaic
37. Which tumor typically expresses CD117 and OCT4 is associated with
abnormalities ofchromosome 12
a. Clear cell carcinoma
b. Diffuse large B cell lymphoma
c. Dysgerminoma
d. Metastatic renal cell carcinoma
38. Which neoplasm is most commonly associated with endometriosis
a. Clear cell carcinoma ofthe ovary

b. Diffuse large B cell lymphoma


c. Dysgerminoma
d. Metastatic renal cell carcinoma
39. Which tumor is more commonly associated with gonadal dysgenesis
syndromes e.g. testicular feminization (46XY)
a. Clear cell carcinoma ofovary
b. Diffuse large B cell lymphoma
c. Dysgerminoma
d. Metastatic renal cell carcinoma
40. CD10 expression is pathognomonic ofendometrial stromal sarcoma
a. True
b. False
41. The detection of a JAZF1-JJAZ1 gene fusion may help distinguish an
endometrial stromal sarcoma from a highly cellular smooth muscle tumor
a. True
b. False
42. Given the hormone responsiveness ofendometrial stromal sarcoma,
preservation ofthe ovaries is not recommended

a. True
b. False
43. Which ovarian tumor is characterized by spindle cells with moderate to severe
nuclear atypia and > 4 mitoses/10HPF
a. Cellular fibrothecoma
b. Cellular leiomyoma
c. Endometrial stromal sarcoma
d. Fibrosarcoma
e. GIST
44. Which ovarian tumor is unlikely to recur unless associated with adhesions or
rupture?
a. Cellular fibrothecoma
b. Cellular leiomyoma
c. Endometrial stromal sarcoma
d. Fibrosarcoma
e. GIST
45. Which ovarian tumor is most likely to display tongue-like projections at its
peripheral borders and CD10 immunoreactivity?
a. Cellular fibrothecoma
b. Cellular leiomyoma
c. Endometrial stromal sarcoma
d. Fibrosarcoma
e. GIST
46. Which tumor is characteristically poorly demarcated?
a. Aggressive angiomyxoma
b. Angiomyofibroblastoma
c. Cellular angiofibroma
d. Solitary fibrous tumor
47. Which tumor is typicaly positive for desmin but not MSA or CD34?
a. Aggressive angiomyxoma
b. Angiomyofibroblastoma
c. Cellular angiofibroma
d. Solitary fibrous tumor

48. Which tumor is characterized by hemangiopericytoma-like vascular pattern


and CD34 positivity?
a. Aggressive angiomyxoma
b. Angiomyofibroblastoma
c. Cellular angiofibroma
d. Solitary fibrous tumor
49. Which is characteristic ofovarian sclerosing stromal tumors?
a. They are typically bilateral
b. They typically occur in the 5th decade or later
c. They typically occur in patients 30 years or younger
d. They typically are hormonally active
e. They typically co-exist with ovarian mucinous tumors
50. Which immunomarker is most typically positive in an ovarian sclerosing stromal
tumor?
a. AE1/AE3
b. Inhibin
c. Calretenin

d. Desmin
e. CK7
51. What is most characteristic ofthe signet ring cells seen in some sclerosing
stromal tumors
a. They contain mucin
b. They contain lipid
c. They contain glycogen
d. They will express AE1/AE3 and CK20
e. They are strongly inhibin and desmin positive
52. Which ovarian tumor is most often associated with pseudoprecocious puberty
a. Choriocarcinoma
b. Dysgerminoma
c. Embryonal carcinoma
d. Juvenile granulosa cell tumor
e. Yolk sac tumor
53. Which ovarian tumor is associated with tumor-infiltrating lymphocytes and
epithelioid granulomas
a. Choriocarcinoma
b. Dysgerminoma
c. Embryonal carcinoma
d. Juvenile granulosa cell tumor
e. Yolk sac tumor
54. An ovarian tumor has the following immunohistochemical profile: PLAP positive,
OCT4 positive, CD30 positive, CK positive, and CD117 negative. What is the most

likely diagnosis
a. Choriocarcinoma
b. Dysgerminoma
c. Embryonal carcinoma
d. Juvenile granulosa cell tumor
e. Yolk sac tumor
55. Juvenile granulosa cell tumors are never diagnosed in adults
a. True
b. False
56. Juvenile granulosa cell tumors similar to other sex cord stromal tumors are
always negative for EMA
a. True
b. False
57. Similar to adult granulosa cell tumor recurrences in juvenile granulosa cell
tumor frequently occur more than five years after initial diagnosis
a. True
b. False
58. Which neoplasm frequently lacks evidence of urothelial differentiation on
immunohistochemical study for markers e.g. uroplakin III, thrombomodulin or
CK20
a. Benign Brenner tumor
b. Borderline (proliferative) Brenner tumor
c. Malignant Brenner tumor
d. Metastatic urothelial carcinoma

e. Transitional cell carcinoma ofthe ovary


59. What is correct classification for a primary ovarian tumor with areas of
urothelial squamous or undifferentiated carcinoma found in association with
nests of benign urothelial cells
a. Benign Brenner tumor
b. Borderline Brenner tumor
c. Malignant Brenner tumor
d. Metastatic urothelial carcinoma
e. Transitional cell carcinoma ofthe ovary
60. Which neoplasm should be suspected when the tumor composed of high grade
urothelial cells that stain positive for uroplakinIII, thrombomodulin and CK20
but does not have an adjacent benign or borderline component ofbrenner
tumor
a. Benign Brenner tumor
b. Borderline Brenner tumor
c. Malignant Brenner tumor
d. Metastatic urothelial carcinoma
e. Transitional cell carcinoma ofthe ovary
61. Which tumor is characterized by the presence ofa malignant epithelial
component and a malignant mesenchymal component
a. Carcinosarcoma
b. Mullerian adeosarcoma
c. Rhabdomyosarcoma
d. Undifferentiated uterine sarcoma
62. Which tumor is characterized by the presence ofa benign epithelial
component and a malignant mesenchymal component
a. Carcinosarcoma
b. Mullerian adeosarcoma
c. Rhabdomyosarcoma
d. Undifferentiated uterine sarcoma
63. Which tumor does not show any particular line of differentiation both by
morphology and immunophenotype
a. Carcinosarcoma
b. Mullerian adenosarcoma

c. Rhabdomyosarcoma
d. Undifferentiated uterine sarcoma
64. Epithelioid trophoblastic tumor is associated with antecedent hydatidiform
moles
a. True
b. False
65. P63 is diffusely positive in epithelioid trophoblastic tumor
a. True
b. False
66. Epithelioid trophoblastic tumor, like choriocarcinoma, responds well to
chemotherapy
a. True
b. False
67. Which entity is characterized by diffusely enlarged, often cavitated
68. In which tumor is the median age ofpatients less than 50 years?
a. Endometrioid adenocarcinoma
b. Endometrioid borderline tumor
c. Metastatic colorectal adenocarcinoma
d. Sertoli-leydig cell tumor
69. Which of the tumors is often associated with endometriosis or an
adenofibroma, and has either a confluent pattern ofat least 5 mm or
infiltration?
a. Endometrioid adenocarcinoma
b. Endometrioid borderline tumor
c. Metastatic colorectal adenocarcinoma
d. Sertoli-leydig cell tumor
70. Which of the tumors is generally negative for EMA?
a. Endometrioid adenocarcinoma

b. Endometrioid borderline tumor


c. Metastatic colorectal adenocarcinoma
d. Sertoli-leydig cell tumor
71. Which tumor is composed ofcells which morphologically resemble proliferative
phase endometrial stroma and shows permeative infiltration ofthe
myometrium
a. Endometrial stromal nodule
b. Endometrial stromal sarcoma, low grade
c. Leiomyosarcoma
d. Undifferentiated uterine sarcoma
72. Which tumor is composed ofcells with marked cellular atypia and numerous
mitoses (including atypical forms) without evidence of differentiation towards a
recognizable uterine component?
a. Endometrial stromal nodule
b. Endometrial stromal sarcoma, low grade
c. Leiomyosarcoma
d. Undifferentiated uterine sarcoma
73. Which tumor is well circumscribed and composed ofcells which
morphologically resemble proliferative phase endometrial stroma?
a. Endometrial stromal nodule
b. Endometrial stromal sarcoma, low grade
c. Leiomyosarcoma
d. Undifferentiated uterine sarcoma
74. Which entity has uniform nuclei with mild to moderate atypia, up to 12 mitoses
per 10 high power fields, and destructive stromal invasion?
a. High grade serous carcinoma

b. Low grade serous carcinoma


c. Retiform sertoli-leydig cell tumor
d. Serous neoplasm of low malignant potential
75. Which entity expresses WT-1 and inhibin?
a. High grade serous carcinoma
b. Low grade serous carcinoma
c. Retiform sertoli-leydig cell tumor
d. Serous neoplasm of low malignant potential
76. The prognosis of which entity depends upon the stage and type of implant?
a. High grade serous carcinoma
b. Low grade serous carcinoma
c. Retiform sertoli-leydig cell tumor
d. Serous neoplasm of low malignant potential
77. Which endometrial carcinoma is frequently associated with PTEN suppressor
gene mutation and histologically is characterized by a predominance of
glandular architecture resembling normal proliferative endometrium?
a. Clear cell carcinoma
b. High grade endometrioid carcinoma
c. Low grade endometrioid carcinoma
d. Serous carcinoma
e. Type 2 endometrial carcinoma
f.

Type 1 endometrial carcinoma

78. Although type 1 carcinoma is infrequent in patients with HNPCC, which type 1
tumor has been noted to occur in this subset ofpatients
a. Clear cell carcinoma
b. High grade endometrioid carcinoma
c. Low grade endometrioid carcinoma
d. Serous carcinoma
e. Type 2 endometrial carcinoma
f.

Type 1 endometrial carcinoma

79. Which type of endometrial carcinoma is frequently associated with HNPCC


a. Clear cell carcinoma
b. High grade endometrioid carcinoma
c. Low grade endometrioid carcinoma

d. Serous carcinoma
e. Type 2 endometrial carcinoma
f.

Type 1 endometrial carcinoma

80. Which lesion is circumscribed, most often occurs in vulva, is highlighted by


perivascular cuffs of epithelioid desmin-positive cells and is normally cured
with simple excision?
a. Angiomyofibroblastoma
b. Cellular angiofibroma
c. Deep (aggressive) angiomyxoma
d. Mammary-type myofibroblastoma
e. Spindle cell lipoma/pleomorphic lipoma
f.

Well-differentiated liposarcoma

81. Which lesion is probably related to spindle cell lipoma including sharing of 13q
and 16q chromosomal abnormalities?
a. Angiomyofibroblastoma
b. Cellular angiofibroma
c. Deep (aggressive) angiomyxoma
d. Mammary-type myofibroblastoma
e. Spindle cell lipoma/pleomorphic lipoma
f.

Well differentiated liposarcoma

82. Which poorly circumscribed myxoid lesion is typically large and apt to recur
with incomplete excision?
a. Angiomyofibroblastoma
b. Cellular angiofibroma
c. Deep (aggressive) angiomyxoma
d. Mammary-type myofibroblastoma
e. Spindle cell lipoma/pleomorphic lipoma
f.

Well differentiated liposarcoma

83. Which entity represents the most common type ofchronic villitis and is
associated with a recurrence risk of up to 20%?
a. Chronic villitis of unknown etiology
b. Chronic villitis 2ry to CMV infection
c. Chronic villitis 2ry to HSinfection
d. Chronic villitis 2ry to syphilis
e. Chronic villitis 2ry to Toxoplasma gondii infection
84. Which entity is characterized by the triad ofhypercellular villi, proliferative
fetal vascular changes with vascular obliteration and "onion-skinning" ofvessels

and chronic villitis?


a. Chronic villitis of unknown etiology
b. Chronic villitis 2ry to CMV
c. Chronic villitis 2ry to HS
d. Chronic villitis 2ry to syphilis
e. Chronic villitis 2ry to Toxoplasma infection
85. The presence of chronic villitis associated with plasma cells, villous necrosis, and
calcifications is most commonly associated with which entity
a. Chronic villitis of unknown etiology
b. Chronic villitis 2ry to CMV
c. Chronic villitis 2ry to HS
d. Chronic villitis 2ry to syphilis
e. Chronic villitis 2ry to Toxoplasma
86. The presence of chronic villitis associated with plasma cells, villous necrosis and
calcifications is most commonly associated with which entity
a. Chronic villitis of unknown etiology
b. Chronic villitis 2ry to CMV
c. Chronic villitis 2ry to HS
d. Chronic villitis 2ry to syphilis
e. Chronic villitis 2ry to toxoplasma
87. Elevated serum CA125 levels are diagnostic ofprimary ovarian carcinoma
a. True
b. False
88. Up to one-third ofpatients with metastatic colorectal adenocarcinoma involving
the ovary may present with an ovarian mass before recognition ofthe colonic

tumor
a. True
b. False
89. Immunohistochemical staining for CDX2 relaibly distinguished metastatic
colorectal adenocarcinoma from yolk sac tumor
a. True
b. False
90. Which entity occurs in women > 30 years of age and is the most common
ovaran tumor associated with estrogenic manifestations
a. Adult granulosa celltumor
b. Clear cell carcinoma
c. Juvenile granulosa cell tumor
d. Small cell carcinoma, hypercalcemic type
e. Yolk sac tumor
91. Which entity typically occurs in young women is characterized histologically by
follicle-like spaces and cells with scanty cytoplasm and has often spread
beyond the ovary at the time ofpresentation?
a. Adult granulosa cell tumor
b. Clear cell carcinoma
c. Juvenile granulosa cell tumor
d. Small cell carcinoma, hypercalcemic type
e. Yolk sac tumor
92. Which entity commonly occurs in women < 30 years of age may be associated
with estrogenic manifestations and is positive for inhibin?
a. Adult granulosa cell tumor
b. Clear cell carcinoma
c. Juvenile granulosa cell tumor
d. Small cell carcinoma, hypercalcemic type
e. Yolk sac tumor
93. Which tumor exhibits periglandular cuffing by malignant stromal cells?
a. Adenosarcoma
b. Cellular leiomyoma
c. Endometrial stromal sarcomas, low grade
d. Intravenous leiomyomatosis

e. Leiomyosarcoma
f.

Undifferentiated uterine sarcoma

g. Uterine tumor resembling ovarian sex cord tumor


94. Which tumor is composed ofmonotonous cells with round to oval nuclei,
contains blood vessels resembling spiral arterioles and typically exhibits a CD10
positive h-caldesmon and desmin negative?
a. Endometrial stromal sarcoma, low grade
95. Which is tumor is most likely to exhibit prominent staining for alpha-inhibin,
CD 99, and / or calretenin?
a. Uterine tumor resembling ovarian sex cord tumor
96. An epithelioid desmin + caldesmon + 3 cm well-circumscribed uterine tumor
with no necrosis no cytologic atypia and a mitotic count < 3 mitoses/10 hpfis
best classified as what type of tumor?
a. Atypical leiomyoma
b. Endometrial stromal nodule
c. Epithelioid leiomyoma
d. Epithelioid leiomyosarcoma
97. An epithelioid desmin + caldesmon + 10 cm well-circumscribed uterine tumor
with multifocal cytologic atypia discernible at low power and a mitotic count of
5 mitoses/ 10 hpfis best classified as what type oftumor?
a. Epithelioid leiomyosarcoma
98. An eoithelioid desmin + caldesmon + infiltrative uterine tumor with bland
cytomorphology and a mitotic count of 5 mitoses/ 10 hpfis best classified as

what type oftumor?


a. Epithelioid leiomyosarcoma
99. Which tumor can have an epithelioid component, but also expresses inhibin and
calretinin?
a. Adenosarcoma
b. Endometrial stromal sarcoma with sex-cord elements
c. Endometrioid adenocarcinoma with prominent spindle cell component
d. Immature teratoma
e. Malignant mixed mullerian tumor
f.

Mucinous tumor of ovary with mural nodule

g. Sertoli-leydig cell tumor with heterologous elements


100.

Which is a biphasic tumor that may have periglandular stromal cuffs

and polypoid intraglandular stromal projections?


a. Adenosarcoma
b. Endometrial stromal sarcoma with sex-cord elements
c. Endometrioid adenocarcinoma with prominent spindle cell component
d. Immature teratoma
e. Malignant mixed mullerian tumor
f.

Mucinous tumor of ovary with mural nodule

g. Sertoli-leydig cell tumor with heterologous elements


101. Which tumor occurs predominantly in young patients and is characterized by
tubule-like structures that stain positive for GFAP, NSE, and NF?
a. Adenosarcoma
b. Endometrial stromal sarcoma with sex cord like elements
c. Endometrioid adenocarcinoma with prominent spindle cell component
d. Immature teratoma
e. Malignant mixed mullerian tumor
f.

Mucinous tumor of ovary with mural nodule

g. Sertoli-leydig cell tumor with heterologous elements


102.

Which entity is most frequently associated with endometriosis?

a. Clear cell carcinoma ofthe ovary

b. Dysgerminoma
c. Endometrioid carcinoma secretory variant
d. Metastatic renal cell carcinoma
e. Yolk sac tumor
103.

Elevated serum AFP levels and positive staining for AFP within tumor

cells characterize which tumor?


a. Dysgerminoma
b. Clear cell carcinoma ofovary
c. Endometrioid carcinoma secretory variant
d. Metastatic renal cell carcinoma
e. Yolk sac tumor
104.

Which tumor is characterized by a papillary growth pattern with

prominent hyalinizaed stromal cores?


a. Dysgerminoma
b. Clear cell carcinoma ofovary
c. Endometrioid carcinoma secretory variant
d. Metastatic renal cell carcinoma
e. Yolk sac tumor
105.

Which ovarian neoplasm may show histologic evidence ofbile

production and is immunohistochemically positive for AFP?


a. Clear cell carcinoma, oxyphilic variant
b. Hepatoid carcinoma
c. Hepatoid yolk sac tumor
d. Steroid cell tumor
106.

Which ovarian neoplasm typically occurs in young women and is

immunohistochemically positive for AFP?


a. Clear cell carcinoma, oxyphilic variant

b. Hepatoid carcinoma
c. Hepatoid yolk sac tumor
d. Steroid cell tumor
107.

Which ovarian neoplasm is typically positive for inhibin and negative for

AFP?
a. Clear cell carcinoma, oxyphilic variant
b. Hepatoid carcinoma
c. Hepatoid yolk sac tumor
d. Steroid cell tumor
108.

Which tumor most commonly presents in older women as a polypoid

intra-uterine mass?
a. Embryonal RMS
b. Leiomyosarcoma
c. MMMT
d. Nodular fasciitis
e. Pleomorphic RMS
109.

Luteinized thecoma occurs more frequently in patients younger than

30 years ofage as compared to fibroma


a. True
b. False
110. Metastatic breast carcinoma and sclerosing stromal tumor can be distinguished
based on the presence or absence ofestrogen receptor expression.
a. True
b. False
111. Keratin and EMA expression helps to distinguish a Krukenberg tumr froma
sclerosing stromal tumor
a. True
b. False
112. Which tumor is characterized by the admixture ofadipocytes and smooth
muscle?
a. Lipoleiomyoma
113. Which tumor is characterized by cytologically atypical cells but infrequent

mitoses?
a. Atypical leiomyoma
114. Which tumor typically exhibits the presence ofcoagulative necrosis of
neoplastic cells and mitotic figures?
a. Leiomyosarcoma
115. Which entity is characterized by bland stratified typically non-mucin
containing glands in a fibromatous stroma that may be associated with
squamous morules
a. Endometrioid adenofibroma
116. Which entity may have solid nests ofepithelial cells that stain positively for
uroplakin IIIcystic cavities containing eosinophilic secretions and a fibromatous
stroma?
a. Brenner tumor
117. Which entity is frequently found in association with a Brenner tumor?
a. Mucinous cystadenoma
118. Which tumor most commonly contains heterologous elements?
a. MMMT
119. In which tumor is benign appearing squamous metaplasia most commonly
observed?
a. Adenosarcoma
b. MMMT
c. Endometrial stromal sarcoma, low grade
d. Endometrioid adenocarcinoma

e. Leiomyosarcoma
f.
120.

Serous carcinoma
Which entity is characterized by massive enlargement ofthe placenta,

diffuse villous edema and fetal hydrops?


a. Hydrops placentalis
b. Mesenchymal dysplasia
c. Metabolic storage disorder
d. Partial hydatidiform mole
121. Which entity is characterized by marked placental enlargement chorionic
vascular abnormalities and cystic villous edema with approximately 50% of
cases also having associated findings suggestive ofBeckwith-Wiedemann
syndrome?
a. Mesenchymal dysplasia
122.

Which entity may be caused by Parvovirus B19 infection?

a. Hydrops placentalis
123.

Which entity is characterized by chronic inflammation including plasma

cells in the villi and may recur in subsequent pregnancies?


a. Villitis ofunknown etiology
124.

CMV, rubella, HSV, Parvovirus B19, Treponema pallidum, Listeria

monocytogenes, and E. coli may be responsible for which entity?


a. Acute villitis
125.

Which entity is the most common form ofvillitis?

a. Villitis ofunknown etiology


126.

Which tumor most commonly occurs in post menopausal women has a

poor prognosis and demonstrates epithelail and mesenchymal differentiation?


a. MMMT

127.

Which entity typically occurs in middle-aged women comprises

approximately one third ofovarian neoplasms and is characterized by papillary


fronds covered by stratified epithelial cells with scant cytoplasm mild to
moderate nuclear atypia inconspicuous cytoplasmic mucin or mucin localized
only to the tips of the cells and abundant psammoma bodies?
a. Serous tumor of borderline malignancy
128.

Which entity typically occurs in combination with anovarian germ cell

tumor exhibits a hemorrhagic cut surface is biphasic and has the following
immunoprofile: CK + and hCG+
a. Choriocarcinoma
129.

Which tumor is characterized by a microfollicular pattern and Call-

Exner bodies?
a. Granulosa cell tumor
130.

Which tumor is composed predominantely ofmature gastrointestinal

epithelium glial tissue and respiratory epithelium with a microscopic focus of


thyroid tissue?
a. Mature teratoma
131. Which tumor is characterized by a predominantely follicular pattern containing
intraluminal eosinophilic material a secondary intermixed trabecular ad insular
component and immunoreactivity for thyroglobulin and synaptophysin?
a. Strumal carcinoid
132.

Which is the most important factor predicting survival in patients with

borderline mucinous tumor ofovary?


a. Tumor stage
133.

Which one of the following statements about primary mucinous tumors

in the ovary is true?


a. At the time ofdiagnosis, they are bilateral in 30-40% ofcases

b. Malignant mucinous tumors are more common than borderline mucinous


tumors
c. Mucinous tumors of the ovary are the most common cause of
pseudomyxoma peritonei
d. In the ovary, primary mucinous carcinoma is less common than metastatic
mucinous carcinoma
e. Most borderline mucinous tumors are composed of endocervical-like cells
134.

In pathologic staging ofborderline and malignant ovarian epithelial

tumors by the TNM/FIGO classification, which ofthe following is true?


a. pT1 tumors involve only one ovary
b. the presence of malignant cells in ascites or peritonela washings means
that a tumor is at least pT2
c. the presence of pelvic extension or implants without more distant disease
places a tumor in the pT2 category
d. the presence of pleural effusion indicates that the tumor is pM1
e. regional lymph node metastasis (pN1) is required to classify a tumor as
stage III
135.

which entity occurring in middle-aged women may be associated with

ascites and pleural effusion (Meig's syndrome)?


a. Fibroma
136.

Which entity typically occurs in children, teens, and young adults is

commonly composed of sheets ofcells having rounded, hyperchromatic nuclei,

occasional variably-sized follicles containing eosinophilic or basophilic material,


numerous leutenized cells, is inhibin + , alpha-fetoprotein -, and hCG -?
a. Juvenile granulosa cell tumor
137.

Which entity is typically yellow, solid and lobulated, composed of

uniform, uninucleate tumor cells with a central, large, round nucleus containing
one or more prominent nucleoli, brisk mitotic activity, occasionally has
granulomatous inflammation and rarely syncitiotrophoblastic cells?
a. Dysgerminoma
138.

Which is the most common sex cord-stromal tumor ofthe ovary?

a. Fibroma
139.

Which ovarian sex cord-stromal tumor is most likely to metastasize?

a. Adult granulosa cell tumor


140.

Which primary ovarian neoplasm occurs in young adult women?

a. Sclerosing stromal tumor


141. Which entity occurring in older women is characterized by predominantly solid,
white to yellow firm mass with dense fibrous bands and occasional calcification?
a. Fibrothecoma
142.

Which entity typically exhibits a solid and cystic cut surface, eosinophilic

intracellular and extracellular hyaline bodies, Schiller-Duval bodies, and alpha


fetoprotein expression?
a. Yolk sac tumor
143.

Which entity is typically solid and cystic with variably sized follicles lined

by cells with non-grooved euchromatic nuclei and eosinophilic cytoplasm may

have luminal eosinophilic mucicarcimophilic fluid within follicles and is inhibin


positive and negative for alpha fetoprotein and hCG?
a. Juvenile granulosa cell tumor
144.

Which lesion is characterized by the finding ofstellate and

multinucleated stromal cells near the epithelial stromal interface?


a. Fibroepithelail stromal polyp
145.

Which tumor is composed ofspindled cells with moderately abundant

eosinophilic cytoplasm and centrally placed elongated nuclei with blunt ends,
shows a mitotic count greater than 5 per 10 hpf, is positive for actin, desmin,
and h caldesmon and negative for CD10?
a. Leiomyosarcoma
146.

Which entity is a low grade malignancy showing indolent behavior with

potential for very late recurrences?


a. Adult granulosa cell tumor
147.

Which is the most frequent benign uterine tumor in women?

a. Leiomyoma
148.

A tamoxifen induced endometrial polyp typically has small to irregular

staghorn shaped glands proliferating parallel to the axis of the polyp and focal
periglandular stromal condensations
a. True
b. False
149.

Which uterine lesion is typically found in the clinical setting ofdiabetes

hypertension and obesity?


a. Endometrial hyperplasia

150.

Which is the most common inhibin positive tumor

a. Adult granulosa cell tumor


151. Squamous differentiation would be most commonly encountered in which
tumor?
a. Endometrioid carcinoma
152.

Which tumor is most likely to show bilateral ovarian involvement?

a. Metastatic carcinoma
153.

Which one of the following is not a characteristic feature ofserous

borderline cystadenofibroma
a. A 5 mm or greater confluent area ofmicropapillary growth
154.

Most surface epithelial tumors ofthe ovary are CK 7, EMA and

estrogen and progesterone receptor positive


a. True
b. False
155.

The most common surface epithelial tumor of the ovary is

a. Benign serous cystadenoma


156.

Immunopositive for panCK with high Ki67 index (> 50%), negative for

inhibin and CK18, negative serum hCG


a. Squamous cell carcinoma
157.

Microscopic lesion with low cellularity hyalinized stroma and ki67 index

less than 10% immunopositive for inhibin, CK18, focally positive for hPL and
CD146
a. Placental site nodule
158.

Immunopositive for inhibin, panCK, CK18, diffusely positive for hPL, Ki67

index < 5%, low elevation ofserum hCG


a. Placental site trophoblastic tumor
159.

Occurs in younger patients and is composed oflarge, pleomorphic cells

that line glands and papillae and grow as solid sheets. Tumor cells are
occasionally immunoreactive for AFP and may show hCG and associated
syncitiotrophoblastic cells

a. Embryonal carcinoma
160.

What is the commonly used diagnostic label to describe the infant

accompanied by congenital listeriosis


a. Granulomatous infantisepticum
161. An expected maternal history with congenital listeriosis
a. Ingestion ofpackaged meat or cheese
162.

Which is the most appropriate comment?

a. Th causative organism in congenital listeriosis is the most cause ofneonatal


sepsis
b. The placental changes (intervillous fibrin deposition, intervillous
microabscesses, basophilic aggregates of microorganisms in the inflamed
membranes) suggest that the infant should have been treated with
acyclovir
c. The mother's second untoward pregnancy outcome is characteristic of
congenital listeriosis
d. Successful detection of this condition by placental exam may direct
therapy
e. None ofthe above
163.

Which tumor lacks neuroectodermal tissue and is composed of

mullerian type carcinoma and mesenchymal components


a. Malignant mesodermal mixed tumor
164.

Which tumor is predominantly composed ofprimitive neuroectodermal

tissue with intestinal type glands and immature cartilage?


a. Immature teratoma
165.

A cystic lesion lined entirely or partly by epithelium resembling the

epidermis with its appendages and containing mesodermal endodermal and


ectodermal elements
a. Mature cystic teratoma
166.

Which lesion commonly shows immunohistochemical expression of

epithelial markers in malignant stromal cells?

a. Malignant mesodermal mixed tumor


167.

Which lesion is characterized by the presence ofnumerous small

arteries resembling those oflate secretory type endometrium?


a. Endometrioid stromal sarcoma with sex-cord like differentiation
168.

Typical features of aggressive angiomyxoma include all ofthe following

except?
a. Infiltrative growth pattern
b. High rate of local recurrence
c. High rate of metastasis
d. Origin in pelvis and peritoneum
e. Predilection for females
169.

A sharply circumscribed mass with a predilection for the perineum no

propensity for recurrence and plump epithelioid tumor cells with a


predominantly perivascular arrangement are features of
a. Angiomyofibroblastoma
170.

Helpful microscopic features in the differential diagnosis ofpelvic

myxoid masses include all ofthe following except?


a. An arborizing vascular pattern
b. A nodular growth pattern
c. Mature adipose tissue within the mass
d. Multinucleated cells within the tumor
e. Cytologic atypia and pleomorphism
171. Which of the following is not characteristic ofatypical polypoid adenomyoma
(ofMazur)
a. Proliferative or hyperplastic endometrial glands with metaplsatic changes
b. May be misinterpreted as adenocarcinoma with myometrial invasion on
curettage specimen

c. Multiple dilated cysts with intracystic polypoid projections and periglandular


stromal condensation
d. Stromal smooth muscle component may be quite prominent
e. Stroma may contain occasional mitoses
172.

Which ofthe following features is not found in low grade endometrial

stromal sarcoma?
a. Lymphovsascular invasion
b. Typically contains less than 3 mitoses/10 hpf
c. Viable glands adjacent to necrotic debris
d. Nodular or diffuse permeation ofmyometrium
e. Minimal nuclear pleomorphism
173.

Adenosarcoma may show all ofthe following characteristics except

a. Cystic spaces with intraluminal projections


b. Biphasic tumor with glandular and stromal components
c. Cartilaginous and rhabdomyoblastic stromal differentiation
d. Periglandular stromal condensation
e. Syncitiotrophoblastic stromal cells
174.

Which single feature accurately predicts the behavior ofa uterine

smooth muscle tumor?


a. Intravenous growth
b. Mitotic activity
c. Tumor size
d. Tumor necrosis
e. Cellular atypia
f.

None ofthe above

175.

Which special stain accurately distinguishes endometrial stromal cells

from myometrial cells


a. CK
b. Desmin
c. SMA
d. Vimentin
e. Reticulin
f.
176.

None ofthe above


Which feature is ofleast importance in evaluating an epithelioid smooth

muscle tumor?
a. Cellular atypia
b. Necrosis
c. Size
d. Mitotic activity
177.

Which ofthe following features favors primary ovarian carcinoma over

metastatic colonic adenocarcinoma?


a. CK 20 + , CK 7b. CEA +
c. Normal serum CA125
d. Cribriform architecture with dirty necrosis
e. Presence ofsquamous cell differentiation
178.

Endometrioid carcinoma of the ovary may show all ofthe following

characteristics except
a. Squamous differentiation
b. Extracellular mucin

c. High grade micropapillae and numerous psammoma bodies


d. Association with endometriosis
e. Association with a synchronous endometrial primary
179.

Which lesion is congenital in origin and can present as an enlarging

mass in the labia?


a. Ectopic breast tissue
180.

Which lesion is located in the lower midline abdomen and regarded as a

developmental malformation?
a. Urachal cyst
181. Which lesion can manifest stromal decidual change?
a. Endometriosis
182.

Which lesion is composed oftransitional cells with pale grooved nuclei

and may contain scattered benign mucinous glands


a. Brenner tumor
183.

Which lesion may have sarcoma-like mural nodule?

a. Mucinous cystadenoma
184.

Calcification in this ovarian tumor characteristically occurs in the form

ofstromal plaques?
a. Brenner tumor
185.

Which ofthe following statements about ovarian transitional cell tumors

is correct?
a. The epithelial cells are ultrastructurally identical to urothelial cells of the
urinary tract

b. Squamous differentiation may be extensive in the tumor


c. The epithelial cells are typically CK7+ and CK20+
d. Malignant Brenner tumor contains a component of benign Brenner tumor
or Brenner tumor oflow malignant potential
e. Up to 20% ofthe epithelial cells in a transitional tumor may demonstrate
columnar mucinous differentiation
186.

Which ofthe following statements about ovarian tumors is correct?

a. Insular granulosa cell tumor is commonly admixed with other histologic


patterns ausch as microfollicular or gyriform
b. The presence of neuroendocrine granules in the cytoplasm oftumor cells
excludes the diagnosis ofan ovarian surface epithelial cell tumor
c. Mucinous adenofibroma often produces an estrogenic clinical syndrome
d. Primary squamous cell carcinoma most commonly arises in a Brenner
tumor
e. Stromal luteinization is common in ovarian adenofibromas
187.

Child or young adult, large size, right side predominance, acute

abdomen, preserved ovarian structure


a. Massive edema
188.

Reproductive age patient, asymptomatic mass, low cellularity, colloidal

iron rich stroma, effaced architecture


a. Myxoma
189.

Young adult, menstrual irregularity, large size, pseudolobular histology,

hemangiopericytoma-like vasculature
a. Sclerosing stromal tumor ofthe ovary
190.

Which characteristically produces lymphoplasmacytic villitis with villous

fibrosis, karyorrhexis and hemosiderin deposition?


a. CMV
191. Infection with which organism shows characteristic intranuclear viral inclusions
in fetal red blood cells?
a. Parvovirus
192.

This agent characteristically infects the extraplacental membranes

without eliciting an inflammatory or desmoplastic reaction

a. Toxoplasma gondii
193.

Histologic pattern associated with treponemal infection

a. Lymphoplasmacytic villitis
194.

Lesion associated with triploid karyotype

a. Partial hydatidiform mole


195.

Lesion characterized by villous hypervascularity and clinical diabetes

and preeclampsia
a. Chorangiosis
196.

Which lesion characteristically contains crystalloids ofReinke?

a. Leydig cell tumor


197.

Which lesion is most commonly bilateral and multicystic associated with

gestational trophoblastic tumors and frequently involutes following treatment?


a. Hyperreactio luteinalis
198.

Which lesion is typically solid, white and fibrous on gross examination?

a. Fibroma
199.

The most common primary site for a krukenberg tumor is

a. Stomach

200.

Which ofthe following statements about clear cell carcinoma of the

ovary is true
a. Cytoplasm is glycogen rich
b. Mucin is present in tubules and cysts
c. It may contain hyaline bodies
d. Immunoreactivity for alpha fetoprotein may be present
e. All of the above
201.

Which tumor is positive for CK and parathyroidhormone-related

protein but negative for LCA


a. Small cell carcinoma ofthe ovary
202.

Which ofthe following statements about borderline mucinous tumor is

true?
a. Bilateral ovarian involvement is seen in the majority of patients
b. It accounts for the majority of ovarian mucinous tumors
c. The tumor cell resemble endometrial cells
d. Destructive stromal invasion can be present
e. None ofthe above
203.

Foci ofinvasion smaller than 1 mm may be seen in borderline mucinous

tumors
a. True
b. False
204.

Which lesion is the most frequent benign tumor of the fallopian tube?

a. Adenomatoid tumor
205.

Which lesion consists ofa circumscribed nodular mass ofbenign

smooth muscle with interspersed islands of benign endoterial glands and

stroma?
a. Adenomyoma
206.

Which ofthe following ovarian neoplasms would be least likely to

exhibit immunoreactivity for inhibin?


a. Endometrioid adenocarcinoma
207.

Which ofthe following is most common ovarian sex cord stromal tumor?

a. Granulosa cell tumor


208.

A large malignant uterine neoplasm typically containing cartilage

a. Malignant mixed mullerian tumor carcinosarcoma, heterologous type


209.

Squamous cell carcinoma is the most common histologic component in

this neoplasm
a. Teratoma with malignant transformation
210.

Which entity is characterized by diffusely enlarged often cavitated villi

with concentric trophoblastic proliferation and is associated with the greatest


risk for persistent trophoblastic disease?
a. Complete hydatidiform mole
211. Which entity is associated with a lack ofimmunostaining for the paternally
imprinted maternally expressed gene product p57 in villous mesenchymal cells
and villous cytotrophoblast?
a. Complete hydatidiform mole
212.

Which entity can be associated with modestly increased beta HCG

levels a normal uterine ultrasound an admixture ofnormal and hydropic villi


concentric syncitiotrophoblastic hyperplasia and fetal syndactyly?
a. Partial hydatidiform mole
213.

All complete hydatidifrom moles are diploid

a. True
b. False
214.

Staining ofintervillous intermediate trophoblasts for p57 rules out a

complete mole
a. True

b. False
215.

Early complete moles do not have a significant risk for development of

persistent gestational trophoblastic disease


a. True
b. False
216.

Dysgerminoma is almost always part of a mixed germ cell tumor.

a. True
b. False
217.

Which adenocarcinoma within an ovary is most commonly composed of

confluent necrosis and has an immunoprofile ofCK 7 -, CK 20 + , ER -, PR -?


a. Metastatic colorectal adenocarcinoma
218.

Which tumor generally stains positive for calretinin?

a. Sertoli-Leydig cell tumor


219.

Which tumor exhibits similar molecular abnormalities as those seen in

the endometrial cavity (i.e. PTEN, beta catenin, and KRAS mutations, and
microsatellite instability)?
a. Endometrioid adenocarcinoma
220.

Which ofthe following is true regarding adult granulosa cell tumor?

a. Nuclear grooves are a specific feature limited to AGCT and never found in
cellular fibromas
b. Reticulin staining demonstrating distinct fibers surrounding large groups of
cells helps distinguish AGCT from cellular fibromas
c. There is often a fibromatous background in AGCT and at times the
granulosa cells can be a minor component making up less than 10% ofthe
tumor
d. A positive pan CK stain always distinguishes undifferentiated carcinoma
from AGCT
221.

Which immunohistochemical profile is compatible with an AGCT?

a. PanCK+ , EMA -, calretinin + , WT1+


222.
AGCT?

Which ofthe following features is an important prognostic factor for

a. Stage
223.

Which ovarian neoplasm is typically composed of well- formed hollow

or solid tubules lined by cuboidal to columnar cells with moderate to abundant


eosinophilic or vacuolated cytoplasm and round to oval regular nuclei with
nuclear grooves?
a. Sertoli cell tumor
224.

Which ovarian neoplasm is typically positive for CK7 but negative for

inhibin?
a. Clear cell carcinoma ofthe ovary
225.

Which is most likely to simultaneously involve both uterus and ovary?

a. Endometrioid stromal sarcoma


226.

Which is most often associated with virilizing symptoms?

a. Sertoli-Leydig cell tumor

16. Breast
1.

High grade phylloides tumor shows which mutation


a. P53

2. ER is strongly expressed in which breast tumor


a. Ductal
b. Lobular
c. Medullary
d. Apocrine
3. P53 overexpression is seen in which breast tumor
a. Basal-like
b. Luminal A
c. Luminal B
d. Unclassified
e. Her-2
4. Loss of cell adhesion with diffuse infiltration and signet ring appearance ofthe
cells
a. Loss of E-cadherin
b. APC
5. Female with mammogram showing irregular mass and calcification, foamy cells
are seen on microscopy. What is the most likely cause
a. Trauma
6. Which of the following tumor is the most common type ofbreast cancer in
young females
a. Secretory cancer
b. Cribriform
c. Apocrine
d. Ductal

7. 70 year old female with first time diagnosis of breast cancer and no family
history, which of the following mutations is present
a. P53
b. BRCA1
c. BRCA2
d. PTEN
e. ATM
8. What is the tumor stage for a patient with invasive ductal carcinoma ofthe
breast with infiltration ofthe dermal lymphatics
a. T1
b. T2
c. T3
d. T4d
9. 16 year old girl with breast lesion showing nodules with stromal and ductal
proliferation and has adipose tissue in the stroma
a. Hamartoma
b. Sclerosing adenosis
c. Fibroadeonoma
d. Tubular adenoma
e. Fibrocystic disease
10. Intraductal papilloma has all ofthe following EXCEPT
a. Nipple retraction
b. Bloody nipple discharge
c. Growth in lactiferous sinuses
d. Multiple lesions
e. Can be less than 1 cm and difficult to locate
11. Which of the following predicts the behavior ofmalignant phylloides tumor
a. Tumor diameter
b. Ki67 index
c. Stromal overgrowth
12. In staging multifocal invasive mammary carcinoma
a. Stage is based upon the largest linear dimensions of the largest discrete
focus ofcarcinoma

13. When examining core biopsies from breasts with suspicion for
microcalcification and the initial 2 levels didn't show the calcification
a. Perform X-ray and ask for deeper levels
b. Write the report as you see
c. Take deeper sections
14. Paget's disease of the nipple
a. Doesn
t change the stage
b. CISis a pre-requisite
c. T4 stage
d. Lobular carcinoma in situ
15. Bloody nipple discharge without a mass
a. Intraductal papilloma
16. A patient with a breast mass which is E-cadherin negative, which is true
a. May have a lesion in the opposite breast
17. Frozen section in breast is for
a. Assessing the margins
b. Papillary breast tumor
c. Unpalpable mass excision
d. Assessing SBR grade
18. A 35 year old female has an irregular breast mass, USshowed
microcalcification, biopsy showed foamy macrophages, what is the diagnosis
a. Trauma (fat necrosis)
b. DCIS
c. Invasive ductal carcinoma
19. What is the histopathology ofdiabetic mastopathy
a. Increased collagen and fibroblasts

b. Fibrosis with epithelioid fibroblast


20. Regarding the staging ofbreast carcinoma
a. Ifthere are invasive and in-situ components only the size ofthe invasive
component is used
21. Which tumor shows positivity for SMA, desmin, CD34, and Bcl-2
a. Angiosarcoma
b. Fibroadenoma
c. PASH
22. Which of the following choices best allows one to discriminate between
juvenile fibroadenoma and low grade phyllodes tumor
a. The rate of growth and assessment ofa peri- versus intra-canilicular
growth pattern
b. Type ofmargin
c. Degree ofmitotic activity
d. Gross size
e. Sex of the patient
23. High grade phyllodes tumors have been linked to mutations in which of the
following?
a. P53
b. BRAF
c. N-myc

d. P16
e. BRCA1
24. The most common clinical problem associated with low grade phyllodes tumors
is
a. High risk of distant metastasis
b. Chest wall invasion
c. Local recurrence after excision
d. Incidence ofhypercalcemia (paraneoplastic syndrome)
e. Dedifferentiation to high grade phyllodes tumor
25. Which one ofthe following is NOT a helpful feature in distinguishing benign
phyllodes tumor from malignant.
a. High mitotic index
b. Infiltrative margins
c. Sarcomatous growth
d. Squamous differentiation
e. Stromal overgrowth
26. Metaplastic carcinoma can be reliably distinguished from phyllodes tumor by
which one ofthe following features
a. Atypical spindled cells
b. Cytokeratin positive spindled cells
c. Epithelial ducts interspersed within spindled cells
d. High mitotic index
e. Infiltrative margins

27. Cartilaginous elements are present within a vulvar mass with prominent
stromal elements and a leaf-like architecture. Which ofthe following is correct
a. Phyllodes tumors can be present anywhere in the milk-line and can have
heterologous elements such as cartilage
b. This cannot be phyllodes tumor, since these tumors are restricted to
mammary/breast parenchyma only
c. This is a vulvar teratoma
d. This may represent a chondroma or chondrosarcoma
28. The presence of small foci of conventional infiltrating ductal carcinoma or
ductal carcinoma in situ best supports which diagnosis?
a. Angiosarcoma
b. Malignant phyllodes tumor
c. Melanoma
d. Metaplastic carcinoma
e. Undifferentiated pleomorphic sarcoma
29. Which tumor displays a leaf-like architecture and a biphasic growth pattern?
a. Angiosarcoma
b. Malignant phyllodes tumor
c. Melanoma
d. Metaplastic carcinoma
e. Undifferentiated pleomorphic sarcoma
30. Which tumor occurs in a chronically lymphedematous arm following axillary
node dissection?
a. Angiosarcoma
b. Malignant phyllodes tumor
c. Melanoma
d. Metaplastic carcinoma
e. Undifferentiated pleomorphic sarcoma
31. High grade phyllodes tumor can be distinguished histologically from metaplastic
carcinoma and pure sarcomas ofbreast by the presence ofa phyllodes-like

pattern in most cases is adequately sampled


a. True
b. False
32. Metaplastic carcinoma often exhibits a basal-like immunophenotype
a. True
b. False
33. Pure sarcomas of the breast can be reliably distinguished from metaplastic
carcinoma by the presence or absence or immunoreactivity for CK
a. True
b. False
34. A constellation of rapidly evolving clinical manifestations to include reddened
indurated skin and swelling is the sine qua non ofwhich mammary neoplasm?
a. Inflammatory breast cancer
35. The spindle cell component ofwhich tumor commonly expresses one or more
CK a useful diagnostic feature?
a. Metaplastic carcinoma
36. Which neoplasm frequently has the pathologic finding ofdermal lymphatic
tumor emboli?
a. Inflammatory breast cancer
37. Which fibroepithelial tumor has a small but definite potential to metastasize
a. Phyllodes tumor
38. Which entity is a malformation, not a neoplasm?
a. Hamartoma
39. The spindle cell component ofthis biphasic tumor commonly expresses one or
more CK a useful diagnostic feature
a. Metaplastic carcinoma
40. A poorly circumscribed spindle cell lesion that infiltrates breast parenchyma
a. Fibromatosis
41. A well-circumscribed breast lesion with S100 positive spindle cells

a. Adenomyoepithelioma
42. A well-circumscribed breast lesion that contains breast lobules and a stromal
component offat fibrous tissue or smooth muscle
a. Hamartoma
43. Pure invasive micropapillary carcinoma is frequently associated with lymph
node metastases
a. True
b. False
44. The distinct histologic features ofinvasive micropapillary carcinoma include all
ofthe following except
a. Spaces surrounding micropapillary structures
b. Vascular papillary fronds
c. Intermediate or high nuclear grade
d. Frequent tumor emboli
e. Numerous mitoses
45. The most common immunophenotype associated with micropapillary
carcinoma is
a. ER+ , PR + , Her2 +
46. All of the following affect the pathologic staging of an invasive breast tumor
except?
a. Tumor size
b. Lymph node status
c. Extranodal extension
d. Dermal invasion
e. Chest wall extension
47. Which invasive breast tumor has the worst prognosis?
a. Classic lobular carcinoma

b. Metaplastic carcinoma
c. Ductal carcinoma, NOS
d. Tubular carcinoma
e. Mucinous carcinoma
48. All of the following have been clinically validated as predictive and/or
prognostic factors in breast cancer except?
a. Her 2 neu
b. Bcl 2
c. Er
d. Pr
e. Tumor size
49. Histologic grade is an accurate predictor ofoutcome for high grade
undifferentiated carcinoma
a. True
b. False
50. All of the following are accepted criteria for classic medullary carcinoma ofthe
breast, except
a. Prominent syncytial growth pattern
b. Intense lymphoplasmacytic infiltrate
c. High grade nuclei

d. Tumor necrosis
e. Circumscribed margins
51. The presence of ductal carcinoma in situ rules out medullary carcinoma ofthe
breast
a. True
b. False
52. All of the following have proven clinical utility as prognostic factors except
a. Tumor size
b. Tumor grade
c. Tumor necrosis
d. Lymph node status
e. Hormone receptor status
53. All of the following are classified as special type carcinomas with regard to
prognosis
a. Classic infiltrating lobular carcinoma
b. Apocrine carcinoma
c. Mucinous carcinoma
d. Tubular carcinoma
e. Cribriform carcinoma
54. Which predictive factor has been endorsed by the tumor panel ofthe
American society of clinical oncology for routine clinical use in breast cancer
a. Estrogen receptor
55. Which lesion is typically non-palpable, may be bilateral and mutlicentric and
characteristically is composed ofcells that are noncohesive monotonous with

round cytologically bland nuclei, absent nucleoli and scant cytoplasm that fill
and expand acinar spaces?
a. Lobular carcinoma in situ
56. Which lesion is nonplabale and shows an increase in the cellularity of the duct
epithelium that is characterized by a mixed cell population with a streaming
appearance in which the nuclei are oriented parallel to the long axis ofthe
cells with nuclear crowding?
a. Ductal hyperplasia
57. Which lesion is frequently nonpalpable and characterized by an epithelial
proliferation that fills the duct with neoplastic cells of a single cell type with
well defined cell borders and may show bridges and punched out spaces?
a. Ductal carcinoma in situ
58. Which most often demonstrates a sinus catarrh pattern ofmetastasis to
axillary lymph node?
a. Infiltrating lobular carcinoma
59. Which of the following typically shows reactivity with antibodies against SMA,
desmin, CD34, and Bcl-2?
a. Angiosarcoma
b. Fibroadenoma
c. PASH
d. Sclerosing adenosis
60. Which of the following is more typical ofPASH than low grade angiosarcoma?
a. A positive reaction to CD31 and anti-factor VIIIantibodies
b. A tendency to infiltrate individual adipocytes
c. Marked nuclear pleomorphism
d. Slit-like lumina devoid oferythrocytes
61. IfPASH is diagnosed on core biopsy then the lesion is
a. May be followed clinically ifthe patient so desires

b. Should be treated with lumpectomy and a sentinel lymph node biopsy


c. Should be treated with neoadjuvent chemotherapy before excision
d. The diagnosis should be questioned because PASHcannot be reliable
diagnosed on core biopsy
62. Which invasive lobular carcinoma variant has poor prognosis?
a. Pleomorphic
63. The most typical immunohistochemical profile for classical invasive lobular
carcinoma is:
a. ER + , PR + . HER2 -. E-caddherin
64. Which of the following statements about the prognosis of invasive lobular
carcinoma is true?
a. All of the invasive lobular variants have essentially the same prognosis
b. E-cadherin negative lobular tumors have a worse prognosis than Ecadherin positive tumors
c. In lobular tumors, only the nuclear grade has prognostic significance
d. The Notingham combined histologic grade has prognostic significance in
invasive lobular tumors
e. With long term follow-up (10 years or more) invasive lobular tumors show
a better overall survival than invasive ductal tumors
65. Pathologic stage of mammary carcinoma is based upon
a. Maximum dimension of invasive component only

66. In staging multifocal invasive mammary carcinoma


a. Stage is based upon the largest linear dimension of the largest discrete
focus ofcarcinoma
67. A poorly differentiated breast mass is CK7 -, AE1/AE3 -, S100 + , HMB45 + , and
Melan A+ . A likely diagnosis is
a. Melanoma

17. Lymph Nodes and Spleen


1.

Progressively transformed germinal centers can be seen in association with


a. Nodular lymphocyte predominant Hodgkin lymphoma

2. What is the translocation associated with MALT lymphoma


a. T(11;18)
3. 6 year old with abdominal mass positive for CD20, CD43, CD10, Bcl6 and
negative for Bcl2, which translocation will be found
a. T(8;14) [Burkitt lymphoma]
b. T(2;4)
c. T(11;14)
d. T(14;18)
4. Langerhan's cell markers
a. CD1a and S100
5. Which lymphoma commonly occurs in immunocompromised patients
a. Diffuse large B cell lymphoma
b. Follicular lymphoma
c. Hodgkin lymphoma
6. Lymphoma which is negative for CD5, CD23, CD10, and cyclin-D1
a. MALT lymphoma (marginal zone)
b. Burkitt lymphoma
c. SLL
d. Follicular lymphoma
e. Mantle cell lymphoma
7. Lymphoma which is positive for CD5 and CD23
a. CLL/SLL
8. Lymphoma which is positive for CD5 and cyclin-D1

a. Mantle cell lymphoma


9. Tumor which is positive for CD99, CD43, CD34, and CD79a
a. Lymphoblastic lymphoma
b. Ewing's sarcoma
10. Lymph node showing monocytoid B-cells and reactive follicular hyperplasia
with epithelioid histiocytic infiltration
a. HIV
b. Reactive lymph node
c. Cat scratch disease
d. Toxoplasma infection
11. Which of the following lymphomas involve the red pulp and spare the white
pulp when they metastasize to the spleen
a. Hairy cell leukemia
b. Mantel cell
c. B cell lymphoma
d. Marginal zone
12. Oriental patient with peripheral eosinophilia, high IgE, recurrent chronic
inflammatory process, and reactive lymph nodes, what is the diagnosis
a. Kimura disease
13. Hematolymphoid neoplasm classification through
a. WHO 2008

b. WHO 2010
14. A baby with skull lesion composed ofspindle cells which are positive for CD117
and with prominent eosinophils
a. Histocytosis
b. Systemic mastocytosis
15. Which tumor is positive for CD1a and S100 and has nuclear grooves
a. Langerhan's cell histiocytosis
16. Which lymphoma is diagnosed by peripheral lymphocytosis and
immunophenotyping
a. CLL/SLL
17. T(14;18)
a. Follicular lymphoma (prevention of apoptosis)
18. How can you distinguish between follicular hyperplasia and follicular lymphoma
a. Bcl 2
19. Which lymphoma has the highest Ki67 index (almost 100%)
a. Burkitt lymphoma
20. Which lymphoma is CD10 and Bcl2 positive
a. Follicular lymphoma
21. Which of the following can be diagnosed by a peripheral blood smear
a. CML
22. T(11;14)
a. Mantle cell lymphoma
23. Nodular appearance of a lymph node with scattered malignant cells that are
CD45 ve, CD20 -ve and CD30 + ve
a. Classical Hodgkin lymphoma
b. Diffuse large B cell lymphoma
24. Which of the following does NOT metastasize to lymph nodes
a. Osteosarcoma

b. Leiomyosarcoma
c. Rhabdomyosarcoma
d. Epithelioid sarcoma
e. Synovial sarcoma
25. T(2;5)
a. Anaplastic large cell lymphoma
26. Young man with abscess in axillary lymph node showing central or stellate
necrosis with neutrophils and palisading histiocytes
a. Cat-scratch disease
27. Which of the following is a CD 45 ve B cell malignancy
a. Hodgkin lymphoma
28. Enteropathy-related T cell lymphoma is a complication of
a. Celiac disease
b. Crhon's
c. Whipple
29. Lymphoma showing positivity for CD45 and CD30, and negativity for CD20
a. Anaplastic large cell lymphoma
30. Regarding post-transplant lymphoproliferative disease, all are true except
a. Early lesion is EBV ve
b. It depends on the type of transplant
c. It regresses after stopping immunosuppression
d. Associated with EBV infection
31. Most common site ofcat-scratch disease
a. Lymph node

32. Which is true about lacunar RS cells


a. The space is a fixation artifact
33. Which of the following immunophenotypic findings is most typical of mantle
cell lymphoma?
a. CD5+ , CD19+ , CD20+ , CD23-, igM+
34. Which of the following chromosomal translocations leading to overexpression
ofcyclin D1 is associated with mantle cell lymphoma?
a. T(8;14)
b. T(11;14)
c. T(14;18)
d. T(3;14)
e. T(2;5)
35. Which of the following statements concerning splenic involvement by B-cell
lymphomas is true
a. All B cell lymphomas involving the spleen are characteristically positive for
CD23
b. Hairy cell leukemia/lymphoma ofthe spleen is an aggressive B cell
lymphoma
c. Mantle cell lymphomas are typically distinguished from chronic lymphocytic
leukemias/small lymphocytic lymphoma by the absence of CD23 reactivity
d. Splenic marginal zone lymphoma are characterized by a recurring t(8;14)
e. Splenic marginal zone lymphoma may be discriminated from hairy cell
leukemia/lymphoma involvement ofthe spleen by the presence ofCD123
and CD11c in SMZL tumor cells
36. Which of the following lesions is characterized by sclerotic germinal centers,
lollipop lesions, and onion-skinning ofmantle zones
a. Follicular hyperplasia
b. Follicular lymphoma
c. Hyaline vascular variant ofCastleman disease
d. Mantle zone lymphoma
e. Plasma cell variant ofCastleman disease
f.

Thymoma

37. Which of the following lesions is characterized by an interfollicular expansion


by large sheets ofmature-appearing plasma cells

a. Follicular hyperplasia
b. Follicular lymphoma
c. Hyaline vascular variant ofCastleman disease
d. Mantle cell lymphoma
e. Plasma cell variant ofCastleman disease
f.

Thymoma

38. Which of the following is characterized by an atypical lymphoid infiltrate that


effaces lymph node architecture and is comprised ofmonomorphic small to
intermediate sized CD5 positive B cells with irregular nuclear contours and has
a balanced chromosomal translocation involving the CCND1 (cyclin D1) gene
a. Follicular hyperplasia
b. Follicular lymphoma
c. Hyaline vascular variant ofCastleman disease
d. Mantle cell lymphoma
e. Plasma cell variant ofCastleman disease

f.

Thymoma

39. Which of the following immunophenotypes is most compatible with a diagnosis


ofBurkitt lymphoma
a. CD20+ , BCL2+ , CD10-, BCL6-, CD43+
b. CD20+ , BCL2-, CD10+ , BCL6+ , CD43+
c. CD20+ , BCL2+ , CD10+ , BCL6-, CD43d. CD20+ , BCL2-, CD10-, BCL6-, CD43+
40. The immunodeficiency-related Burkitt lymphoma is the clinical variant most
often associated with Epstein-barr virus
a. True
b. False
41. Which statement is most correct concerning Burkitt lymphoma
a. Burkitt lymphoma commonly presents as acute leukemia
b. Burkitt lymphoma consists oflarge transformed cells with nuclear
pleomorphism
c. Cases that do not morphologically resemble classic Burkitt lymphoma can
share the genetic signature ofBurkitt lymphoma
d. The most common genetic abnormality seen is t(14;18)
42. Which entity is characterized by splenic red pulp involevemtn and is positive
for flow cytometry markers CD103, CD11c, and CD25?
a. Hepatosplenic T cell lymphoma
b. Sickle cell crisis Hairy cell leukemia
c. Splenic marginal zone lymphoma
d. Gaucher disease
e. Classical Hodgkin lymphoma
f.

Aggressive NK-cell lymphoma/leukemia

43. Which entity is a hematopoietic malignancy ofpredominantly activated


cytotoxic cells, positive for perforin and granzyme?
a. Hepatosplenic T cell lymphoma

b. Sickle cell crisis Hairy cell leukemia


c. Splenic marginal zone lymphoma
d. Gaucher disease
e. Classical Hodgkin lymphoma
f.

Aggressive NK-cell lymphoma/leukemia

44. A patient with type B symptoms has nodular abnormalities in the spleen
grossly. A well sampled flow cytometry analysis did not reveal T cell or B cell
abnormalities. Histologically reveals scattered large bi-nucleated and
mononuclear cells with a prominent nucleolus. What is the best possible
diagnosis
a. Hepatosplenic T cell lymphoma
b. Sickle cell crisis
c. Hairy cell leukemia
d. Splenic marginal zone lymphoma
e.

Gaucher disease

f.

Classical Hodgkin lymphoma

g. Aggressive NK-cell lymphoma/leukemia


45. Which of the following is the most correct concerning primary amyloidosis?
a. Most patients with plasma cell myeloma or MGUSwill show some
manifestation ofamyloidosis
b. T(11;14) is very specific for patients with primary amyloidosis
c. Most cases of primary amyloidosis occur in patients with a plasma cell
dyscarsia
d. The amyloidogenic protein is usually an intact immunoglobulin with both
heavy and light chains

46. Most cases of familial amyloidosis produce amyloidogenic immunoglobulin light


chain
a. True
b. False
47. Which of the following is most correct concerning splenic amyloidosis?
a. Splenic amyloidosis is usually a manifestation ofgeneralized disease
b. Red pulp involvement is more common in systemic than primary
amyloidosis
c. When rupture occurs, it is usually secondary to a traumatic event
48. Which of the following pathologic processes is inherited in an autosomal
recessive manner?
a. Chronic myelogenous leukemia
b. Gaucher disease
c. Hairy cell leukemia
d. Rosai-Dorfman disease
e. Splenic lymphoma
49. What immu stain is used to differentiate Gaucher cells from pseudo-Gaucher
cells?
a. Prussian blue
b. Ziehl-Neeelson
c. PAS
d. Sudan black b
50. Which is a positive immune marker for Gaucher cells?
a. S100
b. CD68
c. CD1a

d. CD11b
51. A lymph node from an upper cervical biopsy in a young Asian male shows
increased eosinophils some in large clusters along with plasma cells, nodules
separated by fibrocollagenous sclerosis and frequent binucleated giant cells
that are reactive for CD30 and CD15
a. Angiolymphoid hyperplasia with eosinophilia
b. Classical Hodgkin lymphoma
c. Histiocytosis
d. Kimura lymphadenopathy
e. Systemic mastocytosis
52. A 3 year old child presents with bony lesions involving predominantly flat
bones (skull). Biopsy shows increased eosinophils along with a spindle cell
infiltrate which is positive for CD117
a. Angiolymphoid hyperplasia with eosinophilia
b. Classical Hodgkin lymphoma
c. Histiocytosis
d. Kimura lymphadenopathy
e. Systemic mastocytosis
53. Which of the above conditions present as a popular rash in a young Caucasian
female with hitopathology revealing soft tissue and dermal eosinophilic
infiltrate, proliferation of small capillaries with prominent endothelial
hypertrophy and no nodal involvement?
a. Angiolymphoid hyperplasia with eosinophilia
b. Classical Hodgkin lymphoma
c. Histiocytosis
d. Kimura lymphadenopathy
e. Systemic mastocytosis
54. Viral associated hemophagocytic syndrome is a life-threatening entity that is
treated with specific antiviral medications
a. True
b. False
55. Which of the following conditions is not associated with HPS
a. Herpes virus
b. Antiviral therapy

c. Lymphoma
d. Lupus erythematosus
e. Mutations ofthe perforin gene
f.

Immunodeficiency disorders

g. None ofthe above


56. How does one differentiate sinus histiocytosis with massive lymphadenopathy
from HPS?
a. SHML cells have much more cytologic atypia
b. SHML cells typically contain more intact lymphocytes and plasma cells
c. HPShistiocytes are S100 +
d. SHML is associated with a dense eosinophilic infiltration
57. Ofthe small lymphocytic processes that can involve the white pulp of the
spleen, which of the lesions characteristically has the highest proliferation
index within the small lymphocytes
a. Classical Hodgkin lymphoma

b. Diffuse large B cell lymphoma


c. Hairy cell leukemia
d. Mantle cell lymphoma
e. Small lymphocytic lymphoma/CLL
f.

Splenic marginal zone lymphoma

58. Tumor cells of these entity are characteristically sparse within the nodules and
are usually positive for CD30
a. Classical Hodgkin lymphoma
b. DLBL
c. Hairy cell leukemia
d. Mantle cell lymphoma
e. SLL/CLL
f.

Splenic diffuse red pulp small B cell lymphoma

g. Splenic marginal zone lymphoma


59. The tumor cells ofthis B cell leukemia/lymphoma are CD103 positive and DBA
44 +
a. Classical Hodgkin lymphoma
b. DLBL
c. Hairy cell leukemia
d. Mantle cell lymphoma
e. SLL/CLL
f.

Splenic diffuse red pulp small B cell lymphoma

g. Splenic marginal zone lymphoma


60. Which of the following B cell lymphomas most commonly involves
predominantly the red pulp ofthe spleen?
a. CLL/SLL
b. DLBCL
c. Hairy cell leukemia
d. Hodgkin lymphoma
e. Mantle cell lymphoma
f.

SMZL

61. Which of the following associations is most correct?


a. CD5 expression and SMZL
b. CLL and nuclear irregularities
c. CLL/SLL and cyclin D1 overexpression

d. DLBCL and red blood cell lakes


e. Mantle cell lymphoma and t(11;14)
62. Cyclin D1 expression is unique to mantle cell lymphoma
a. True
b. False
63. Which of these lymphomas/leukemias characteristically shows CD20 + , CD11c
+ , CD25 + , CD103+ ?
a. CML
b. Hairy cell leukemia
c. Large B cell lymphoma
d. Mantle cell lymphoma
e. SLL/CLL
f.

SMZL

64. Which hematolymphoid proliferation most commonly shows strong nuclear


expression ofcyclin D1?
a. CML
b. Hairy cell leukemia
c. Large B cell lymphoma
d. Mantle cell lymphoma
e. SLL/CLL
f.

SMZL

65. Which of these lymphomas/leukemias most commonly preferentially involves


the red pulp ofthe spleen with relative sparing ofthe white pulp?
a. CML
b. Hairy cell leukemia
c. Large B cell lymphoma
d. Mantle cell lymphoma
e. SLL/CLL
f.

SMZL

66. Which tumor has nuclear atypia, increased mitotic activity and necrosis?
a. Angiosarcoma
b. Hemangioma
c. Littoral cell angioma
d. Splenic hamartoma
67. Which tumor is characterized by positivity for CD68, von Willebrand factor,
CD31, and CD21?
a. Angiosarcoma
b. Hemangioma
c. Littoral cell angioma
d. Splenic hamartoma
68. Which tumor is composed ofdisorganized stroma and vascular channels of
varying width lined by CD8 positive cells?
a. Angiosarcoma
b. Hemangioma
c. Littoral cell carcinoma
d. Splenic hamartoma
69. Which splenic tumor is found in a subcapsular location and consists of vasculartype spaces filled with lymphocytes, proteinaceous material, and sometimes
erythrocytes, and generally stains positively for CD31, CD34, and VEGFR3?

a. Angiosarcoma
b. Hemangioendothelioma
c. Hemangioma
d. Littoral cell angioma
e. Littoral cell angiosarcoma
f.

Lymphangioma

70. Which splenic tumor is grossly deep red purple with random distribution
throughout the spleen and microscopically reveals small to large spaces filled
largely with erythrocytes separated by delicate fibrous septae or splenic pulp
and lined by bland endothelial cells which generally stain negatively for
VEGFR3 and D2-40
a. Angiosarcoma
b. Hemangioendothelioma
c. Hemangioma
d. Littoral cell angioma
e. Littoral cell angiosarcoma
f.

Lymphangioma

71. Which splenic tumor is a sponge-like partially solid mass microscopically


consisting ofcomplex anastomotic spaces filled with RBCs and lined by plump
bizarre endothelial cells with hyaline globules?
a. Angiosarcoma
b. Hemagioendothelioma
c. Hemangioma
d. Littoral cell angioma
e. Littoral cell angiosarcoma
f.

Lymphangioma

72. Which entity typically is CD20 + , CD 5+ , CD23 -, CD43 -, CD103 -, and DBA44 -,
Bcl2 + , and cyclin D1 -?
a. Follicular lymphoma
b. Hairy cell leukemia
c. Mantle cell lymphoma
d. SLL
e. Splenic marginal zone lymphoma
73. Which entity typically forms splenic lakes, is CD 103 + and DBA44 + , and often
results in a dry tap bone marrow?

a. Follicular lymphoma
b. Hairy cell leukemia
c. Mantle cell lymphoma
d. SLL
e. Splenic marginal zone lymphoma
74. Which enity is typically CD5 -, CD10 -, CD23 -, CD43 -, CD103 -, and DBA44 -,
Bcl2 + and cyclin D1 -?
a. Follicular lymphoma
b. Hairy cell leukemia
c. Mantle cell lymphoma
d. SLL splenic marginal zone lymphoma
75. Which tumor is characterized by an S100 + , fascin + , vimentin + , CD68 + ,
phenotype?
a. Interdigitating dendritic cell sarcoma
76. Which tumor is characterized by cells that have fibrillary cytoplasm and stain
faintly with PAS?

a. Gaucher disease
77. Which typically CD5 negative lymphoma may have reactive lymphoid follicles,
plasmacytic differentiation and lymphoepithelial lesions?
a. Extranodal marginal zone B cell lymphoma ofmucosa associated lymphoid
tissue (MALT lymphoma)
78. Which lymphoma typically demonstrates CD5 and cyclin D1 positivity?
a. Mantle cell lymphoma
79. Which lymphoma would be expected to express both CD10 and Bcl6?
a. Follicular lymphoma
80. Which is a cause of massive splenomegaly and is associated with t(9;22)?
a. Chronic myelogenous leukemia
81. Which is a cause of massive splenomegaly and shows marked red pulp
expansion by CD11c + , CD20 + , CD25 + population?
a. Hairy cell leukemia
82. Which involves the spleen in anodular fashion and shows rare CD30 + , CD15 +
cells in a polymorphous inflammatory background?
a. Hodgkin lymphoma
83. Which is the most common primary tumor ofthe spleen?
a. Hemangioma
84. Which tumor is characterized by CD21 + , CD31 + , CD68 + phenotype?
a. Littoral cell angioma
85. Which tumor is characterized by a zonal histologic pattern?
a. Inflammatory myofibroblastic tumor
86. Which tumor characteristically has a translocation involving the immunoglobulin
heavy chain (IgH) and bcl2 genes?

a. Follicular lymphoma
87. Which can present in children and adults and when it exhibits a t(9;22)
translocation (Philadelphia chromosome) protends a poor prognosis?
a. Precursor B cell acute lymphoblastic leukemia,/lymphoma
88. Which tumor strongly expresses CD99 and TdT?
a. Lymphoblastic lymphoma
89. Which splenic lesion presents grossly as a solitary or multiple nodules
frequently has PAS positive globules and is commonly positive for CD68 and
endothelial markers?
a. Littoral cell angioma
90. An association with the multicentric form ofCastleman disease has been
described in all ofthe following except
a. Follicular dendritic cell sarcoma
b. Primary effusion lymphoma
c. EBV
d. Poems syndrome
91. Which of the following is true regarding systemic mastocytosis?
a. Skin lesions are common in the aggressive forms of the disease
b. Indolent systemic mastocytosis usually does not have bone marrow
involvement
c. A normal serum tryptase level in a pediatric patient with urticaria
pigmentosa essentially rules out the diagnosis of systemic mastocytosis
d. Malabsorption in patients with systemic mastocytosis is usually attributed to
mediator release from the mast cells
e. There is no correlation between the percentage ofmast cells in the bone
marrow and survival
92. Immunophenotyping ofwhich entity typically exhibits surface light chain
clonality without expression ofCD5 or CD10
a. Marginal zone B cell lymphoma

93. Which splenic tumor is highly aggressive and commonly metastasizes?


a. Angiosarcoma
94. An unusual coexpression ofendothelial (CD31) and histiocytic (CD68) antigens
characterizes this splenic lesion
a. Littoral cell angioma
95. Which splenic lesion exhibits a haphazard arrangement ofblood filled spaces
but does not form a discrete mass
a. Peliosis
96. Aggressive lymphoma with diffuse pattern of splenic involvement and
cytoplasmic TIA1 expression
a. Hepatosplenic T cell lymphoma
97. Which tumor is characterized by an immunophenotypic profile ofCD19 + , CD5
-, CD23 -, CD10 -, is TRAP -, and doesn't express cyclin D1?
a. Splenic marginal zone lymphoma
98. Most of the large neoplastic cells are in sinuses have abundant cytoplasm
retiniform to U-shaped nuclei and small nucleoli and are CD15 -, CD20 -, CD30
+ , CD45 + , and EMA +
a. Anaplastic large cell lymphoma, T cell and null cell type
99. Most of the large neoplastic cells have amphophilic to basophilic cytoplasm
round to oval nuclei and large nucleoli and are CD15 -, CD20 + , CD30 -, CD45

+ , and EMA
a. Large B cell lymphoma, T cell rich/histiocyte rich
100.

Most ofthe large neoplastic cells have abundant clear cytoplasm b or

multilobated nuclei easily identified nucleoli and are CD15 + , CD30+ , CD45 -,
and EMA -. Architecture is nodular with dense collagen bands
a. Hodgkin lymphoma, nodular sclerosis type
101. Which lymphoma is typically bcl-1 positive?
a. Mantle cell lymphoma
102.

Which tumor characteristically occurs in the genital region ofboth men

and women?
a. Cellular angiofibroma
103.

The majority of cases ofwhich tumor form angiomatous pseudosinuses

in the liver and spleen?


a. Hairy cell leukemia
104.

Which displays the following phenotype positive for CD43 and

myeloperoxidase, negative for CD20 and CD3


a. Acute myelogenous leukemia
105.

Aggressive malignancy that most commonly expresses gamma-delta T

cell receptor
a. Hepatosplenic T cell lymphoma
106.

All ofthe following are true about mantle cell lymphoma, except?

a. Moderately aggressive with a survival rate ofapproximately five years


b. May involve multiple sites including lymph nodes, spleen, peripheral blood,
bone marrow and GItract
c. May present as intestinal polyps

d. Characterized by clusters ofprolymphocytes and paraimmunoblasts


e. Blastoid transformation denotes a more aggressive course
107.

The most common non-Hodgkin lymphoproliferative disorder involving

the spleen is
a. DLBL
108.

Waldenstrom macroglobulinemia is most commonly associated with

mantle cell lymphoma


a. True
b. False
109.

Which lesion contains toluidine blue positive cytoplasmic granules?

a. Systemic mastocytosis
110. All of the following are characteristic oflow grade MALT lymphoma except
a. May have prominent reactive appearing lymphoid follicles
b. Neoplastic cells have lobulated nuclei that may mimic Reed-Sternberg cells
c. May have groups ofplasma cells below the surface epithelium
d. Lymphoepithelial lesions are characteristic
e. Neoplastic cells have the phenotype of marginal zone B cells
111. The most common site for extranodal lymphoma is
a. GIT
112. The treatment of choice for gastric large cell lymphoma is eradication therapy
ofH pylori
a. True
b. False
113. Which neoplasm is cytologically and immunophenotypically simial to hariy cell
leukemia but shows a different pattern ofinvolvement in the spleen?
a. Marginal zone lymphoma
114. Classical Hodgkin lymphoma differs from NLPHL by which of the following?
a. Multilobated neoplastic cells
b. Neoplastic cells stain positive for CD30 and CD15

c. Nodular growth pattern


d. Sparse neoplastic cells in the background ofmixed inflammatory cells
115. Nodular sclerosis HL can be distinguished from mediastinal large B cell
lymphoma by which ofthe following features?
a. NSHL is CD30 positive
b. NSHL is seldom seen in young patients, whereas PMBL is mostly seen in
younger patients
c. Only PMBL arises in a mediastinal location
d. Sclerotic bands in NSHL separate nodules, while in PMBL they tend to
encircle individual large cells
116. Immunoglobulin heavy chain gene rearrangement studies in NSHL are
frequently negative. This is because
a. NSHL is not a B cell malignancy and only B cell lymphomas are positive for
IGHgene rearrangement studies
b. NSHL is not a clonal disorder, it is an inflammatory disease
c. Neoplastic cell typically make up less than 1% of the sampled specimen
d. The IGHgene locus is lost in NSHL
117. Which entity has the highest genetic predisposition ofall hematologic
malignancies?
a. CLL/SLL
118. Based on gene expression profiling, which entity has two prognostic subtypesgerminal center B cell like (GCB) and activated B cell like (ABC)?
a. DLBL, NOS
119. Which entity is associated with t(11;18) translocation?
a. Extranodal marginal zone lymphoma ofmucosal-associated lymphoid tissue
120.

Which ofthe following entities is characterized by splenic white pulp

involvement and by flow cytometry shows a population that is positive for

CD19 , CD20 with light chain restriction?


a. Classical Hodgkin lymphoma
b. 1ry myelofibrosis
c. Splenic extramedullary hematopoiesis 2ry to 1ry myelofibrosis
d. Splenic marginal zone lymphoma

121. A patient with HIV infection shows symptoms suggestive of BM failure and
pancytopenia . Large binucleated cells with prominent eosinophilic nucleoli are
seen in the BM bx and within spleen. The large atypical cells are CD30 and
CD15 positive. What is the Dx?
a. Classical Hodgkin lymphoma
b. 1ry myelofibrosis
c. Hairy cell leukemia
d. Splenic marginal zone lymphoma

122.

A characteristic morphology seen in 1ry myelofibrosis which is less

common in essential thrombocytosis or CML is the following:


a. Megakaryocytes showing compact cloud like nucleus with condensed
chromatin
b. Enlarged megakaryocytes with branched nuclear lobes
c. Mononuclear megakaryocytes in tight clusters
d. Megakaryocytes with crinkled tissue paper cytoplasm

g.

18. Bone and Joint


1.

15 year old male, radiology shows proximal femur metaphysis bone destruction
and a soft tissue mass
a. Osteosarcoma

2. In multiple myeloma what is released from the malignant plasma cells which
causes lytic bone lesions
3. Patient showing juxtaarticular position oflow grade cartilage neoplasm with
high grade pleomorphic sarcoma
a. Chondrosarcoma
b. Chondroblatic osteosarcoma
c. Dedifferentiated chondrosarcoma
4. Which is not a differential diagnosis ofEwing's
a. Leiomyosarcoma
b. Rhabdomyosarcoma
c. Neuroblastoma
5. Which of the following is false regarding giant cell tumor ofthe bone
a. Age less than 20
6. Which of the following tumors is least likely to metastasize to the bone
a. Squamous cell carcinoma ofthe skin
b. Breast
c. Kidney
d. Thyroid
e. Prostate
7. Which tumor is positive for HMB-45
a. PEComa
8. A 60 year old male patient with a well-demarcated lytic mass in the right iliac
fossa with focal destruction ofthe bone and infiltration into the soft tissue. By

radiology it shows spotty calcification


a. Chondrosarcoma
b. Osteosarcoma
c. Paget's disease
d. Pyogenic osteomyelitis
e. TB osteomyelitis
9. Least common site ofbone metastasis
a. Small bones of hands
b. Skull
c. Ribs
d. Femur
10. Tumor showing high grade osteoid and high grade chondroid areas
a. Chondroblastic osteosarcoma
b. Dedifferentiated chondrosarcoma
11. Most common location ofosteochondroma
a. Metaphysis offemur
b. Ribs
c. Skull
12. A child with blue sclera, multiple fractures, lax joints and teeth abnormalities
(features ofosteogenesis imperfect)
a. Collagen
b. Fibrillin
c. FGFR
13. Ollier's disease is associated with which tumor
a. Enchondroma
14. Tumor showing elongated grooved nuclei, CD1a and S100 +
a. Langerhans cell histiocytosis

15. High risk for developing osteosarcoma


a. P53 mutation
16. Which tumor is morphologically and immunohistochemically similar to Ewing?
a. Lymphoblastic lymphoma
17. Genetic alteration in osteosarcoma
a. Mutation ofRB, P53, INK4a, P16, and P14
18. Which of the following is least likely to be associated with villous synovial
hypertrophy (large nodules and fronds)?
a. Articular prosthesis (synovial reaction to loosened joint arthroplasty)
b. Hemosiderotic synovitis
c. Pigmented villonodular synovitis
d. Rheumatoid synovitis
19. Which of the following is most indicative ofa malignant pigmented villonodular
synovitis?
a. Bone infiltration and destruction oflarge segments ofbone
b. Increased mitoses by itself
c. Marked nuclear atypia, zonal necrosis, sarcomatoid growth pattern, and
> 20 mitoses per 10 HPF
d. Osteoclast-like giant cell reaction
20. Which of the following findings are consistent with a pigmented villonodular
synovitis?
a. Lobulated architecture with osteoclast-like giant cells

b. Villous architecture with foreign body macrophages


c. Villous architecture with osteoclast-like giant cells and xanthoma cells
d. Villous architecture with pigmented limited to the surface ofthe synovium
21. Which of the following lesions is histologically characterized by concentric
arrangement offibrocartilage around hyaline cartilage or bone?
a. Synovial chondromatosis
b. Loose bodies
c. Osteoarthritis
d. Periosteal chondroma
22. Which of the following histologic characteristics can be seen in
chondrosarcoma, but is usually not observed in synovial chondromatosis?
a. Hypercellular hyaline cartilage
b. Ossification
c. Necrosis, increased cellularity and tumor liquefaction, invasion into or from
bone
d. Mild nuclear atypia
23. Which of the following joints are most commonly affected by synovial
chondromatosis?
a. Elbow and knee
b. Temperomandibular joint
c. Ankle and distal interphalangeal joints
d. Proximal and distal interphalangeal joints
24. T(x;18)(p11;q11), SYT-SSX1 is usually seen in which tumor
a.

Desmoplastic small round cell tumor

b. Ewing sarcoma/PNET
c. Lymphoma
d. Neuroblastoma
e. Rhabdomyosarcoma
f.

Small cell carcinoma

g. Synovial sarcoma
25. FLI-1 and CD99 are expressed in which one of these tumors?
a. Desmoplastic small round cell tumor
b. Ewing sarcoma/PNET
c. Lymphoma
d. Neuroblastoma
e. Rhabdomyosarcoma
f.

Small cell carcinoma

g. Synovial sarcoma
26. The histochemical assay ofPASwith diastase can be useful in characterizing
ewing sarcoma cells
a. True
b. False
27. Which entity is characterized by a poorly lobulated architecture, an increased
cell population per unit area with hyperchromatic nuclei increased binucleate
cells lack ofosteoid formation and irregular margins on imaging
a. Chondroblastic osteosarcoma

b. Chondrosarcoma, well-differentiated
c. Dedifferentiated chondrosarcoma
d. Enchondroma
e. Extraskeletal myxoid chondrosarcoma
f.

Fracture callus

28. Which cartilage producing tumor can be identified by the characteristic


separation ofhyaline cartilage lobules by hematopoietic elements
a. Chondroblastic osteosarcoma
b. Chondrosarcoma, well differentiated
c. Dedifferentiated chondrosarcoma
d. Enchondroma
e. Extraskeletal myxoid chondrosarcoma
f.

Fracture callus

29. Which solid tumor demonstrates reproducible chromosomal rearrangements


involving 9q22 which results in the production ofa NR4A3 fusion gene
a. Chondroblastic osteosarcoma
b. Chrondrosarcoma, well-differentiated
c. Dedifferentiated chondrosarcoma
d. Enchondroma
e. Extraskeletal myxoid chondrosarcoma
f.

Fracture callus

30. Which lesion can complicate benign and malignant tumors


a. Aneurysmal bone cyst
b. Desmoplastic fibroma
c. Fibrous dysplasia
d. Fibrosarcoma ofbone, low grade
e. Intraosseous osteosarcoma, low grade
f.

Parosteal osteosarcoma

31. Which condition involves an activating mutation ofthe G protein signaling


pathway
a. Aneurysmal bone cyst
b. Desmoplastic fibroma
c. Fibrous dysplasia
d. Fibrosarcoma ofthe bone, low grade
e. Intraosseous osteosarcoma, low grade

f.

Parosteal osteosarcoma

32. Which entity is characterized by fairly uniform spindle cells arranged in a


herringbone pattern and aggressive growth pattern radiologically
a. Aneurysmal bone cyst
b. Desmoplastic fibroma
c. Fibrous dysplasia
d. Fibrosarcoma ofbone, low grade
e. Intraosseous osteosarcoma, low grade
f.

Parosteal osteosarcoma

33. Which primary bone tumor is characterized by production ofboth high grade
neoplastic cartilage and osteoid?
a. Chondroblastic osteosarcoma
b. Chondrosarcoma
c. Dedifferentiated chondrosarcoma
d. Enchondroma
e. Synovial chondromatosis
34. Which neoplasm is characterized by production ofextraosseous lobules of
bland hyaline cartilage with mildly increased cellularity "clonal" arrangement of
chondrocytes and nesting within a thin connective tissue membrane?
a. Chondroblastic osteosarcoma
b. Chondrosarcoma
c. Dedifferentiated chondrosarcoma
d. Enchondroma
e. Synovial chondromatosis
35. Which bone neoplasm is characterized by juxtaposition oflow grade neoplastic
hyaline cartilage and high grade pleomorphic sarcoma
a. Chondroblastic osteosarcoma

b. Chondrosarcoma
c. Dedifferentiated chondrosarcoma
d. Enchondroma
e. Synovial chondromatosis
36. Which entitiy is a well circumscribed lesion composed of various proportions of
fibrous and osseous elements. The osseous component is represented by
irregular curvilinear discontinuous trabeculae ofwoven or rarely lamellar bone
with no osteoblastic rimming?
a. Fibrous dysplasia
b. Malignant fibrous histiocytoma ofbone
c. Mesenchymal chondrosarcoma
d. Osteoid osteoma
e. Osteosarcoma
f.

Reactive changes secondary to infection

37. Which entity frequently complicates preexisting osseous lesions (i.e radiation
damage, Paget disease, cartilaginous neoplasms, and rarely bone infarcts) and
is microscopically composed ofpleomorphic spindle cells arranged in a
storiform pattern and not associated with tumoral osteoid formation?
a. Malignant fibrous histiocytoma ofbone
38. Which entity is most often associated in later life with an underlying primary
pathologic condition of bone such as fibrous dysplasia or Paget disease?
a. Osteosarcoma
39. This malignant neoplasm can display complex karyotypes but is best known for
translocations involving 1,2,and 13 e.g. t(1;13), t(2;13)
a. DLBL
b. Malignant melanoma, metastatic
c. Multiple myeloma
d. Plasmacytoma
e. RMS

40. Patients with this malignant neoplasm can develop additional disorders due to
deposition ofabnormal immunoglobulin chains (amyloid) in various tissues.
a. DLBL
b. Malignant melanoma
c. Multiple myeloma
d. Plasmacytoma
e. RMS
41. Which tumor is the most common neoplasm ofbone?
a. DLBL
b. Malignant melanoma
c. Multiple myeloma
d. Plasmacytoma
e. RMS
42. Which may be polyarticular and represents a secondary metaplasia of
synovium often due to chronic osteoarthritis?
a. Osteocartilagenous loose bodies
43. Which is most often monoarticular and represents a primary de novo
metaplasia or possible neoplasia ofsynovium?
a. Synovial chondromatosis
44. Which intraarticular process is composed of fibrin?
a. Rice bodies
45. Sacrococcygeal developmental cysts occur most commonly in the pediatric age
group and are more common in males
a. True
b. False
46. For sacrococcygeal teratomas occurring in the pediatric age group, prognosis
is generally better for those patients presenting after the age oftwo months
a. True
b. False
47. Which tumor is likely to express CD99 and demonstrate an EWStype 1 fusion
transcript?

a. Ewing sarcoma/PNET
48. which tumor occurs almost exclusively in the spheno-occipital region?
a. Chondroid chordoma
49. Which tumor is characterized by a cartilage-scapped bony projection arising
on the external surface ofbone with a marrow cavity continuous with that of
the underlying bone?
a. Osteochondroma
50. Which tumor usully arises in the epiphysis ofyoung patients and frequently
shows a fine network ofpericellular calcification (chicken-wire calcification)
a. Chondroblastoma
51. Which tumor most commonly occurs in the long bones ofchildren or young
adults, and is characterized by a spindle cell sarcomatous component with

osteoid formation intimately admixed with a cartilaginous component of


equivalent grade?
a. Osteosarcoma, chondroblastic type
52. Which tumor most commonly occurs in the pelvis or femur ofolder individuals
and consists of a low grade cartilaginous component which abruptly transitions
to a high grade spindle cell sarcoma that may contain osteoid?
a. Dedifferentiated chondrosarcoma
53. Which tumor occurs primarily in the 2nd or 3rd decades, most commonly
involves the jaw but may arise in the soft tissue or meninges and consists of
lobules of well-differentiated cartilage in a background ofundifferentiated
small blue cells?
a. Mesenchymal chondrosarcoma
54. Chordomas are considered tumors ofnotochordal vestiges and most
commonly affect patients in the 2nd and 3rd decades oflife.
a. True
b. False
55. chordoma can typically be distinguished from chondrosarcoma by which of the
following immunohistochemical stains:
a. Brachyury, D2-40, and pan-cytokeratin
b. Galectin-3, pan-cytokeratin, and S-100
c. GFAP, pan-cytokeratin, and CD99
d. S-100, chromogranin, and cytokeratin-7
e. S-100, pan-cytokeratin, and GFAP
56. which of the following statement concerning chordoma is false?
a. Chordomas classically demonstrate physaliphorous cells on histologic
examination
b. Chordomas typically affect younger adults and have no risk of
recurrence
c. Chordomas may be distinguished from myxopapillary ependymoma by
immunohistochemical staining characteristics
d. The candidate gene locus for chordoma has been mapped to chromosome 1p
e. The primary treatment approach for chordoma is en bloc excision with

negative margins
57. Osteosarcoma arising in a 66 y/o man may be associated with paget
s disease
ofbone or Li-Fraumeni syndrome
a. True
b. False
58. Chondroblastoma may produce chicken wirelinear calcification around
mononuclear tumor cells
a. True
b. False
59. Primary chondrosarcomas are the most common primary malignancy ofbone
a. True
b. False

19. Soft Tissue


1.

Bland tumor which is locally aggressive with late metastases


a. Epithelioid hemangioendothelioma
b. Giant cell tumor
c. Cellular schwannoma
d. Fibromatosis
e. Solitary fibrous tumor

2. Angioma with GLUT-1


3. Translocation (x;18) occurs in which tumor
a. Synovial sarcoma
4. Most common location ofglomus tumor is
5. What is the most common soft tissue tumor arising in the first and second
decades oflife
a. Rhabdomyosarcoma, alveolar type
b. Neuroblastoma
c. Ewing sarcoma
d. Lymphoblastic lymphoma
e. Rhabdomyosarcoma, embryonal type
6. Which tumor shows t(x;17)
a. Alveolar soft part sarcoma
7. Loss of INI-1 is seen in
a. Epithelioid sarcoma
b. Angiosarcoma
c. Kaposi sarcoma
8. Which ofthe following tumors strongly expresses ALK-1 (ALK-1 as a prognostic
factor)

a. Inflammatory myofibroblastic tumor


b. Alveolar soft part sarcoma
c. Epithelioid sarcoma
d. Anaplastic large cell tumor
9. A tumor which is positive for SMA and HMB-45 belongs to
a. PEComa
10. Adult male with a lesion in the shoulder and posterior neck
a. Spindle cell lipoma
11. Immunohistochemistry ofGlomus tumor
a. SMA
b. CD34
c. CD31
d. Desmin
12. Which ofthe following lesions shows nuclear expression ofB-catenin, negative
for S100, and variable positivity for c-kit, SMA, and Desmin
a. Mesenteric fibromatosis
b. GIST
c. Leiomyoma
d. Schwannoma
13. Which tumor is least likely to show CD34 expression
a. Synovial sarcoma
b. Epithelioid sarcoma
c. Solitary fibrous tumor

d. Spindle cell lipoma


e. DFSP
14. All of the following tumors have hemangiopericytoma pattern, except
a. Solitary fibrous tumor
b. Infantile hemangioendothelioma
c. Synovial sarcoma
d. Glomus tumor
15. Which tumor mimics breast phylloides tumor
a. Adenosarcoma
16. A patient with tuberus sclerosis has a tumor which is HMB45 +
a. Leiomatosis
b. Rhabdomyoma
17. Tumor showing epithelial, neural, and muscular markers
a. DSCT
18. Tumor showing epitheloid, round tumor cells CAM 5.2 +
19. T(12,16)
a. Myxoid liposarcoma
20. Vascular lesion with "histiocytic" endothelial cells
a. Epithelioid hemangioma
21. Which ofthe following tumors of the scapula is common in elderly females
a. Elastofibroma
22. T(12;22)
a. Clear cell sarcoma
b. Synovial sarcoma
c. Rhabdoid tumor
23. Tumor with FLI-1 positivity
a. Clear cell sarcoma
b. Rhabdoid tumor

c. PTEN/Ewing
d. Synovial sarcoma
24. Immunohistochemistry ofsynovial sarcoma
25. Which ofthe following tumors arising has the greatest potential for malignant
transformation?
a. Cutaneous neurofibroma
b. Optic pathway glioma
c. Palisaded encapsulated neuroma
d. Plexiform neurofibroma
e. Plexiform schwannoma
26. All of the following are true ofmalignant peripheral nerve sheath tumor except
a. Compared to the spindle cell variant, the epithelioid variant shows more
focal and less intense S100 positivity
b. Elongated wavy nuclei help identify neural differentiation
c. The sciatic nerve is commonly involved
d. They typically have uniform cellularity throughout the tumor
e. When associated with NF1 these lesions occur at younger ages
27. Which ofthe following is true regarding perineuromas?
a. Lesional cells show EMA positivity
b. Recent evidence suggests they may be a reactive process
c. They are homogenous in their histologic and clinical presentation
d. They commonly undergo malignant transformation
e. They represent a neoplastic expansion of the most inner portion ofthe
peripheral nerve
28. All of the following histologic features may be seen in hibernoma except
a. Cytoplasmic cross striations
b. Granular eosinophilic cells
c. Myxoid stroma
d. Numerous univacuolated white fat cells
e. Spindle cells
29. Which ofthe following is the most characteristic of the vacuolated cells of
hibernoma

a. They contain glycogen


b. They contain lipid
c. They contain mucin
d. They express CD34
e. They express desmin
30. Hyperchromatic, scalloped nuclei are most characteristic of which ofthe
following soft tissue tumors
a. Adult rhabdomyoma
b. Granular cell tumor
c. Hibernoma
d. Lipoma
e. Liposarcoma
31. A tumor mass attached to the adrenal gland shows mature adipocytes and a
single focus of neutrophils restricted to a congested blood vessel. What is the
diagnosis
a. Extramedullary hematopoiesis
b. Lipoma
c. Liposarcoma
d. Myeloid lipoma
e. Meylolipoma
32. 56 yr old presents with recent onset bluish swelling in the fat pad ofher chin,
she has fever, petechiae and malaise. Biopsy reveals immature atypical cells

with Auer rods and rare granules which stain positive for CD34. What is the
diagnosis?
a. Extramedullary hematopoiesis
b. Lipoma
c. Liposarcoma
d. Myeloid sarcoma
e. Mkyelolipoma
33. Which entity is composed ofadipocytes and myeloid cells that according to
recent evidence belong to the same clone?
a. Extramedullary hematopoiesis
b. Lipoma
c. Liposarcoma
d. Myeloid sarcoma
e. Myelolipoma
34. Which tumor is characterized by the presence ofintracytoplasmic PAS positive
diastase resistant rhomboid crystals
a. Alveolar soft part sarcoma
b. Granular cell tumor
c. Malignant rhabdoid tumor
d. Paraganglioma
e. Rhabdomyosarcoma
35. Which tumor is characterized by loss ofINI1 gene product
a. Alveolar soft part sarcoma
b. Granular cell tumor
c. Malignant rhabdoid tumor
d. Paraganglioma
e. Rhabdomyosarcoma
36. Which tumor is differentiated from alveolar soft part sarcoma on the basis of
positivity for chromogranin and synaptophysin

a. Alveolar soft part sarcoma


b. Granular cell tumor
c. Malignant rhabdoid tumor
d. Paraganglioma
e. Rhabdomyosarcoma
37. Which entity is characterized by the translocation t(X;17)
a. Alveolar soft part sarcoma
b. Metastatic malignant melanoma
c. Metastatic RCC
d. Paraganglioma
e. Well-differentiated neuroendocrine carcinoma
38. Which entity displays positivity for NSE in tumor cells, negativity for CK in
tumor cells and S100 positivity is sustentacular cells
a. Alveolar soft part sarcoma
b. Metastatic malignant melanoma
c. Metastatic RCC
d. Paraganglioma
e. Well-differentiated neuroendocrine carcinoma
39. Which entity may exhibit mutations in mitochondrial succinate dehydrogenase
subunit genes

a. Alveolar soft part sarcoma


b. Metastatic malignant melanoma
c. Metastatic RCC
d. Paraganglioma
e. Well-differentiated neuroendocrine carcinoma
40. Which lesions can be found in Mazabraud syndrome?
a. Deep "aggressive" angiomyxoma
b. Intramuscular myxoma
c. Low grade fibromyxoid sarcoma
d. Myxofibrosarcoma
e. Myxoid liposarcoma
f.

Myxoid neurofibroma

41. A translocation involving t(12;16)(q13;p11) is characteristic of which lesions?


a. Deep "aggressive" angiomyxoma
b. Intramuscular myxoma
c. Low grade fibromyxoid sarcoma
d. Myxofibrosarcoma
e. Myxoid liposarcoma
f.

Myxoid neurofibroma

42. In which lesion would you expect to see giant collagen rosettes?
a. Deep "aggressive" angiomyxoma
b. Itramucular myxoma
c. Low grade fibromyxoid sarcoma
d. Myxofibrosarcoma
e. Myxoid liposarcoma
f.

Myxoid neurofibroma

43. What tumor is most likely to co-express CK5/6 and WT-1 and desmin in a dotlike pattern on immune?
a. Dysgerminoma
b. Epithelioid sarcoma
c. Inflammatory myofibroblastic tumor
d. Intra-abdominal desmoplastic small round cell tumor
e. Renal cell carcinoma

44. Which neoplasm expresses SMA and MSA and is associated with mutations in
the ALK gene on chromosome 2?
a. Dysgerminoma
b. Epithelioid sarcoma
c. Inflammatory myofibroblastic tumor
d. Intra-abdominal desmoplastic small round cell tumor
e. Renal cell carcinoma
45. Which tumor is associated with a reciprocal translocation ofEWSR-WT1 with
gene fusion t(11;22)(p13;q12)?
a. Dysgerminoma
b. Epithelioid sarcoma
c. Inflammatory myofibroblastic tumor
d. Intra-abdominal desmoplastic small round cell tumor
e. Renal cell carcinoma
46. Which tumor is typically positive for desmin and myogenin?
a. Infantile fibromatosis
b. Infantile fibrosarcoma
c. Infantile myofibromatosis
d. Malignant peripheral nerve sheath tumor

e. Spindle cell rhabdomyosarcoma


f.

Synovial sarcoma

47. Which tumor is typically characterized by a biphasic pattern on histology and


CK positivity?
a. Infantile fibromatosis
b. Infantile fibrosarcoma
c. Infantile myofibromatosis
d. Malignant peripheral nerve sheath tumor
e. Spindle cell rhabdomyosarcoma
f.

Synovial sarcoma

48. Which tumor shares a cytogenetic abnormality with cellular mesoblastic


nephroma?
a. Infantile fibromatosis
b. Infantile fibrosarcoma
c. Infantile myofibromatosis
d. Malignant peripheral nerve sheath tumor
e. Spindle cell rhabdomyosarcoma
f.

Synovial sarcoma

49. PEComas ofthe gynecologic tract are as frequently associated with the
tuberous sclerosis complex as renal angiomyolipomas
a. True
b. False
50. Atypical features of PEComas include all ofthe following except
a. Size > 5cm
b. Mitotic activity > 1/50HPF
c. Pushing border
d. High cellularity
51. PEComas involving the uterus are easily distinguished from smooth muscle
tumors beaceuse they are positive for HMB45 and all uterine smooth muscle

tumors are negative for HMB45


a. True
b. False
52. Which ofthe following spindle cell tumors ofsoft tisuue is least likely to show a
prominent herringbone and hemangiopericytoma pattern ofgrowth
a. Fibrosarcoma
b. Synovial sarcoma
c. Leiomyosarcoma
d. Solitary fibrous tumor
e. Malignant peripheral nerve sheath tumor
53. Which ofthe following combinations of immunohistochemical stains are most
consistent with a diagnosis ofsynovial sarcoma
a. Strong and diffuse panCK positivity, CEA + , MOC31+ , CK19+
b. Focal CK + , bcl2+ , calponin+ , CD99+ , EMA+ , TLE1+
c. Focal CK+ , bcl2+ , S100+ , CD56+ , CD10+ , TLE1

d. CK-, bcl2+ , CD99+ , CD34+ , SMA-, TLE1e. CK8/18+ , SMA+ , desmin+ , calponin+ , CD99-, bcl254. Which ofthe following molecular genetics results using FISHwould be
consistent with a diagnosis ofsynovial sarcoma
a. SS18-SSX1 or SS18-SSX2 fusion gene
b. EWSR1-FLI1 fusion gene
c. FUS-DDIT3 fusion gene
d. COL1A1-PDGFB fusion gene
e. EWSR1-NR4A3 fusion gene
55. Which entity is characterized by a reciprocal translocation t(12;22)(q13;q12)
resulting in EWS/ATF1 gene fusion
a. Cellular schwannoma
b. Clear cell sarcoma ofsoft tissue
c. Fibrosarcoma
d. Leiomyosarcoma
e. Malignant peripheral nerve sheath tumor
f.

Synovial sarcoma

56. Which tumor is characterized by sweeping fascicles ofirregularly shaped


spindle cells with lightly-stained indistinct cytoplasm and hyalinized cords that
resemble giant rosettes?
a. Cellular schwannoma
b. Clear cell sarcoma ofsoft tissue
c. Fibrosarcoma
d. Leiomyosarcoma
e. Malignant peripheral nerve sheath tumor
f.

Synovial sarcoma

57. Which tumor shows positive staining for CK, vimentin, EMA, CD99 and bcl2
a. Cellular schwannoma
b. Clear cell sarcoma ofsoft tissue
c. Fibrosarcoma
d. Leiomyosarcoma
e. Malignant peripheral nerve sheath tumor
f.

Synovial sarcoma

58. Which lesions is most often associated with severe thrombocytopenia, purpura,

and consumptive coagulopathy (Kasabach-Merritt phenomenon)


a. Infantile hemangioma
b. Kaposiform hemangioendothelioma
c. Angiosarcoma
d. Venous malformation
e. Neonatal hemangiomatosis
59. Which ofthe following immunohistochemical markers is expected to be positive
in Kaposiform hemangioendothelioma
a. HHV8
b. Glut-1
c. D2-40
d. HMB45
e. Myogenin
60. The biologic behavior ofKaposiform hemangioendothelioma is typically
characterized by
a. Proliferative phase of growth over a period ofmonths, followed by slow
involution during the first year of life
b. Rapid near-complete involution in the first weeks oflife
c. Partial involution in the first weeks oflife, followed by a prolonged period
ofstability
d. Progressive growth
e. Distant metastasis
61. Which lesion is characterized by round, epithelioid cells with duct-like
structures and stains with CAM5.2
a. Angiosarcoma
b. Glomus tumor
c. Hemangiopericytoma/solitary fibrous tumor
d. Hidradenoma
e. Myopericytoma

62. Which lesion is characterized by round cells surrounding vessels, with those
cells showing a round, centrally placed nucleus
a. Angiosarcoma
b. Glomus tumor
c. Hemangiopericytoma/solitary fibrous tumor
d. Hidradenoma
e. Myopericytoma
63. Which lesion may be immunoreactive for CD99
a. Angiosarcoma
b. Glomus tumor
c. Hemangiopericytoma/solitary fibrous tumor
d. Hidradenoma
e. Myopericytoma
64. All of following have been correlated with adverse outcome in myxoid
liposarcomas except
a. FUS-CHOP fusion type
b. Large size
c. Necrosis
d. Proliferation index
e. Round cell differentiation
65. Which ofthe following classic syndromes may be associated with multiple
myxomas?
a. Angelman syndrome
b. Cotard's syndrome
c. Dandy-walker syndrome
d. Ellis van-Creveld syndrome
e. McCune-Albright syndrome
66. Myxofibrosarcomas recur frequently, but metastases are extremely rare (< 2%)
a. True

b. False
67. Neuroblastic tumors are most likely to arise from which ofthe following
anatomic sites
a. Adrenal gland
b. Anterior mediastinum
c. Optic nerve
d. Organ ofzuckerkandl
e. Spinal cord
68. Neuroblastic tumors are most often associated with which ofthe following
genetic abnormalities or syndromes
a. Multiple endocrine neoplasia syndrome
b. MYCN gene amplification
c. Neurofibromatosis type I
d. Neurofibromatosis type II
e. RET oncogene mutation
69. Which morphologic feature distinguishes intermixed ganglioneuroblastoma from
ganglioneuroma?
a. High mitotic-karrhectic index
b. Nodular gross morphology
c. Presence ofislands ofneuropil and differentiating cells
d. Presence ofmature ganglion cells
e. Schwannian stroma > 50% oftumor area

70. Which ofthe following lesions is not immunoreactive to epithelial antigens


a. Fibrosarcoma
b. Leiomyosarcoma
c. Malignant peripheral nerve sheath tumor
d. Synovial sarcoma
71. Which ofthe following is most typically c-kit positive on immunohistochemistry
a. GIST
b. Leiomyosarcoma
c. Malignant peripheral nerve sheath tumors
d. Synovial sarcoma
72. The translocation t(x;18) is specific for which tumor
a. Fibrosarcoma
b. GIST
c. Leiomyosarcoma
d. Synovial sarcoma
73. Which tumor characteristically shows the following immunohistochemical profile:
Ck + , vimentin + , CD34 + , S100 -, CD45 -?
a. Epithelioid sarcoma
b. Hodgkin lymphoma
c. Lymphoma, diffuse large B cell type
d. Melanoma
e. Metastatic RCC
74. Which tumor shows the following immunohistochemical pattern: CK + , vimentin
+ , S100 -, CD45 -, EMA + , CD34 -?
a. Epithelioid sarcoma
b. Hodgkin lymphoma
c. Lymphoma, diffuse large B cell type
d. Melanoma
e. Metastatic RCC

75. Which tumor shows the following immunohistochemical profile: CK -, vimentin


+ , s100 -, CD45 -, CD15 + , CD30 + ?
a. Epithelioid sarcoma
b. Hodgkin lymphoma
c. Lymphoma, diffuse large B cell type
d. Melanoma
e. Metastatic RCC
76. Which entity is typically a rapidly growing process characterized by cells with a
tissue culture appearance and extravasated RBCs?
a. DFSP
b. Myxoid liposarcoma
c. Neurofibroma
d. Nodular fasciitis
e. Spindle cell lipoma
77. Which entity is a slowly growing well-circumscribed tumor that is composed of
a mixture of amature adipocytes and spindle cells and is typically positive for
CD34?

a. DFSP
b. Myxoid liposarcoma
c. Neurofibroma
d. Nodular fasciitis
e. Spindle cell lipoma
78. Which tumor is characterized by mitotically active CD34 positive spindle cells
arranged is a storiform pattern with an infiltrative border?
a. DFSP
b. Myxoid liposarcoma
c. Neurofibroma
d. Nodular fasciitis
e. Spindle cell lipoma
79. Whivh tumor is typically characterized by an immunohistochemical profile of
CD99 + , CD43 + , CD34 + ,CD10 + , CD79a + ?
a. Alveolar rhabdomyosarcoma
b. Desmoplastic small round cell tumor
c. Ewing sarcoma/PNET
d. Lymphoblastic lymphoma
e. Metastatic small cell carcinoma
f.

Neuroblastoma

80. Which tumor frequently shows translocation t(2;13)(q35;q14)?


a. Alveolar rhabdomyosarcoma

81. Which tumor typically arises within the peritoneal cavity ofadolescent males?
a. Desmoplastic small round cell tumor
82. Which myxoid tumor may be locally aggressive and is characterized by bland
stellate-shaped muscle specific actin + mesenchymal cells in an abundant
myxoid stroma containing thick walled blood vessels?
a. Aggressive angiomyxoma
b. Angiomyofibroblastoma
c. Myxoid leiomyoma
d. Myxoid neurofibroma
83. Which tumor is well-circumscribed usually does not recur after excision and is
characterized by epithelioid cells clustered around capillary sized blood vessels?
a. Aggressive angiomyxoma
b. Angiomyofibroblastoma
c. Myxoid leiomyoma
d. Myxoid neurofibroma
84. Which tumor shows interlacing fascicles of wavy elongated cells in association
with strands of collagen within a myxomatous background with no mitoses and
can be associated with mutations involving the long arm ofchromosome 17?
a. Aggressive angiomyoma
b. Angiomyofibroblastoma
c. Myxoid leiomyoma
d. Myxoid neurofibroma
85. Which enity is characterized by strong immunoreactivity for EMA and CK?
a. Alveolar rhabdomyosarcoma

b. alveolar soft part sarcoma


c. Granular cell tumor
d. Metastatic melanoma
e. metastatic renal cell carcinoma
f.

Paraganglioma

86. Which entity is characterized by membrane bound rhomboid crystals that are
PASpositive diastase resistant?
a. Alveolar rhabdomyosarcoma
b. Alveolar soft part sarcoma
c. Granular cell tumor
d. Metastatic melanoma
e. Metastatic renal cell carcinoma
f.

Paraganglioma

87. Which entity is associated with a t(2;13)(q35;q14)translocation involving the


PAX3 gene?
a. Alveolar rhabdomyosarcoma
b. Alveolar soft part sarcoma
c. Granular cell tumor
d. Metastatic melanoma
e. Metastatic renal cell carcinoma
f.

Paraganglioma

88. Which vanishingly rare primary tumor of bone or soft tissue classically displays
a herringbone pattern?
a. Desmoplastic fibroma
b. Fibromatosis

c. Fibrosarcoma
d. Post-radiation osteosarcoma, fibroblastic type
e. Reactive/radiation changes
f.

Solitary fibrous tumor

89. Which tumor is known for its infiltrative borders and high rate of local
recurrence but is without metastatic potential?
a. Desmoplastic fibroma
b. Fibromatosis
c. Fibrosarcoma
d. Post-radiation osteosarcoma, fibroblastic type
e. Reactive/radiation changes
f.

Solitary fibrous tumor

90. Which tumor can occur years after radiation therapy and produces osteoid by
its malignant cells?
a. Desmoplastic fibroma
b. Fibromatosis
c. Fibrosarcoma
d. Post-radiation osteosarcoma, fibroblastic type
e. Reactive/radiation changes
f.

Solitary fibrous tumor

91. Which tumor is the most common subtype ofRMS in children and adults
a. Embryonal RMS
b. Leiomyosarcoma
c. MMMT
d. Nodular fasciitis
e. Pleomorphic RMS
92. Which variant of RMSoccurs essentially only in adults?
a. Embryonal RMS
b. Pleomorphic RMS
93. Which ofthe following tumors shows an alveolar growth pattern with strong
nuclear staining for Myo-D1 and cytoplasmic staining for desmin?
a. Alveolar RMS
94. Which ofthe following immunohistochemical stains is most specific for alveolar

soft part sarcoma?


a. S100
b. SMA
c. CK AE1/3
d. TFE3
95. Which tumor is characterized by myxoid stroma that is chondroitin sulfate rich
instead ofhyaluronic acid-rich and shows chromosomal rearrangements
involving 9q22 resulting in NR4A3 fusion gene?
.aE xtraskeletal myxoid chondrosarcoma
96. Which lesion prefers the cervicofacial areas ofyoung women and is S100
positive?
a. Myxoid neurothekoma (nerve sheath myxoma)
97. Which lesion may be associated with precocious puberty and fibrous dysplasia?
a. Intramuscular myxoma (often as part of McCune-Albright syndrome)
98. Which lesion demonstrates expression ofCD68 and SMA?
a. Plexiform fibrohistiocytic tumor
99. Which lesion is most likely to occur in association with a genetic disorder
involving chromosome 22?
a. Eight nerve (acoustic) schwannoma
100. Which lesion is most strongly associated with neurofibromatosis type 1 (NF1)?
a. Plexiform neurofibroma
101. Which tumor is most commonly associated with NF1 deletions?
a. Malignant peripheral nerve sheath tumor
102. Which malignant tumor most often arises in the retroperitoneum in adults?
a. Cellular schwannoma
b. Fibrosarcoma
c. Hemangiopericytoma
d. Leiomyosarcoma
e. Malignant peripheral nerve wheath tumor
f.

Poorly differentiated synovial sarcoma

103. Which lesion contains hyaline globules and extravasated red cells?

a. Kaposi sarcoma
104. Which lesion is non-neoplastic caused by Bartonella species and responsive to
antibiotics?
a. Bacillary angiomatosis
105. Which lesion is malignant and not associated with HIV/AIDS?
a. Angiosarcoma
106. Which lesion contains increased and abnormal elastic fibers?
a. Elastofibroma
107. Which entity may show progressive maturation with each recurrence?
a. Lipoblastoma
108. Which entity is subclassified based on the tissue or site involved by the tumor?
a. Lipoblastoma
109. Which tumor expresses SMA and melanocytic markers including HMB45?
a. Perivascular epithelioid cell tumor (PEComa)
110. Which ofthe following favors a diagnosis ofangiosarcoma over epithelioid
hemangioendothelioma ofthe liver?
a. Grossly appears as a hemorrhagic tumor with a solid and cystic areas
b. Tumor calcification

c. Central area of the tumor is often fibrotic and paucicellular


d. Marked predilection for invading vascular structures such as portal and
central veins often mimickingvascular thrombosis
e. Immunohistochemical expression ofendothelial markers like CD31 and
factor VIII
111. Which ofthe following is not a characteristic feature ofsolitary fibrous tumor?
a. Pleural location
b. Alternating hypercellular and paucicellular areas
c. Bland spindle cells with broad zones ofhyalinized collagen
d. CD34 expression by immunohistochemistry
e. Aggressive clinical behavior
112. Which tumor in a 6 month old child has a pattern of uniform spindle cells in
tightly packed fascicles with focal areas ofa complex network ofthin walled
sinusoidal blood vesssels and shows a t(12;15)(p13;q25) and/or fusion gene ETV6
-NTRK3 by RT-PCR?
a. Infantile fibromatosis, cellular type
b. Infantile fibrosarcoma
c. Infantile hemangiopericytoma
d. Infantile myofibromatosis
e. Inflammatory myofibroblastic tumor
f.

Spindle cell rhabdomyosarcoma (embryonal RMS, spindle cell type)

113. Which spindle cell tumor in an 18 month old child mayhave a prominent
background ofmostly lymphocytes and plasma cells is positive for ALK1 and or
p80?

a. Inflammatory myofibroblastic tumor


114. Which tumor has uniform spindle cells with eosinophilic cytoplasm, variable
amounts ofcollagen and is vimentin + , CK =, EMA -, and desmin + , and
myogenin + ?
a. Spindle cell RMS
115. Which tumor is frequently associated with Carney syndrome?
a. Psammomatous melanotic schwannoma
116. Which tumor exhibits a local recurrence rate ofup to 50% lacks a high mitotic
rate and shows perivascular hyalinization?
a. Pleomorphic hyalinizing angiectatic tumor
117. Which tumor has a strong predilection for men?
a. Spindle cell lipoma
118. Which tumor is characterized by a mixture ofheavily collagenized and myxoid
zones, bland spindle cells, and arcades ofblood vessels?
a. Low grade fibromyxoid sarcoma
119. Which malignant appearing spindle cell neoplasm has histologic and
immunophenotypic smooth muscle features and is negative for CK and HMB45?
a. Leiomyosarcoma
120. Which tumor is characterized by a unique polyphenotypic
immunohistochemical profile?
a. Desmoplastic small round cell tumor
121. Which neoplasm is typically and uniformly CD99 negative?
a. Neuroblastoma

122. Which is the most common malignant soft tissue tumor in the 1st two decades
oflife?
a. Embryonal RMS
123. Which tumor is usually characterized by the presence of uniform spindle
shaped cells absent mitotic activity prominent stromal hyalinization diffuse CD34
immunoreactivity and absence ofepithelial markers or S100?
a. Solitary fibrous tumor
124. Which tumor is usually characterized by the presence of uniform cells with low
mitotic rate inconspicuous to dilated and ramifying vascular spaces CK or EMA
demonstration oft(x;18) and CD34 negativity?
a. Monophasic synovial sarcoma
125. Which tumor is usually characterized by fascicular growth with fascilces
intersecting at right angles, blunt ended nuclei often with paranuclear vacuoles
immunoreactivity for SMA or desmin and absence ofCD34, bcl2 and epithelial
markers?
a. Leiomyosarcoma
126. Spindle cell lipoma is characterized by all ofthe following except
a. High frequency of recurrence
127. Schwannoma is characterized by all of the following except
a. Frequent axons within the tumor
128. Low grade fibromyxoid sarcoma most often
a. Has a mixture ofmyxoid and collagenous areas
129. Aggregates ofovoid tumor cells surrounding prominent endothelial lined
sinusoidal blood vessels keratin -, CD34 + , phenotype
a. Hemangiopericytoma
130. Long sweeping fascicles ofspindled cells with a herring-bone pattern and
conspicuous mitotic activity, vimentin + only phenotype
a. Fibrosarcoma
131. Short fascicles of plump spindled cells prominent mast cell infiltrate calcification
keratin + , CD99 + , bcl 2+

a. Monophasic synovial sarcoma


132. Cellular schwannoma are characterized by all ofthe following except?
a. Metastatic potential
133. Features ofhemangiopericytoma include all of the following except
a. EMA positivity
134. Which lesion is typically seen in a subcutaneous location and often associated
with pain?
a. Angiolipoma
135. Which tumor is also known as malignant melanoma of soft parts?
a. Clear cell sarcoma
136. Supernumerary ring chromosomes ofchromosome 12 are a common feature
of
a. Well-differentiated liposarcoma (atypical lipomatous tumor)
137. The head and neck region is the most common site for which tumor?
a. Adult rhabdomyoma
138. Which condition typically presents in childhood and is a hereditary progressive
debilitating disorder?
a. Fibrodysplasia ossificans progressive

139. Which condition is self-limiting ossifying process that follows trauma and
morphologically demonstrates a zonal phenomenon with immature tissues in
the center of the lesion and mature bone at the periphery?
a. Myositis ossificans
140. Which condition typically presents in the 6th -7th decades oflife and is a
primary malignancy of the soft tissues?
a. Extraskeletal osteosarcoma
141. Ring chromosomes characteristic of well differentiated liposarcoma are also
commonly seen in
a. Dedifferentiated liposarcoma
142. The presence of lipoblasts is diagnostic ofliposarcoma
a. True
b. False
143. Which of the types of RMSis considered an unfavorable histologic type?
a. Alveolar RMS
144. Which immunohistochemical markers are used for diagnosis ofRMS?
a. Myogenin
b. Myo-D1
c. MSA
d. Desmin
e. All of the above
145. Which lesion often occurs in the hand or forearm and may simulate
granulomatous inflammation?
a. Epithelioid sarcoma

146. Which lesion often involves tendons or aponeuroses ofthe foot or ankle and
may contain melanin pigment?
a. Clear cell sarcoma
147. Which entity is closely related to round cell liposarcoma?
a. Myxoid liposarcoma
148. Which lesion shows an alcian blue positive but hyaluronidase resistant
mucopolysaccharide stroma?
a. Myxoid chondrosarcoma
149. Which lesion is thought to represent an unusual host response to bacterial
infection that is characterized by the presence ofPAS positive diastase resistant
intracytoplasmic calcospherites?
a. Malakoplakia
150. Which is a milgnant neoplasm that is characterized by an intense neutrophilrich inflammatory infiltrate with intracytoplasmic neutrophils often present
within neoplastic or xanthoma cells?
a. Inflammatory malignant fibrous histiiocytoma
151. Which one of the factors is associated with a good prognosis in solitary fibrous
tumor
a. Complete surgical excision
152. Which is the most common location for extra-abdominal desmoid tumors?
a. Neck
153. Which lesion may display a t(12;15)(13;q25) chromosome rearrangement with
fusion ofthe ETV6 and NTRK3 genes?
a. Congenital (infantile) fibrosarcoma
154. Which lesion is a fibrous proliferation ofinfants that characteristically involves
the fingers and toes and demonstrates fibroblasts that contain intracytoplasmic

eosinophilic inclusions on H&E stain?


a. Infantile digital fibromatosis
155. Which associated with recurrence ofdesmoid tumor?
a. Positive resection margins
b. Abundant centrally located slit-like vessels in the mass
c. Familial adenomatous polyposis
d. Younger age
e. All of the above
156. Familial adenomatous polyposis coli is associated with which ofthe following
tumors?
a. Desmoid tumor
157. Which of the following features distinguish reactive fibrosis from desmoid
tumor?
a. Recent and or old hemorrhage
b. Abundant collagen
c. Circumscription
d. Cellular atypia
e. Abundant fibroblasts
158. Which of the following is true regarding tumoral calcinosis?
a. Associated with abnormalities in calcium metabolism
b. Lesions with identical histological appearance may be seen in patients with
chronic renal disease and secondary hyperparathyroidism
c. Patients most often present with a solitary lesion
d. Radiographs typically demonstrate associated osteoporotic changes or
other bony abnormalities
e. The disease usually has its onset in the sixth to seventh decade oflife
159. A three-stage classification tier has been described for tumoral calcinosis and
includes cellular lesions with no calcification, cellular cystic lesions with
calcification, and hypocellular calcified lesions
a. True
b. False

160. What lesion is characterized by multiple tumor nodules (usually on the arm or
hand) composed histologically ofbland mononuclear cells, short spindle cells,
and osteoclastic giant cells?
a. Aneurysmal bone cyst
b. Dystrophic calcification secondary to gluteal ischemic necrosis
c. Giant cell reparative granuloma
d. Giant cell tumor ofsoft tissue
e. Tumoral calcinosis
161. Which tumor is characterized by small to medium sized blood vessels with
mural hyalinization?
a. Aggressive angiomyxoma
b. Angiomyofibroblastoma-like tumor ofmale genital tract
c. Malignant fibrous histiocytoma
d. Spindle cell lipoma
e. Well-differentiated liposarcoma
162. Which tumor can show areas of dedifferentiation to high grade sarcoma?
a. Aggressive angiomyxoma
b. Angiomyofibroblastoma-like tumor ofthe male genital tract
c. Malignant fibrous histiocytoma
d. Spindle cell lipoma
e. Well-differentiated liposarcoma
163. Which tumor is most often characterized by MDM2 gene amplification?
a. Well-differentiated liposarcoma
164. Translocation oft(12;22)(q13;q12)-ATF1 and EWSis usually seen in which
neoplasm?
a. Clear cell sarcoma
165. Which tumor is usually positive for FLI-1?
a. Primary renal Ewing sarcoma/PNET
166. Angiosarcoma can be associated with prior radiation treatment in breast
cancer patients, with relative short (often less than 2 years) latency period

a. True
b. False
167. Almost all Kaposi sarcomas express the HHV8 latency associated nuclear
antigen (LANA) a marker not expressed by other vascular lesions, including
angiosarcoma
a. True
b. False
168. Angiosarcomas almost never express CK
a. True
b. False
169. Which of the following is true regarding myxoid round cell liposarcoma?
a. It is associated with mutations in the KIT gene on chromosome 4
b. It shares the same chromosomal translocation as Ewing sarcoma, t (11;22)
c. The gastrointestinal tract is a typical site for this tumor
d. The proportion of round cell component has prognostic significance
e. It has a typical curvilinear vascular pattern with pericytic investment
170. Which of the following is/are typical histologic features of myxoid/round cell
liposarcoma?
a. Branching thin walled blood vessels (chicken wire pattern)
b. Primitive non-lipogenic mesenchymal cells
c. Prominent myxoid stroma
d. Signet ring lipoblasts
e. All of the above
171. Which ofthe following is not a typical site for metastasis from a myxoid/round
cell liposarcoma?
a. Brain
b. Liver
c. Lung
d. Lymph nodes
e. Soft tissue
172. Elastofibroma tends to recur even when completely excised
a. True
b. False

173. Nuchal-type fibroma is more common in men and shows thick, haphazardly
arranged collagen fibers and elastic-poor fibrous tissue
a. True
b. False
174. Which is the most common tumor found in NF 1?
a. Localized neurofibroma
175. Which lesion arises as a plaque like lesion of the dermis and subcutaneous
tissue, and has abundant Meissner-like bodies?
a. Diffuse neurofibroma
176. Regardening myogenin immunohistochemical staining, which ofthe following
patterns correlate with embryonal and alveolar rhabdomyosarcoma staining
respectively?
a. Embryonal rhabdomyosarcoma shows cytoplasmic staining and alveolar
rhabdomyosarcoma shows strong diffuse nuclear staining
b. Embryonal rhabdomyosarcoma is positive and alveolar rhabdomyosarcoma is
negative for myogenin
c. Embryonal rhabdomyosarcoma shows membrneous staining and alveolar
rhabdomyosarcoma shows cytoplasmic staining
d. Embryonal rhabdomyosarcoma shows variable nuclear staining and
alveolar rhabdomyosarcoma shows strong diffuse nuclear staining
177. Which ofthe following genetic changes is most commonly found in alveolar
rhabdomyosarcoma ?

a. N-MYC gene amplifications


b. t(1:13)
c. t(2:13)
d. t(11:22)

178. Which ofthe following is not a favorable prognostic factor in rhabdomyosarcoma?


a. Age less than 10 years
b. Orbital location
c. Parameningeal location
d. Size less than 5 cm

176. Peritoneum and


Retroperitoneum
1.

Young male patient with a tumor in the peritoneal cavity, the tumor is most
likely (the most common cause of retroperitoneal mass in adolescent males)
a. Desmoplastic round cell tumor
b. Ewing sarcoma/PNET
c. Alveolar soft part sarcoma
d. Embryonal rhabdomyosarcoma

2. Which entity is characterized by a collection ofendothelial-lined cysts


surrounded by a fibrous stroma containing smooth muscle fibers
a. Florid cystic endosalpingiosis
b. Lymphangioma
c. Malignant peritoneal mesothelioma
d. Multicystic peritoneal mesothelioma
e. Pseudomyxoma peritonei
3. In addition to ymphangioma, in which other cystic lesion do D2-40 + cells line
the cystic structures
a. Florid cystic endosalpingiosis
b. Lymphangioma
c. Malignant peritoneal mesothelioma
d. Multicystic peritoneal mesothelioma
e. Pseudomyxoma peritonei
4. Which entity typically occurs in women ofreproductive age and may contain
cilia in the cyst lining
a. Florid cystic endosalpingiosis

b. Lymphangioma
c. Malignant peritoneal mesothelioma
d. Multicystic peritoneal mesothelioma
e. Pseudomyxoma peritonei
5. Which entity is characterized by diffuse peritoneal involvement by a tumor
comprised ofabundant pools ofextracellular mucin containing identifiable
neoplastic epithelium with prominent solid or cribriform growth with obvious
cytologic atypia or the presence of signet ring cells?
a. Metastatic mucinous adenocarcinoma (mucinous carcinoma peritonei), high
grade
b. Metastatic mucinous adenocarcinoma (mucinous carcinoma peritonei), low
grade
c. Peritoneal mesothelioma
d. Primary peritoneal serous carcinoma
6. Which tumor may rarely produce abundant hyaluronic acid mimicking
extracellular mucin, but the tumor cells are positive for calretenin WT-1 and
CK5/6?
a. Metastatic mucinous adenocarcinoma (mucinous carcinoma peritonei), high
grade
b. Metastatic mucinous adenocarcinoma (mucinous carcinoma peritonei), low
grade
c. Peritoneal mesothelioma
d. Primary peritoneal serous carcinoma
7. Which entity is characterized by diffuse peritoneal involvement by a tumor
comprised ofabundant pools ofextracellular mucin containing identifiable
neoplastic epithelium with bland cytologic features and cells arranged in strips
or simple glands without a signet ring cell component?
a. Metastatic mucinous adenocarcinoma (mucinous carcinoma peritonei), high
grade
b. Metastatic mucinous adenocarcinoma (mucinous carcinoma peritonei), low
grade
c. Peritoneal mesothelioma

d. Primary peritoneal serous carcinoma


8. Which entity is characterized by a HMB45 + , EMA -, keratin -, S100 -,
phenotype?
a. Angiomyolipoma
b. Benign multicystic peritoneal mesothelioma
c. Lymphangioma (lymphatic malformation)
d. Mesenteric cyst
e. Solid pseudopapillary tumor ofthe pancrease
9. Which potentially malignant tumor is characterized by a grossly cystic and solid
appearance and positivity for CD10 and B catenin?
a. Angiomyolipoma
b. Benign multicystic peritoneal mesothelioma
c. Lymphangioma
d. Mesenteric cyst
e. Solid pseudopapillary tumor ofthe pancreas
10. Which tumor exhibits CD117 and CD34 immunoreactivity and can have an
epithelioid morphology?
a. GIST
b. Malignant mesothelioma
c. Mesothelial hyperplasia
d. Metastatic colorectal adenocarcinoma
e. Metastatic pulmonary adenocarcinoma
11. Which is a common incidental finding in the peritoneum ofwomen with
chronic salpingitis and endometriosis?
a. GIST
b. Malignant mesothelioma
c. Mesothelial hyperplasia
d. Metastatic colorectal adenocarcinoma
e. Metastatic pulmonary adenocarcinoma
12. Nuclear staining for TTF-1 is a common pattern of immunoreactivity for which
tumor?
a. GIST
b. Malignant mesothelioma
c. Mesothelial hyperplasia

d. Metastatic colorectal adenocarcinoma


e. Metastatic pulmonary adenocarcinoma
13. Which tumor typically shows nuclear accumulation of B catenin protein aas the
result of either APC or B catenin gene mutations?
a. Desmoid type fibromatosis
b. GIST
c. Sclerosing mesenteritis
d. Sclerosing well-differentiated liposarcoma
e. Solitary fibrous tumor
14. The presence of extensive fat necrosis is most characteristic ofwhich entity?
a. Desmoid-type fibromatosis
b. GIST
c. Sclerosing mesenteritis
d. Sclerosing well-differentiated liposarcoma
e. Solitary fibrous tumor
15. Which tumor contains enlarged hyperchromatic cells within fibroblastic septa
and abnormally configured thick walled blood vessels containing similar cells?
a. Desmoid type fibromatosis
b. GIST
c. Sclerosing mesenteritis
d. Sclerosing well-differentiated liposarcoma
e. Solitary fibrous tumor

177. Cardiovascular System


1.

A woman died ofcomplications of SLE and was found to have pink plaques on
her tricuspid valve, what is the cause ofthese white plaques
a. ANA

2. Female with diarrhea, cramps, nausea and vomiting, on/offskin flushing


developed vegetation on mitral and tricuspid valves. These vegetations are
caused by
a. Carcinoid
3. Young female with neurologic deficit, visual disturbances and peripheral
numbness. Which of the following vasculitis is the cause ofthese symptoms
a. Takayasu
b. Kawasaki
c. Temporal (giant cell) arteritis
d. PAN
e. Wegner's
4. Which of the following drugs of abuse are associated with myocardial
infarction
a. Opiates
b. Heroin
c. Cocaine
5. Commonest organism causing infective endocarditis in native valve
a. S. aureas
b. S. viridans

c. S. pyogens
d. E. coli
e. Strept pneumonia
6. Commonest organism causing infective endocarditis in prosthetic valve
a. S. epidermadis (coagulase negative staph)
7. Restrictive cardiomyopathy
a. Amyloidosis
8. Non-bacterial thrombotic endocarditis is seen in association with
a. Terminal neoplasia
b. SLE
9. A patient with jaundice is found to have a tumor in the head ofthe pancreas
and multiple masses in the lung and liver, the problem with the heart is
a. Non-bacterial thrombotic endocarditis
b. Acute fibrous endocarditis
c. Dilated cardiomyopathy
d. Right sided heart failure
e. Left sided heart failure
10. Most commonly occurs with rheumatic heart disease
a. Mitral stenosis
11. All of the following can cause sudden cardiac death, except
a. Acute rheumatic fever
b. Congenital heart disease
c. Dilated cardiomyopathy
d. Mitral valve prolapse
e. Aortic valve stenosis
f.

Hereditary conduction system abnormalities

12. A young female with ocular disturbance and marked weakness of pulses in the
upper extremities and thickened aortic arch and great vessels

a. Takayasu arteritis
13. C-ANCA and granulomatous vasculitis
a. Wegner's
14. Regarding pure right heart failure
15. Cor pulmonale
a. Right ventricular hypertrophy
16. Patient has symptoms ofpulseless disease
a. Takayasu disease
17. Rheumatic heart disease
a. Aschoffbodies
18. Cyanosis most likely occurs with
a. Tetralogy offallot
19. 72 year old man with aortic valve stenosis and nodular deposits on it is due to
a. Dystrophic calcification
b. Lipofuscin deposition
c. Amyloidosis
d. Hemosiderosis
e. Fatty changes
20. Patient has developed mural thrombus in the left ventricle post MIdue to
a. Endocardial injury
b. Stasis ofblood
c. Heart failure
d. Hypercoagulability
21. Which of the following cardiac diseases is diagnosed by taking a biopsy from
another part of the body
a. Infiltrative cardiomyopathy

b. Hypertrophic cardiomyopathy
c. Dilated cardiomyopathy
d. Loffler carditis
e. Restrictive cardiomyopathy
22. In a child, which is the most common cardiac tumor which could be congenital
a. Rhabdomyoma
b. Myxoma
c. Papillary fibroepithelial polyp
23. Most common cause ofdissecting aortic aneurysm
a. Hypertension
24. Cardiac myxoma is most commonly found in
a. Left atrium
25. What type ofcell is involved in temporal arteritis?
a. Neutrophils
26. The findings of benign mucin-producing CK 7 positive glands and
extramedullary hematopoiesis have been described in this entity
a. Cardiac myxoma
b. Hemangioma
c. Organizing mural thrombus
d. Papillary fibroelastoma
27. Which entity most often involves the cardiac valves?
a. Papillary fibroelastoma
28. Which entity may present as an intracavitary mass which usually involves the
ventricles?
a. Hemangioma
29. Cytoplasmic vacuolar changes myofibrillar loss and myocyte necrosis are
characteristic features of myocardial doxorubicin toxicity
a. True
b. False
30. A positive immunostain for kappa light chain and a negative immunostain for

transthyretin are diagnostic ofAL type amyloid


a. True
b. False
31. Primary and secondary forms ofhemosiderosis are readily distinguished from
one another by light and EM
a. True
b. False
32. Which tumor is the most common primary tumor of the vana cava?
a.leiomyosarcoma

33. The most common tumor ofthe heart in an adult


a. Cardiac myxoma
34. The most common tumor ofthe heart in a child is
a. Cardiac rhabdomyosarcoma
35. The most likely diagnosis in a young woman with spotty skin pigmentation a
cardiac tumor adrenal hyperplasia and similar signs and symptoms in a family

member is
a. Familial myxoma syndrome
36. Which vasculitic disease is commonly associated with a history of asthma?
a. Granulomatous cholecystitis associated with Churg-Strauss syndrome
37. A histologic picture ofacute and healing arteritis with adjacent segments of
normal arteries and sparing ofveins and capillaries is considered characteristic
ofwhich disease?
a. Polyarteritis nodosa
38. The triad ofnecrotizing granulomatous lesions ofthe upper and lower
respiratory tract, focal segmental glomerulonephritis and necrotizing vasculitis
ofsmall arteries and veins is characteristic ofwhich entity?
a. Wegner's granulomatosis
39. Which disease is characterized by seriginous areas ofnecrosis with numerous
multinucleated histiocytes?
a. Giant cell myocarditis
40. Which disease is characterized by an infiltrate ofhistiocytes and eosinophils
around vessels and in cardiac connective tissue planes?
a. Hypersensitivity / eosinophilic myocarditis
41. Which disease is characterized by the presence ofcardiac mural thrombi?
a. Hypereosinophilic syndrome
42. The most common cardiac tumor is
a. Metastatic neoplasm
43. The most common benign primary cardiac tumor is
a. Myxoma
44. A primary malignant cardiac tumor that usually occurs on the right side ofthe
heart is

a. Angiosarcoma
45. Which tumor typically shows characteristic bi-polar projections in peripheral
blood?
.aS plenic marginal zone lymphoma
46. Which of the following stains is most specific for Kaposi sacome
a. LANA1 (same as HHV8)
47. Which one ofthe following markers is expressed by cystic tumor of the
atrioventricular region?
a. B72.3 (tumor-associated glycoprotein 72)
48. Which one ofthe following antigens is most specific for lymphatic endothelial
cells?
a. D2-40
49.T he organism responsible for causing Chagas disease?
a. Cytomegalovirus
b. Epstein Barr virus
c. Listeria monocytogenes
d. Tinea Cruris
e. Trypanosoma cruzi

50. The chronic phase ofT. cruzi infection is characterized by all the following
except?
a. Cardiac aneurysm
b. Dilated colon
c. Few intracellular amastigote forms
d. Parasitemia
e. Spans many years duration
51. Immunosupression is a risk factor of systemic infection by which ofthe
following ?
a. Candida
b. Cytomegalovirus
c. Toxoplasma gondii
d. Trypanosoma cruzi
e. All of the above

178. Skeletal Muscle


1.

2.

ATPase in neuronal atrophy in muscles


a.

Group atrophy

b.

Type two fibers

c.

Internalization of nucleus

Which ofthe following muscle disease shows positive ATPase stain


a. Myotonic dystrophy
b. Duchenne muscular dystrophy
c. Degenerative disease
d. Mitochondrial myopathy

3.

179. Central Nervous System and


Pituitary
1.

Which of the following tumors is associated with calcification

2. Destruction ofthe ventromedial (?) nucleus ofhypothalamus is associated with


3. Prolactin secreting adenoma presents usually as
4. Which aneurysm is associated with hypertension
a. Charcot Bouchard aneurysm
b. Atherosclerotic
c. Saccular
d. Berry
5. Most common lymphoma ofthe brain is
a. Diffuse large B cell lymphoma
6. Which of the following is grade Imeningioma
a. Papillary
b. Microcystic
c. Rhabdoid
d. Clear cell
e. Chordoid
7. 50 year old male patient experienced sudden acceleration ofdementia and
died 10 days later. Autopsy revealed spongiform encephalopathy. Which the
mechanism
a. Prion protein misfolding
8. Patient with multiple skin nodules, high urine catecholamines, 4 cm mass in the
temperoparietal area of the brain and an adrenal mass
a. NF-1
b. N-Myc
c. P53

d. KRAS
e. Rb
9. Familial history with multiple meningiomas
a. NF2
b. Li-Fraumani syndrome
c. NF1
d. Tuberous sclerosis
e. VHL
10. Which of the following familial syndromes show in the patient
hemangioblastomas in the retina and brain, cysts in the kidney, pancreas, liver,
and renal cell carcinoma
a. VHL
11. Which is the commonest glioma in the brain?
a. Oligodendroglioma
b. Low grade astrocytoma
c. Glioblastoma multiforme
d. Pilocytic astrocytoma
12. The fried egg appearance ofoligodendrocytes represents what
a. Cell cytoplasm
b. Delayed fixation artifact
c. Lipid in the cytoplasm
d. Glycogen in the cytoplasm
13. CSF shows increased lymphocytes in which condition
a. Viral meningitis
b. Bacterial meningitis
14. Middle meningeal artery injury leads to
a. Epidural hemorrhage
b. Subdural hemorrhage

c. Intracranial hemorrhage
15. Rupture ofberry aneurysm causes
a. Subarachnoid hemorrhage
16. What is the most common presentation ofpilocytic astrocytoma
a. A child with a well-circumscribed cystic tumor in the cerebellum
17. A child with a history ofvomiting and cerebellar tumor most likely has
a. Medulloblastoma
b. Astrocytoma
c. Oligodendroglioma
d. Schwannoma
18. Which genetic mutation makes pituitary adenoma more aggressive
a. P53
b. RAS
c. P16
d. N-myc
e. Deletion of3p
19. All of the following are features ofatypical meningioma except
a. Brain invasion
20. Regarding juvenile pilocytic astrocytoma, all of the following are true except
a. Never occur in younger than 20 years
21. What is the characteristic finding of the brain of an old lady with alzheimer's
disease
a. Atrophy
b. Hyperplasia
c. Hypertrophy
d. Dysplasia
e. Anaplasia

22. What is seen in Parkinson's disease


a. Lewy body
b. Negri body
c. Neuritic plaques
23. Which of the following is the most common tumor in children in the
cerebellum
a. Pilocytic astrocytoma
b. GBM
c. Ependymoma
d. Neuroblastoma
24. Which is most likely to be seen in NF1
a. Plexiform neurofibroma
25. Which brain tumor shows macroscopic edema
26. What is the most common location ofAT/RT?
a. Posterior fossa
b. Lateral ventricle
c. 3rd ventricle
27. Which immunostain can differentiate between AT/RT and choroid plexus
carcinoma?
a. EMA
b. Keratin
c. Vimentin

d. Synaptophysin
e. GFAP
28. All ofthe following are features of GBM, except
a. Necrosis
b. Hemorrhage
c. Vascular proliferation
29. What is the translocation ofoligodendroglioma
a. 1p 19q
30. Which of the following features of glial tumors is associated with a good
prognosis
a. Cells with hair-like processes
b. Cystic changes
c. Necrosis
d. Pseudopalisading
e. Vascular proliferation
31. A small dural lesion is resected and is composed ofepithelioid cells with
prominent eosinophilic cytoplasm. Some cells have single vacuoles resembling
signet ring forms while other cells have bubbly multivacuolated cytoplasm.
These cells are present in amucoid matrix. The lesional cells express brachyury
EMA CK and S100 but do not express D2-40 or GFAP. Which ofthe following is
the best diagnosis
a. Chordoid glioma, WHO grade II
b. Chordoid meningioma WHO grade II
c. Chordoid sarcoma
d. Intradural chrodoma
e. Metastatic mucinous adenocarcinoma
32. A 45 year old man presents with a radiographic circumscribed non-infiltrating
third ventricle tumor that is resected. It is composed ofepithioid cells forming

cords and nests within a mucinous stroma that has a prominent


lymphoplasmacytic infiltrate. Surrounding non-tumoral tissue exhibits piloid
gliosis. There is no mitotic activity and tumor cells express CD34 (membranous)
and GFAP. Tumor cells do not express EMA, Ck, CD18 or brachyury. Which of
the following is the best diagnosis
a. Chordoid clioma, WHO grade II
b. Chordoid mengioma, WHO grade II
c. Chordoid sarcoma
d. Intradural chordoma
e. Metastatic mucinous adenocarcinoma
33. A 9 year old girl presents with a dura based 2.4 cm lesion that is resected. It is
composed ofepithelioid cells some containing cytoplasmic vacuoles forming
loose nests and cords in a faint basophilic mucoid background. There is no
mitotic activity or necrosis present. Tumor cells express D2-40, EMA, and
weakly express S100 and CK. Tumor cells do not express GFAP or brachyury.
There is a mononuclear lymphoid infiltrate present that expresses CD79a and
CD138. What additional clinical feature might be expected to be present in this
patient pre-operatively
a. Hypercalcemia
b. Hypercoagulable state
c. Hypoglycemia
d. Microcystic hypochromic anemia
e. Polycythemia
34. The most commonly involved anatomic location for gliomatosis cerebri
a. Basal ganglia
b. Cerebellar vermis
c. Cerebral white matter
d. Hippocampus
e. Optic nerves

35. How is gliomatosis cerebri currently classified?


a. Extensively infiltrative glial neoplasm
b. Highly reactive inflammatory condition
c. Low-grade glioma
d. Neurodegenerative disorder
e. Uniformly fatal infectious condition
36. The single key histologic feature that allows accurate diagnosis is which ofthe
following?
a. Cytologic atypia
b. Demyelination
c. Microglial clusters
d. Spongiform change
e. Viral inclusions
37. Which are the most common systemic extracranial sites of metastasis in CNS
hemangiopericytomas?
a. Bones, lungs, liver
b. Heart, lungs
c. Kidney, lungs
d. Skin, lungs, adrenals

e. Spleen, lungs, adrenals


38. Which marker shows immunoreactivity in tumor cells in both solitary fibrous
tumor and hemangiopericytoma?
a. CD31
b. CD34
c. GFAP
d. CK7
e. Inhibin
39. Which is true regarding recurrences for this tumor
a. Tumor almost never recurs locally after surgical excision
b. Recurrences tend to occur within months after surgery
c. Recurrent tumors are associated with hyperostotic bone lesions
d. Greater than 80% oftumors recur on long-term follow up
e. Recurrent tumors are typically heavily calcified
40. Pituitary adenoma always requires postoperative adjuvant radiotherapy
a. True
b. False
41. Which is most typical for null cell adenomas of the pituitary gland
a. Brisk mitotic activity
b. Frequent recurrence
c. Markedly elevated MIB-1 labeling
d. Patchy immunoreactivity for leutenizing and follicles stimulating hormones
e. Strong diffuse immunostaining for TP53

42. The diagnosis of pituitary carcinoma requires


a. Cavernous sinus invasion
b. Elevated MIB-1 cell cycle labeling index
c. Elevated mitotic rate
d. Metastasis
e. Microscopic dural invasion
43. Which entity is characterized by a GFAP -, EMA + , vimentin + , keratin -, S100
phenotype?
a. Chordoid glioma
b. Chordoid meningioma
c. Chordoma
d. Mucinous adenocarcinoma, metastatic
e. Myxoid chondrosarcoma
44. Which entity is characterized by a GFAP + , EMA -, keratin -, S100 phenotype?
a. Chordoid glioma
b. Chordoid meningioma
c. Chordoma
d. Mucinous adenocarcinoma, metastatic
e. Myxoid chondrosarcoma
45. Which entity is characterized by abundant mucoid matrix with variable
cellularity and scattered cells showing abundant vacuolated cytoplasm
(physaliferous cells)
a. Chordoid glioma
b. Chrodoid meningioma
c. Chordoma
d. Mucinous adenocarcinoma, metstatic
e. Myxoid chondrosarcoma
46. All of the following are true ofparaganglioma of the cauda equine, except:
a. Cytoplasm oftumor cells is GFAP positive
b. May be CK positive
c. May show ganglionic differentiation
d. Sustentacular cells are S100 positive
e. Ultrastructural features show neurosecretory granules

47. Which of the following characteristics are unique to paragangliomas of the


cauda equine?
a. EM shows filament skeins, intercellular microvilli, and numerous zonula
adherens junctional complexes
b. May show ganglionic differentiation and CK distribution
c. May secrete neuropeptides
d. NSE is positive in a cytoplasmic distribution
48. Which of the following is true in distinguishing paraganglioma from
myxopapillary ependymomas?
a. Although both tumors may express GFAP, the stain shows predominance in
the sustentacular cells ofparagangliomas and the tumor cells of
ependymomas
b. EM of myxopapillary ependymomas shows neurosecretory granules
c. Ependymomas show a characteristic zellballen configuration
d. Myxopapillary ependymomas characteristically express synaptophysin and
chromogranin
49. Which of the following features is required for the diagnosis ofanaplastic
meningioma?
a. Brain invasion
b. Geographic necrosis
c. Greater than or equal to 20 mitotic figures / 10 hpf
d. Prominent nucleoli
50. Which of these meningioma variants is a WHO grade IIItumor?
a. Clear cell
b. Chordoid
c. Papillary
d. Secretory
51. Which of the following is typically seen in meningiomas?
a. CD34 positivity
b. EMA positivity
c. Pericellular reticulin
d. Strong GFAP reactivity
52. Which of the following is not significant in the diagnosis ofglioblastoma
a. Hich cell density

b. Mitotic activity
c. Necrosis
d. Vascular proliferation
53. Which molecular marker would most likely show an abnormality in recurrent
high grade astrocytoma with gemistocytic features
a. EWS-Fli1 gene fusion PCR
b. LOHofPTEN locus (10q.23)
c. MYCC amplification analysis
d. OTX2 amplifiction
54. What is the patient's expected 5 year survival when diagnosed with recurrent
high grade astrocytoma with gemistocytic features
a. 2%
b. 25%
c. 50%
d. 75%
55. Which of the following meningioma subtypes is considered to be WHO grade I?
a. Chordoid cmeningioma
b. Clear cell meningioma
c. Microcystic meningioma
d. Papillary meningioma
e. Rhabdoid meningioma
56. The estimated 5 year recurrence rate for totally resected WHO grade II
(atypical) meningioma?
a. 40%
57. Multiple meningiomas commonly accompany which ofthe following syndromes?

a. NF type 2
58. Patients with intravascular lymphoma present clinically with neurological signs
due to which ofthe following?
a. Hemorrhages
b. Infarcts
c. Intraparenchymal infiltration
d. Opportunistic infection
e. Vasculitits
59. Which of the following is the most important histologic finding in intravascular
lymphoma?
a. Distended vessels
b. Fibrin and platlet thrombi
c. Immunoreactivity ofneoplastic cells to factor VIII
d. Intravascular proliferation of large B cells
e. Perivascular inflammation
60. Which of the following is a characteristic feature of intravascular lymphoma?
a. Associated HIV positivity
b. Peak occurance in childhood
c. Predominantly cerebral and cutaneous involvement
d. Presentation with mass effect and enhancement on imaging
e. Relatively favorable prognosis
61. Which of the following favors a diagnosis ofmeningioma over meningeal
hemangiopericytoma?
a. Young age at presentation

b. Lytic bony involvement


c. Lack ofnuclear pseudoinclusions
d. Expression ofCD34 and absence of epithelial membrane antigen by
immunohistochemistry
e. Allelic loss on 22q in th region ofNF2
62. Which disorder is characterized by the perivascular accumulation of
multinucleated macrophages in the brain?
a. Subacute encephalitis of AIDS/HIV 1 infection of brain
63. Which disorder may cause myelitis with associated flaccid paraparesis?
a. West nile virus encephalitis
64. Which disorder is characterized by angiocentric inflammation with fibrinoid
necrosis ofvessel walls and thrombi?
a. 1ry angiitis of the CNS
65. Which is the most likely neoplasm when a patient with AML develops a nonhemorrhagic mass lesion in the brain?
a. Myeloid sarcoma (chloroma)
66. Specific immunohistochemical confirmation of which neoplasm is not currently
feasible?
a. Oligodendroglioma
67. Which brain neoplasm is most commonly encountered in patients with HIV1
infection?

a. Primary CNSlymphoma
68. In which lesion is allelic loss ofchromosomal arms 1p and 19q strongly
associated with chemosensitivity?
a. Oligodendroglioma
69. Which lesion features a columnar arrangement ofsmall round cells, delicate
fibrillary processes, and fine capillaries surrounded by pools ofmucin with
"floating" neurons?
a. Dysembryoplastic neuroepithelial tumor
70. Which lesion is associated with JC virus infection?
a. Progressive multifocal leukoencephalopathy
71. The four most common solitary circumscribed tumors ofthe lumbar cistern
are
a. Schwannoma, meningioma, myxopapillary ependymoma, paraganglioma
72. Which of the following is most often positive in myxopapillary ependymoma?
a. S100
73. The most helpful ultrastrucutral failure in the diagnosis of myxopapillary
ependymoma would be the presence of
a. Intercellular lumens with microvilliand/or cilia
74. Metastatic carcinoma, glioblastoma, cerebral abscess, primary central nervous
system lymphoma and demyelinating pseudotumor all may share which of the
following appearances on MRI
a. Solitary contrast enhancing ring lesion
75. An important pathophysiologic mechanism in cerebral abscess
a. Loss ofintegrity ofa normal physical barrier against infection
76. Granular mitoses and creutzfeldt astrocytes are characteristic histologic
features that are most closely associated with
a. Demyelinating disease

77. Which immunostain would be most helpful in the diagnosis ofcentral


neurocytoma?
a. Synaptophysin
78. The most helpful ultrastructural feature in the diagnosis of central
neurocytoma is the presence of
a. Neurosecretory granules
79. The most appropriate treatment for central neurocytoma?
a. Gross total resection alone
80. Which of the following procedures would be most helpful in the diagnosis of
PML?
a. In situ hybridization for JC virus
81. Progressive multifocal leukoencephalopathy is ost frequently associated with
a. Immunosuppression
82. The typical clinical course ofPML is
a. Relentlessly progressive over several months
83. Which of the following is not characteristic ofpilocytic astrocytoma?
a. Cerebellar location
b. Childhood presentation
c. Circumscription
d. Cyst formation
e. Tendency for malignant for malignant transformation
84. Histologic features of PA include
a. Rosenthal fibers
85. An immunestain that may be most helpful in differentiating PA from other
astrocytic neoplasms is
a. Neurofilament protein
86. Multiple lobes of the brain are diffusely involved by neoplastic astrocytes in
which condition?

a. Gliomatosis cerebri
87. Relative axonal preservation is a characteristic feature ofwhich condition?
a. Multiple sclerosis
88. Pseudopalisading with central necrosis is the classic histologic finding in wwhich
lesion?
a. GBM

180. Cytology
1.

What is a common contaminant for transthoracic FNA


a. Liver
b. Colon
c. Nerves
d. Pancreas
e. Stomach

2. In thyroid FNA, according to Bethesda, which is considered insufficient


a. 7 groups of follicular cells, each containing 10-12 cells
b. Only colloid, no follicular cells
c. 2 groups of follicular cells with foamy macrophages
d. Hurthle cells with lymphoid aggregates
e. 3 groups of follicular cells only
3. Woman had Pap which showed spider cells, no increase NC ratio, regular
nuclear membranes, what does she have
a. Squamous metaplasia
b. Adenocarcinoma in situ
c. Adenocarcinoma
4. What is the management for a patient who has ASC-Hon Pap
a. Colposcopy
b. Repeat Pap in 4-6 months
c. Repeat Pap after treatment with estrogen
d. LEEP
e. Test for high risk HPV
5. FNA of a 2x3 cm lung mass shows uniform cells in dyscohesive sheets and
clusters and single cells, with coarse evenly spaced chromatin, inconspicuous

nucleoli, no cilia or terminal bars


a. Carcinoid
b. Squamous cell carcinoma
c. Basal cell carcinoma
d. Small cell carcinoma
6. Common fungal contaminant ofCSF samples
7. To judge sputum adequacy
a. Carbon stained macrophages (alveolar macrophages)
b. Ciliated bronchial cell
c. Type I pneumocyte
d. Clara cell
e. Goblet cell
8. What is abnormal in urine cytology
a. Sperm
b. Squamous cells
c. Renal tubular epithelium
d. Multinucleated histiocytes
9. In sputum, Creola bodies are found in
a. Aspergillosis
b. Sarcoidosis
c. Squamous cell carcinoma
d. Adenocarcinoma
10. Which of the following is not found in FNA ofa patient with sarcoidosis
a. Giant cells
b. Schumman bodies
c. Creola bodies
d. Asteroid bodies
e. Epithelioid cells
11. Which of the following findings help in diagnosing aspergillosis
a. Creola bodies
b. Asteroid bodies
c. Oxalate

12. Ferruginous bodies suggest exposure to


a. Pollen
b. Iron
c. Asbestos
d. Lead
e. Aspergillosis
13. What is the most common easy to diagnose disease in lymph node FNA
a. Metastatic carcinoma
b. Hodgkin lymphoma
c. Non-hodgkin lymphoma
d. Reactive lymph node
e. Follicular lymphoma
14. Which of the following features is seen in invasive carcinoma on cytology
a. Prominent nucleoli
b. Hyperchromasia
c. Naked nuclei
d. Small clusters of hyperchromatic nuclei
15. In the cytology smear, which of the following does NOt show peri-nuclear halo
a. HPV
b. Pregnancy
c. Treatment with estrogen
d. Trichomonas vaginalis
e. Chlamydia trachomanis
16. All of the following show clusters ofsmall hyperchromatic nuclei EXCEPT
a. LSIL
b. HSIL
c. Atrophy
d. Endometrial cancer
e. In situ cancer
17. Female with watery vaginal discharge, PAP shows clusters of malignant
glandular cells in a clean background
a. Fallopian tube carcinoma

b. Transition zone carcinoma


c. Endometrial carcinoma
d. Endocervical carcinoma
18. All of the following features favor HSIL over LSIL EXCEPT
a. Smaller cell
b. High NC ratio
c. More coarse chromatin
d. More irregular nuclear membrane
e. High nuclear size
19. 30 year old female with a breast mass, FNA showed 3D clusters with high NC
ratio and nuclear atypia along with bipolar naked nuclei in the epithelial groups.
What is the diagnosis
a. Fibroadenoma
b. Ductal carcinoma
c. Lobular carcinoma
20. Urine cytology show multinucleated giant cells
a. BCG therapy
b. Urothelial carcinoma
c. Cystitis
21. Regarding regressive Pap stain
a. Nuclei overstained
b. Nuclei understained
c. Normal staining
d. Used with orange B stain
22. Regarding Diffquick
a. Cytoplasm details and background

b. Good for nuclear features


c. Not used for hematological cancers
d. Formalin fixed
e. Takes a long time to prep
23. A pregnant woman, pap smear shows multinucleated cells with central nuclei
and has a cytoplasmic tail, what is the diagnosis
a. Syncitiotrophoblasts
b. Cytotrophoblasts
c. Squamous cell carcinoma
d. Choriocarcinoma
24. Cervical Pap smear shows columnar cells with hyperchromatic nuclei,
anisocytosis and intact cilia with centrally located nuclei
a. Tubal metaplasia
b. Squamous cell carcinoma
c. Adenocarcinoma
25. What is the commonest cause of interobserver disagreement on pap smears
a. ASCUS
b. HSIL
c. LSIL
d. Adenocarcinoma
26. A 40 year old female underwent cervical pap smear which showed group of
hyperchromatic atypical cells with lots of dirty necrosis and apoptotic bodies.

Her previous colposcopy was normal. What is the most likely diagnosis
a. Colon
b. Breast
c. Vagina
d. Liver
e. Kidney
27. Cervical pap smear ofa lady showed small clusters ofhyperchromatic nuclei
and scant cytoplasm. All ofthe following will show this feature except
a. LSIL
28. Which of the following will help in differentiating between atypical
regenerative cells and squamous cell carcinoma
a. Presence ofnucleoli in the squamous cell carcinoma
b. Irregular coarse chromatin
c. Polarity in carcinoma
d. Enlarged nuclei
29. Which of the following features are NOT present in carcinoid tumor in a smear
a. Nucleoli
30. FNA cells showing margination, molding, and multinucleation
a. Herpes
31. Cytology air-drying artifact
a. Cellular swelling
b. Nuclear enlargement
c. Nuclear hyperchromasia
32. FNA showing tumor cells arranged around cylindrical hyaline material
a. Adenoid cystic carcinoma

181. Grossing, Lab Procedures, and


Others
1.

To ensure that no exogenous tissue contaminates the block or slide, what


should be done
a. All instruments washed properly after each use
b. Clean microtome blade every morning
c. Clean waterbath at the end of each day

2. The turn around time (TAT) for the frozen section according to CAP
a. 90% offrozen section block must be completed by 20 min
b. 100% ofthe frozen section blocks must be completed by 20 min
c. 90% ofthe frozen section blockes must be completed by 10 min
d. 50% ofthe frozen section blocks must be completed by 20 min
3. Rhodanine stain is used for
a. Copper
b. Zinc
c. Lead
d. Iron
e. Fat
4. Which fixative is used to dissolve fat in lymph node dissection specimens
a. Bouin
b. Zenker's
c. Formalin
5. Which tumor shows abundant glycogen on EM?
a. Ewing's sarcoma
b. Mesothelioma
c. Neuroblastoma
d. Alveolar soft part sarcoma
e. Rhabdomyosarcoma

6. Renal tumor showing 150-200 nm vesicles in the cytoplasm on EM, which stain
is positive
a. Coilloidal iron
7. Renal tumor which shows lysosomes on EM
a. Chromophobe RCC
b. Clear cell RCC
c. Papillary
d. Oncocytoma
8. A boy is diagnosed with osteosarcoma and is given radiotheraoy and then has
an amputation. What do you comment on to show response to treatment
a. Extent oftumor necrosis
9. What is optional in breast reporting
a. Perineural invasion
b. Histologic type
c. Lymphovascular invasion
d. Number oflymph nodes involved
e. Size of the tumor
10. Which of the following shows long thin cylindrical microvilli on EM?
a. Mesothelioma
11. Which renal tumor has the greatest number ofmitochondria on EM?
a. Oncocytoma
12. What is the EM finding in a case ofrenal oncocytoma
a. Mitochondria
13. On EM, which renal tumor arises from the intercalated cells and has
microvesicles
a. Chromophobe RCC
14. What is the embedding media for EM sections
a. Resine
b. Paraffin
c. Non-paraffin wax

d. Gelatin
15. What is the fixative for EM specimens
a. Glutaraldehyde
16. Most common source ofcontamination (external tissue) in the slides comes
from
a. Waterbath
b. Knife carry over
c. Embedding
d. Cover slip
17. Red bag for the disposal of
a. Biological waste products and contaminated containers
b. Sharp objects
c. Biologically contaminated towels, papers, gowns, and gloves
d. Empty biomedical containers
e. Confidential reports
18. What should immediately be done if one injures himself during grossing
a. Squeeze out the blood and wash with water
b. Go to ER immediately
c. Write an incident report
d. Cover the wound
19. What is the first thing you do if you have a needle stick injury
(same options and answer as above)

20. What is the most common cause oferror in diagnosis in anatomic pathology
reports
a. Overwhelming workload
b. Incomplete history and clinical data
c. Inadequate sampling
d. Improper grossing and processing
e. Inadequate processing of the specimen
21. Which study could be either retrospective or prospective and the investigators
are interested in the rare exposures
a. Cohort study
b. Case control study
c. Prevalence study
d. Interventional trial
22. Pre-analytical review of control management of immunohistochemistry
a. New reagents lots
b. Antibody validity
c. Procedure protocol
d. Tissue fixation
23. Calculate the mean ofthe following weights
a. [sum ofweight/no. ofpatients]
24. Investigators want to study the effect ofexercise program on death, they
found 30 deaths in 100 persons in the control group (not taking the exercise

program) and 50 deaths in 100 persons in the study group (taking the exercise
program) what is the relative risk ofdeath in the study group compared to the
control group
a. [30/100 50/100]= 0.6
25. The benefit of literature review in the study
a. Precision
b. Applicability
26. P. value significant if
a. > 0.05
27. Pre-analytic variable of quality improvement plan
a. Specimen submission
28. What will you do if a patient who has already consented to FNA comes to you
but then refuses to do it
a. Try to convince the patient
b. Leave the room then come back
29. Which stain is used for lipid
a. Sudan black
30. When is inking a specimen not important
a. Ovary surface
31. When is gross examination only enough
a. Bone fragments in nonunion fracture
b. Spleen in sickle cell anemia
c. Placenta for eclampsia
d. Adenoids for a 40 year old
e. Abdominal scar
32. Which term shouldn't be used during grossing
a. Lymphatic invasion
33. Snap freezing can be used for all the following cases except
a. EM
b. H&E

c. Muscle/nerve biopsy
d. Frozen section
e. Flow cytometry
34. EM is a good diagnostic tool for all ofthe following cases, except
a. Amyloid
b. Whipple disease
c. Renal biopsy
d. Ciliary dysmorphology (Kartagener syndrome)
35. The optimal freezing temperature for breast tissue (frozen section)
a. -20
36. The reason of negative immunostaining of the tissue at the center, while the
periphery is positive
a. Fixation problem
b. Antigen retrieval
37. The reason for black pigmentation artifact in the slide
a. Formalin artifact
38. The reason for bubble artifact
a. Coverslip
39. Advantage offrozen section
a. It may change the intra-operative management
b. Give final diagnosis
40. Advantage ofsynoptic reporting
a. Include diagnosis, prognostic factors, grading and staging
b. Delay the turn around time
c. May lead to independent thinking
41. What is the name of the process of reviewing and checking weekly on the
temperature ofthe water bath
a. Quality control
b. Quality assurance
c. Quality improvement
d. System review

42. Review concordance between frozen section diagnosis and permanent


diagnosis is
a. Quality assurance
b. Quality control program
c. Quality improvement program
d. System review
43. Disconcordance between frozen section diagnosis and permanent diagnosis is
most commonly due to (or most common cause for frozen section mistake is)
a. Misinterpretation
b. Improper specimen sampling
c. Improper number of blocks
d. Mislabeled sample
44. Discrepancy allowed between the frozen section diagnosis and final diagnosis
a. Less than 2%
b. 5% or less
c. 0%
d. Less than 10%
45. Routine biopsy requires no more than
a. 2 days
46. What stain is used to differentiate between hemosiderin and lipofuscin
a. Iron
b. Calcium
c. Mucin
d. Sodium
47. For proper specimen handling
a. Refuse unlabeled specimens
48. What are the steps in specimen processing
49. Frozen section is least likely used for
a. Evaluation ofmargins
b. Evaluation ofgrade
c. Adequacy ofsample for permanent section

d. Differentiating between lymph nodes and parathyroid


50. What is the initial step in total nephrectomy specimen
a. Locate vascular and ureter margins
b. Look for the adrenal gland
c. Take from Gerota's fascia margin
51. Patient with a diagnosis ofcomplex endometrial hyperplasia with atypia cannot
rule out carcinoma had a hysterectomy and it was sent fresh, why?

a. To send fresh tissue for EM


b. To send fresh tissue for molecular studies
c. To cut open and fix
d. To assess depth ofinvasion
e. To perform a frozen section on one section taken from the lesion
52. Which can be used to help with orientation of the uterus
a. Round ligament
53. In ovarian neoplasm, it is important to comment on what grossly
a. Intact or ruptured capsule
54. Total neck dissection which is used as a landmark for orientation
a. Internal jugular vein
55. Where do you search for lymph nodes in cystectomy specimens
a. Perivesical soft tissue
56. Supraglottic components of larynx include all the following except
a. Anterior commissure
57. Birbeck granules are seen in
a. Langerhan cell histiocytosis
58. Lip grossing (how best to submit lip specimen)
a. Cut perpendicular to the free edge margin from side to side
b. Parallel to free end one section
c. Perpendicular to free end one section
59. In lung grossing, the following is important for staging purposes
a. Measure the distance between the tumor and the bronchus
60. Grossing ofendometrial curetting
a. Submit the entire specimen
61. Grossing ofWhipple procedure
62. Grossing ofliver biopsy (?)

63. EM of myocardium
a. Intercalated discs
64. EM finding oflipid body
65. Which tumor shows rhomboid crystals on EM
a. Alveolar soft part sarcoma
66. Know EM of
a. Fibrosarcoma
b. Leiomyosarcoma
c. Liposarcoma
d. Ewing sarcoma
e. Mesothelioma
f.

Adenocarcinoma

g. Alveolar soft part sarcoma


67. Which tumor shows pre-melanosomes by EM
a. Angiomyolipoma
68. EM of which tumor shows cytoplasmic filaments, lipid bodies, pinocytic
vacuoles
a. Leiomyosarcoma
b. Rhabdomyosarcoma
c. Adenocarcinoma

d. Squamous cell carcinoma


e. Liposarcoma
69. Pathologist A was reviewing a carcinoma ofbreast case ofPathologist B to
present it in the tumor board. He came across a lymph node showing cluster
ofatypical cells in the subcaprular area. According to Bethesda 2001 what is
the most appropriate action to be taken by Pathologist A
a. Defer the case and discuss it with Pathologist B
b. Ignore the whole story
c. Do IHC and report the case
d. Inform the chairman ofthe department
e. Show the case in the tumor board
70. Ifyou receive a slide for second opinion from outside without patients
identification. according to Bethesda 2001, what is the most appropriate action
to be taken
a. Reject the slides and do not process them
b. Call the physician and retrieve patient's data and identification
c. Process the slides and don't release the report
d. Process and report the case as any other case
71. Which adrenal tumor shows neurosecretory granules on EM
a. Pheochromocytoma
72. MSDSstands for
a. Material safety data sheet
73. Benefit of flow cytometry
a. S-phase fraction
74. Important elements of the final pathology report for RMS resected following
treatment include?
a. Presence and extent ofcytodifferentiation

b. Assessment of resection margins


c. Measurement oftumor size in three dimensions
d. Comparison ofpre and post treatment histologic features
e. All of the above

182. Forensics
1.

Raccoon eyes are associated with


a. Fracture ofthe base ofthe skull

2. A 40 year old male patient was brought to the hospital after he has been
burned in a fire up to 20% of his body surface area involving mainly his
upper limbs. Three weeks later he dies in the hospital due to
a. Extensive burns
b. Full thickness burns
c. Inhalational injury
d. Prolonged hypoxia
3. A young lady has visited her 60 year old mother at her home and she was
completely fine and fit. Four hours after she leaves, a police officer calls
her and informs her that her mother was found dead in the bathtub.
Autopsy showed subarachnoid hemorrhage and inferior frontal bruises.
What is the most likely cause ofher death
a. Fall
b. Drowning
c. Electric shock
d. Strangulation
e. Homicidal
4. Forensic expert receives a dead body ofa man with a 1 cm opening in his
forehead. The surrounding skin shows small black stippling measuring 6 cm
a. Intermediate range gunshot
b. Close contact gunshot
c. Blunt injury

d. Stab wound by knife


5. A smoker, known case ofGBM, visits his physician and he is well and fit.
Seven days later he dies and the same physician is called to examine him
and write the death certificate. What is the most appropriate thing
a. Smoking is written in other conditions
b. The immediate cause ofdeath is cardiac arrest
c. The underlying cause ofdeath is GBM
6. Fetus with recent intrauterine demise will show
a. Desquamation
b. Cryoposis
c. Mummification
d. Umbilcal cord knot
7. How do you know that a deceased person was alive in the burn?
a. Alveolar injury or smoke inhalation
8. Hanging leads to death by
a. Cervical spine fracture
9. Cyanide poisoning
a. Pink cherry red skin
10. At autopsy, conjunctival petechiae is seen in all of the following situitation
except
a. Frequently seen in normal adults
11. A man involved in RTA suffers multiple fractures and dies 48 hours later
after he develops pulmonary insufficiency, neurological symptoms, anemia,
and thrombocytopenia. What is the cause ofdeath
a. Air embolism
b. Disseminated intravascular coagulation
c. Fat embolism
d. Pulmonary thromboembolism
e. Systemic arterial thromboembolism
12. At autopsy of an alcoholic, a large amount of blood is found in the upper GI
tract along with hepatic cirrhosis. The gastric and duodenal mucosa are

both normal. What is the source ofthe bleed


a. Angiodysplasia
b. Crohn's disease
c. Jejunal diverticulum
d. Esophageal varices
e. Trauma to the mouth

Você também pode gostar